N487 Leadership in Nursing: NCLEX Quiz Review - ALL QUESTIONS

¡Supera tus tareas y exámenes ahora con Quizwiz!

An infant has an active acquired immunity. Which statement best explains this type of immunity? 1. The infant has received immunizations. 2. Immunity was transferred from the mother to the infant. 3. The infant is recovering from a childhood disease that conferred immunity. 4. The infant has received gamma globulin after exposure to hepatitis.

1 Active acquired immunity occurs when a client (infant, child, or adult) receives an immunization against a specific disease. Natural active immunity occurs when the client has had the disease. Natural passive immunity occurs with transfer of antibodies from the mother to the infant at birth or through breast milk. Passive artificial immunity occurs with injection of gamma globulins; the response is immediate but short term. (Lewis et al., 10 ed., p. 192)

The nurse is preparing to administer spironolactone to a client. After assessing the client, what data indicate the need to withhold the medication? 1. Potassium level of 5.8 mEq/L (mmol/L) 2. Apical pulse rate of 58 beats/min 3. BP of 130/90 mmHg 4. Urine output of 30 mL/hr

1 Aldactone is a potassium-sparing diuretic. The client's potassium level is high; therefore the medication should be held and the doctor should be notified. Urine output of 30 mL/hr is normal output. The BP is elevated, which is the reason the client is receiving the medication. The pulse rate is not affected by this medication. (Igntavicius & Workman, 8th ed., pp. 262, 712, 714)

The nurse would question which medication order for a client with PACG (primary angle-closure glaucoma)? 1. Atropine 1 to 2 drops in each eye now 2. Hydrochlorothiazide 25 mg PO daily 3. Propranolol 20 mg PO two times a day 4. Carbamylcholine eye drops, 1 drop two times a day

1 Atropine causes mydriasis or pupillary dilation, which can precipitate an attack of acute glaucoma. It should be questioned if it is ordered for a client with glaucoma. The other drugs would be safe for a client with glaucoma. (Lewis et al., 10 ed., p. 379)

The vital signs of a client with cardiac disease are as follows: blood pressure of 102/76 mmHg, pulse of 52 beats/min, and respiratory rate of 16 breaths/min. Atropine sulfate is administered IV push. What nursing assessment indicates a therapeutic response to the medication? 1. Pulse rate has increased to 70 beats/min. 2. Systolic blood pressure has increased by 20 mmHg. 3. Pupils are dilated. 4. Oral secretions have decreased.

1 Atropine is administered for symptomatic bradycardia. An increase in pulse rate is the therapeutic response for this client. All other options (increase in BP, dilated pupils, and dry mouth) are characteristic of atropine but are not desired actions for this client. (Lewis et al., 10 ed., p. 763)

What symptoms would the nurse expect to observe in a 19-month-old client with a diagnosis of laryngotracheobronchitis (LTB)? 1. Stridor on inspiration 2. Expiratory wheezing 3. Paroxysmal coughing 4. Hemoptysis

1 Because croup causes upper airway obstruction, inspiratory stridor is a predominant symptom. Expiratory wheezing is heard in the asthmatic client. Paroxysmal coughing occurs more with spasmodic laryngitis. Hemoptysis is not common with croup syndromes. (Hockenberry & Wilson, 10 ed., p. 1186)

The nurse is caring for a client who is 6 hours postpartum. What nursing actions are directed toward the prevention of postpartum thrombophlebitis? 1. Encourage early ambulation and increased fluid intake. 2. Allow bathroom privileges only and elevate the lower extremities. 3. Administer anticoagulants and evaluate the clotting factors. 4. Encourage the client to breastfeed the infant as soon as possible.

1 Early ambulation is the most effective and safe way to prevent thrombophlebitis with any type of client. This promotes venous return and prevents venous stasis. Anticoagulants (heparin and warfarin) are administered as ordered postpartum with a diagnosis of thrombophlebitis; they are not used for prevention. The legs should be elevated when the client is in a sitting position. There is no evidence that breastfeeding affects blood coagulation in any way. (Ignatavicius & Workman, 8th ed., pp. 730-731)

The nurse is assessing a client with a tentative diagnosis of multiple sclerosis (MS). Which assessment finding would the nurse identify as characteristic of early signs of MS? 1. Diplopia 2. Resting tremor 3. Flaccid paralysis 4. Unilateral neglect

1 Early signs of MS include difficulty with fine motor movement, especially of the head and neck. Often, visual disturbance is the most ominous sign. Tremors, flaccid paralysis, and unilateral neglect are not seen in the client with MS. (Lewis et al., 9 ed., p. 1488)

What is the priority nursing action for the client who is complaining of nausea in the recovery room after gastric resection? 1. Evaluate the NG tube for patency. 2. Call the physician for an antiemetic order. 3. Place client in semi-Fowler's position so that he will not aspirate. 4. Medicate the client with a narcotic analgesic.

1 Evaluate the NG tube patency; it is important to prevent the nausea and vomiting. The next action would be to put the client in semi- Fowler's position. It is important to assess the client and take nursing measures to determine the source of the nausea and to decrease the nausea before calling the doctor. (Lewis et al., 10 ed., p. 918)

Which of the following nursing interventions should be instituted for a client experiencing a manic episode? 1. Place the client in a quiet area, separate from others. 2. Encourage the client to engage in some physical activity. 3. Establish firm, set limits on behavior. 4. Include the client in the group's activities.

1 For the client's protection, he should be moved to a quiet environment away from others. This will help him regain some control and will not produce unneeded stimuli. Setting firm limits would be appropriate for manipulative behavior. Until the manic behavior is under control, including a client in group activities (physical or otherwise) would not be therapeutic for him and would more than likely be disruptive to others. (Halter, 7 ed., p. 237)

A client is diagnosed with an immunodeficiency disease. The nurse would understand what is characteristic of this condition? 1. Occurs when a client's body is unable to defend itself from an invading microorganism 2. Creates a severe, sudden problem that is characterized by increased vascular permeability 3. Is precipitated by the destruction of the normal lymphocytes in the attempt to reduce the serum level of the antigen 4. Is a condition in which the normal immune response is interrupted and the body cells do not recognize healthy tissue

1 Immunodeficiency is the condition when the immune system is depressed, weak, or compromised and is unable to defend the body from invading microorganisms. Immunodeficiency may be primary if it is caused by an absence of immune cells or by poorly developed immune cells. It is secondary if it is caused by illnesses or treatment. A severe, sudden problem characterized by increased vascular permeability describes an anaphylactic reaction. The destruction of normal lymphocytes in the attempt to reduce the serum level of the antigen reflects phagocytosis as the white cells destroy the foreign protein. When the normal immune response is interrupted and body cells do not recognize healthy tissue, this is characteristic of an autoimmune condition. (Lewis et al., 10 ed., pp. 205-206)

What physical characteristics of a client would place the client at highest risk for development of malignant melanoma? 1. Light to pale skin, blond hair, blue eyes 2. Olive complexion, oily skin, dark eyes 3. Dark skin with freckles, dry flaky skin, hazel eyes 4. Coarse skin, ruddy complexion, brown eyes

1 People with light to pale skin and who are excessively exposed to sunlight are most at risk for development of malignant melanoma. Dark-skinned and olive-skinned individuals have more melanin in their skin, which provides a measure of protection from UV exposure. Although those with a ruddy complexion are more prone to the development of skin cancers, the coarseness of the skin does provide some protection from the sun's harmful rays. (Lewis et al., 10 ed., p. 411)

For a client with severe painful osteoarthritis, a regimen of heat, massage, and exercise will: 1. Help relax muscles and relieve pain and stiffness. 2. Restore range of motion previously lost. 3. Prevent the inflammatory process. 4. Help the client cope with pain effectively.

1 Physical therapy relaxes muscles and relieves the aching and stiffness of the involved joints. It usually does not restore lost range of motion, and it does not prevent inflammation. Physical therapy does make the client more comfortable, but it does not assist in coping with pain. (Lewis et al., 10 ed., p. 1470)

The nurse is encouraging a pregnant woman to eat a diet rich in folic acid. Which of the following food sources would provide the most folic acid? 1. Meat and dark green, leafy vegetables 2. Dairy products 3. Carrots and raisins 4. Shellfish

1 Rich dietary sources of folate are dark green, leafy vegetables, whole wheat bread, lightly cooked beans and peas, nuts and seeds, sprouts, oranges and grapefruits, liver and other organ meats, poultry, fortified breakfast cereals, and enriched grain products. Shellfish is rich in iodine. Dairy products are rich in calcium. (Lowdermilk et al., 11 ed., p. 345)

The labor monitor tracing shows variable decelerations. What complication would the nurse anticipate is occurring? 1. Cord compression 2. Fetal hypoxia 3. Placental insufficiency 4. Head compression

1 Variable decelerations most commonly occur during the transition phase of the first or second stage of labor as a result of umbilical cord compression or stretching during fetal descent and are correctable. Fetal hypoxia and uteroplacental insufficiency are associated with late decelerations. Head compression is associated with early decelerations. (Lowdermilk et al., 11 ed., p. 420)

A client in the emergency department has been hydrated with normal saline over the last hour for hypovolemia. Assessment changes now include a rapid bounding pulse and shortness of breath. What additional information would the nurse want to gather? Select all that apply. 1. Blood pressure 2. Level of consciousness 3. Urinary output 4. Hemoglobin level 5. Oxygen saturation

1, 2, 3, 5 The client who had been hypovolemic is not demonstrating assessment changes of fluid volume excess. Appropriate assessments include vital signs such as pulse oximetry, weight, urine output, and changes in level of consciousness. Hemoglobin would not be affected by fluid status. (Lewis et al., 10 ed., p. 277)

A nurse is caring for a client with Addison's disease who has been in a car accident and presents to the emergency department with severe hypotension, fever, weakness, and confusion. Place the nurse's action in a priority order. 1. Vital sign assessment. 2. Delivery of 0.9% saline and 5% dextrose solution. 3. Placement of an IV. 4. Delivery of high-dose hydrocortisone replacement. 5. Health history information.

1, 3, 2, 4, 5 (listed in order of priority) Addison's disease is due to a hypofunctioning of the adrenal cortex. This client is demonstrating signs of addisonian crisis. Since the client is known to have Addison's disease and is demonstrating signs of crisis, assessment of vital signs should be followed by the placement of an IV, IV fluids, and IV hydrocortisone to allow the body the hormone needed for the stress response. Circulatory collapse can occur in these clients, so there is urgency in the delivery of cortisol. They are often unresponsive to vasopressors and fluid replacement. (Lewis et al., 10 ed., p. 1178)

The nurse has been assigned a group of cardiac clients. What would be the most important information for the nurse to assess during the initial visit? Select all that apply. 1. Presence of cardiac discomfort 2. Medications taken before hospitalization 3. Presence of jugular vein distention 4. Heart sounds and apical rate 5. Presence of diaphoresis 6. History of difficulty breathing

1, 3, 4, 5 A focused cardiac assessment is directed toward assessing physiologic symptoms (cardiac pain, JVD distention, heart sounds and rate, presence of diaphoresis) that provide immediate information regarding the client's condition, which is appropriate for the nurse to do at the beginning of each shift. After the physiologic parameters have been evaluated, the nurse can determine any history of difficulty breathing and a list of medications the client was taking before admission. (Lewis et al., 10 ed., p. 663)

The nurse is preparing a client for a cardiac catheterization. Which nursing interventions are necessary in preparing the client for this procedure? Select all that apply. 1. Verify consent form has been signed. 2. Explain procedure to client. 3. Provide clear liquid, no caffeine diet. 4. Evaluate peripheral pulses. 5. Obtain a 12-lead ECG. 6. Obtain history for shellfish allergy.

1, 4, 6 In cardiac catheterization with angiography, contrast dye is injected into the coronary arteries, which allows visualization of the coronary arteries and provides information on patency. Informed consent is required before any invasive procedure. The physician is responsible for explaining the procedure to the client, and the nurse can reinforce the information. The client should be NPO for 6 to 18 hours before the procedure. A 12-lead ECG would be done, but this procedure is reflective of the conduction system, instead of the perfusion of the coronary arteries. Evaluating peripheral pulses is a nursing measure after the cardiac catheterization. It is important to check for iodine sensitivity or shellfish allergy because the procedure involves injecting contrast medium. (Lewis et al., 10 ed., pp. 677-678)

The nurse is evaluating a client recently diagnosed with primary open-angle glaucoma (POAG). What will be an important nursing action(s)? Select all that apply. 1. Review all medications the client is currently taking to determine whether any of them cause an increase in intraocular pressure as a side effect. 2. Determine whether the client has experienced any sudden loss of vision accompanied by pain. 3. Discuss with the client the importance of controlling blood pressure to decrease the potential loss of peripheral vision. 4. Instruct the client to take analgesics as soon as any discomfort occurs in the eye and to notify clinic if pain is not relieved. 5. Have the client demonstrate the use of eye drops. 6. Assess the client for chronic diseases such as diabetes.

1, 5, 6 Medications must be evaluated in terms of their potential for increasing intraocular pressure. Ophthalmic eye drops are often prescribed for the client with glaucoma, and clients should know how to administer them correctly. Diabetes is a risk factor for the development of glaucoma, and management of the diabetes is important in helping to prevent progression of the glaucoma. An increase in intraocular pressure could cause further damage to the eye in the client with glaucoma. The question states the client has already been diagnosed; primary open-angle glaucoma (POAG) is painless and is not correlated with the blood pressure. (Lewis et al., 10 ed., pp. 379-382)

A client with a diagnosis of type 2 diabetes has been prescribed a course of prednisone for severe arthritis pain. How should the nurse adjust the plan of care? Select all that apply. 1. Monitor blood glucose levels more frequently. 2. Monitor for signs of bleeding. 3. Monitor urine output every 4 hours. 4. Monitor for increased signs of infection. 5. Monitor for increased confusion.

1,4 An adverse reaction to corticosteroids is hyperglycemia. A client with type 2 diabetes must monitor blood glucose levels closely while taking steroids. Clients taking corticosteroids are also at increased risk for infection due to suppressed immune response and not a decrease in white blood cells. Bleeding, confusion, and changes in urine output are not typical a concern. (Lehne, 8 ed., p. 874)

A client is receiving IV antibiotic therapy. The order is for methicillin 750 mg IV. The nurse has a vial on hand that contains 1 g. The instructions for reconstitution say to add 1.5 mL sterile water. Reconstituted solution will contain 500 mg methicillin per milliliter. How much will the nurse give? Answer: __________ mL

1.5 mL Rationale: 500 mg : 1 mL :: 750 mg : x The dosage calculation cannot be made from the amount of solution added to the vial; the ratio of mg per mL after reconstitution is 500 mg per mL. (Potter & Perry, 9 ed., pp. 618-620)

A client has had a kidney stone removed, and the nurse instructs him in measures to decrease kidney stone formation in the future. Which statement by the client indicates to the nurse that he understood the teaching? 1. "I can continue to drink soda if it is sugar free." 2. "I should consume at least 3000 mL of fluid daily." 3. "I should report nocturia that occurs once a night." 4. "I will ingest megadoses of vitamins C and D daily."

2 A high fluid intake will help keep solutes diluted, thus preventing kidney stone formation. Drinking soda (diet or otherwise) is not the best method of increasing urine output. Nocturia and vitamins C and D are not relevant to kidney stones. (Lewis et al., 10 ed. pp. 1045-1046)

A woman explains to the nurse that she thinks she has been exposed to HIV. However, she had a test 1 week after the exposure, and the result was negative. What is most important for the nurse to explain to this client? 1. Make sure she understands the importance of safe sex practices, especially the use of condoms and contraceptive practices to prevent pregnancy. 2. Even though the client tested negative, she needs to have a series of follow-up blood tests because of the possibility of seroconversion. 3. It is important that she obtain counseling regarding the transmission of the virus and how she may protect herself and her partner. 4. The client should abstain from sexual activity for the next 3 months until the blood test confirms that she is negative for HIV.

2 After initial infection, there is a window of seroconversion in which the virus begins to replicate and produce antibodies. The client may have a negative test result early in the window. When the body begins to produce antibodies against the virus, the test result will convert to a positive. She should not get pregnant, but contraceptives (oral birth control) do not protect her against human immunodeficiency virus (HIV). Abstaining from sexual activity is frequently unrealistic, and counseling would be beneficial, but it is not the priority. Although emphasizing the importance of safe sex practices is correct, it is not the best response. The priority in this situation is the necessity for follow-up blood tests because of the initial negative test result. (Lewis et al., 10 ed., pp. 220-221; 225- 226.)

A client comes to the clinic with decreased hearing. Examination of the ear canal reveals a large amount of cerumen. What is the recommended method for removal of the cerumen? 1. Curettage with suction and irrigation 2. Warm sterile solution irrigation 3. Cool tap water irrigation 4. Cotton swab applicator

2 Although the structures of the outer ear are not sterile, sterile drops and solutions are used for irrigations in case the tympanic membrane is ruptured. The addition of nonsterile solutions may result in possible infections of the middle ear. Cool irrigants will be uncomfortable, and tap water is not considered sterile. Curettage with suction and irrigation and use of a cotton swab applicator can damage the tympanic membrane. (Lewis et al., 10 ed., p. 385)

When obtaining a health history, the nurse expects a client with a recent diagnosis of Parkinson disease to report which sign or symptom? 1. Weight loss 2. Slowness of movement 3. Continual motor tremors 4. Depression

2 An early symptom of Parkinson disease is slowness of movements in all normal ADLs. Tremors and weight loss may occur but are not commonly the first symptoms. Depression is a complication, along with other issues such as hallucinations, psychosis, and dementia, which can occur later in the disease process. (Lewis et al., 9 ed., p. 1434)

What nursing observation indicates that an unplanned extubation of an endotracheal tube has occurred? 1. The high-pressure ventilator alarm activates 2. Client is able to speak 3. Increased swallowing efforts by client 4. Increased crackles (rales) over left lung field

2 An unplanned extubation, which would be accidental removal of the endotracheal tube from the trachea, would allow air to pass through the trachea and vocal cords, allowing the client to make a noise—or to speak. Activation of the low-pressure alarm on the ventilator will be noted, along with diminished or absent breath sounds, signs of significant respiratory distress, and gastric distention. Increased swallowing is indicative of irritation in the throat or bleeding. Increase in adventitious sounds indicates excessive mucus in the lungs. (Lewis et al., 10 ed., p. 1573)

The nurse is discussing the importance of breast self-examination with a client who is being discharged after a vaginal hysterectomy. What is important information for the nurse to give this client? 1. Perform breast self-examination 1 week after her normal period. 2. Examine her breasts on a regular basis about the same time every month. 3. Breasts should be palpated while in the sitting position. 4. Use the tips of the fingers to palpate deeply into the breast tissue.

2 Because she no longer has regular periods, the client should pick a date and perform breast self-examination at the same time each month. Self-examination of the breast a week after the normal period is the best time for a woman who still has menstrual periods because the breast tissue is less glandular a week after the normal period. Breasts are examined lying down and standing, not sitting. The pads of the fingers are used to examine the breast using small circular motions in a spiral pattern or in an up-and-down motion. (Lewis et al., 10ed., p. 1205)

What will the nurse anticipate the neurologic nursing assessment of an 88-year-old client with a left cranial hemisphere hemorrhage to reveal? 1. Spasticity, bilateral Babinski sign 2. Right-sided flaccidity and hemiparesis 3. Spasticity and left foot clonus 4. Flaccidity and bilateral foot clonus

2 Because the motor fibers from one side of the brain cross to the opposite side before passing down the spinal cord, hemorrhage on the brain's left side causes right-sided hemiplegia and vice versa. Spasticity is not a common occurrence, and both sides of the body are not affected. (Lewis et al., 9 ed., pp. 1392-1394)

A client who has glaucoma is concerned about her adult children "inheriting" the condition. What is the best nursing response? 1. "There is no need for concern; glaucoma is not a hereditary disorder." 2. "Your children should have an ophthalmologic examination with screening for glaucoma around age 40. After that, examinations should be done every 2 to 3 years." 3. "There may be a genetic factor with glaucoma, and your children over 30 years of age should be screened yearly." 4. "Are your grandchildren complaining of

2 Blindness from glaucoma is preventable. A comprehensive eye examination should be done around age 40, then every 2 to 4 years until 64 years, and then every 1 to 2 years after age 65. There is a familial tendency and a significantly higher incidence in African Americans. Therefore African Americans should have ophthalmic examinations more frequently. (Lewis et al., 10 ed., pp. 380, 382)

The nursing discharge care plan for a 2-month-old infant in a Pavlik harness includes what nursing instructions? 1. Instruct the parents how to perform the Ortolani test daily. 2. Check at least two or three times a day for red areas under the straps. 3. Check with the physician about using a spica cast because the harness is not effective. 4. Adjust the harness straps if the parents think it is necessary.

2 Check for red areas under the straps at least two or three times a day. The Ortolani test is for diagnostic purposes and should only be performed by trained health care practitioners. The success rate of the Pavlik harness is about 95% when a Pavlik harness is used on a full-time basis for 6 weeks, and spica casting is unnecessary. Adjustment of the harness is done by a health care practitioner. (Hockenberry, 10 ed., pp. 423-424)

Which of the following statements by the client who has recently had a total hip replacement indicates that the client does not understand the mobility limitations? 1. "I should not bend down to put on shoes or socks." 2. "It is okay to cross my legs if I am sitting in a chair." 3. "I should put a pillow between my legs when lying on my side." 4. "I should not sit in low chairs or on toilet seats that are low."

2 Clients with total hip replacement should not bring their operative leg across midline, which may result in a prosthesis dislocation. Clients should maintain abduction (pillow between legs) and use elevated toilet seats. Crossing the legs is adduction, which is contraindicated for this client. (Lewis et al., 10 ed., p. 1468)

A child with leukemia is being discharged after beginning chemotherapy. What instructions will the nurse include in the teaching plan for the parents of this child? 1. Provide a diet low in protein and high in carbohydrates. 2. Avoid unwashed fruits and vegetables. 3. Notify the doctor if the child's temperature exceeds 102°F (39°C). 4. Increase the use of humidifiers throughout the house.

2 Fresh fruits and vegetables harbor microorganisms, which can cause infections in the immunosuppressed client. Fruits and vegetables should either be peeled or cooked. The doctor should be notified of a temperature greater than 100°F because of immunosuppression. A diet low in protein is not indicated, and humidifiers may harbor fungi in the water containers. (Lewis et al., 10 ed., pp. 639-640)

A young adult comes to the clinic complaining of dizziness, weakness, and palpitations. What will be important for the nurse to evaluate initially when obtaining the health history? 1. Activity and exercise patterns 2. Nutritional patterns 3. Family health status 4. Coping and stress tolerance

2 Iron deficiency anemia is characterized by fatigue, dizziness, weakness, increased pulse, palpitations, and increased sensitivity to cold. The adult female often becomes anemic for a variety of reasons, such as poor nutrition and heavy menses. (Lewis et al., 10 ed., pp. 609-610)

The nurse is monitoring an IV infusion of sodium nitroprusside. Fifteen minutes after the infusion is started, the client's blood pressure goes from 190/120 mmHg to 120/90 mmHg. What is a priority nursing action? 1. Recheck the BP and call the doctor. 2. Decrease the infusion rate and recheck the blood pressure in 5 minutes. 3. Stop the medication and keep the IV open with D5W. 4. Assess the client's tolerance of the current level of BP.

2 Nipride is a powerful, rapid vasodilator. The nurse should decrease the infusion first before the pressure drops further, then assess the client's response to the decreased rate. If the client's urinary output remains adequate and there is no dizziness or neurologic change, then the client is probably tolerating the blood pressure level. (Lewis et al., 10th ed., p. 692).

For a client with COPD, what is the main risk factor for pulmonary infection? 1. Fluid imbalance with pitting edema 2. Pooling of respiratory secretions 3. Decreased fluid intake and loss of body weight 4. Decreased anterior-posterior diameter of the chest

2 The ineffective clearing of secretions with resultant pooling can lead to an increased risk for infection. The client's appetite is usually decreased. The client has an increased anteroposterior diameter of the chest. (Ignatavicius & Workman, 8 ed., p. 566 )

The nurse receives report on assigned clients. One client is reported to be at the nadir for his cancer chemotherapy. How will this affect the nursing care plan? 1. Implement bleeding precautions. 2. Reinforce measures and teaching regarding preventing infections. 3. Anticipate nutritional problems caused by nausea and vomiting. 4. Assess for problems with fluid balance.

2 The nadir refers to the point in the chemotherapy when the leukocytes or neutrophils are at the lowest level. The client's ability to resist infections is at the lowest point, and the client is at the highest risk for developing an infection. Bleeding precautions are implemented with thrombocytopenia or decreased platelets. Nutritional problems are common throughout chemotherapy, and there is no increased risk for development of problems with fluid balance. (Lewis et al., 10 ed., pp. 250-254)

A client experiencing severe depression is admitted to the inpatient psychiatric unit. During the initial assessment, she says, "I feel like killing myself, but I wouldn't do that because of my kids." The nurse's priority action would be to: 1. Explore the reasons that the client might want to take her life. 2. Determine the severity of her suicidal risk. 3. Prevent the client from harming herself. 4. Guide her to consider alternative ways of coping.

2 The priority nursing action is to determine the suicidal lethality. Ask clients whether they plan to hurt themselves, what the method would be, and what factors might interfere with the rescue. The more detailed the plan, the more lethal and accessible the method. The more effort that is exerted to block rescue, the greater the likelihood will be of the suicidal effort being successful. Although the other three options are definitely plausible, they are not the priority. (Halter, 7 ed., pp. 485-486)

A client has an order for one unit of packed cells. What is a correct nursing action? 1. Initiate the IV with 5% dextrose in water (D5W) to maintain a patent access site. 2. Initiate the transfusion within 30 minutes of receiving the blood. 3. Monitor the client's vital signs for the first 5 minutes. 4. Monitor the client's vital signs every 2 hours during the transfusion.

2 The whole blood must be given within the first 30 minutes to prevent hemolysis and growth of bacteria in the blood. Vital signs are to be checked after administration of the first 50 mL and after the first 15 minutes. (Lewis et al., 10 ed., p. 649)

The nurse is caring for a client who is experiencing an acute asthma attack. He is dyspneic and experiencing orthopnea; his pulse rate is 120 beats/min. In what order will the nurse provide care to this client? Number the following options in the order in which they will be performed, with 1 being the first action and 4 being the last action. 1. ________ Administer humidified oxygen. 2. ________ Place in semi-Fowler's position. 3. ________ Provide nebulizer treatment with bronchodilator. 4. ________ Discuss factors that precipitate attack.

2, 1, 3, 4 1___2___ Because oxygen is a priority, begin administration of oxygen. 2___1___ The first action is to place the client in semi-Fowler's position. Oxygen or inhalation therapy cannot be effective with severe orthopnea if the client is not in a sitting or upright position. 3___3___ Then administer the nebulizer treatment, which would include bronchodilators. 4___4___ Physiologic needs must be addressed before teaching or psychosocial needs are considered. (Ignatavicius & Workman, 8 ed., pp. 548-557)

What is a common side effect of radiation therapy that is not associated with the effect of radiation in the treatment field? 1. Reddened skin 2. Bone marrow suppression 3. Fatigue 4. GI disturbances

3 A general body or system effect is fatigue because it occurs as a result of changes in cell cycle patterns and toxic effects from cell destruction. The other problems (reddened skin, bone marrow suppression, GI disturbance) occur when radiation is directed at specific parts of the body. (Lewis et al., 10 ed., pp. 250-256)

While caring for a client who has recently been diagnosed with Parkinson disease, the nurse should understand that: 1. Intellectual capabilities will decrease. 2. Diversional interests may decrease. 3. Mood fluctuations may occur. 4. Communication skills may fluctuate.

3 Because of the emotional stress of Parkinson disease, mood disturbances often occur. Problems with communication, intellectual skills, and diversional interest are usually not as common as mood disturbance in the early stages. (Lewis et al., 9 ed., pp. 1436-1437)

The nurse prepares to irrigate the external auditory canal for a client with impacted cerumen. What would be included in the correct technique for irrigation? 1. Use cool tap water. 2. Pour solution into ear canal. 3. Assess for signs of pain and tenderness in the ear. 4. Use a cotton-tipped applicator to clean near the tympanic membrane.

3 Before performing the irrigation, the nurse should assess the client for pain and tenderness, which could be caused by a perforated eardrum or impaction of a foreign body, and for dizziness caused by disequilibrium. The temperature of the water or saline solution should be comfortable to the wrist of the client or nurse, and it should be sterile. The nurse uses an ear syringe to inject water onto the superior side of the ear canal. The client should be sitting or lying down to facilitate drainage and to maintain safety in case the client gets dizzy from the irrigation. Cotton-tipped applicators should only be used on the outer (pinna or auricle) ear. (Lewis et al., 10 ed., p. 384)

A client with a diagnosis of schizophrenia repeatedly states, "There are flies eating my brain and making me feel weird." The client is most likely experiencing which of the following? 1. Ideas of reference 2. Grandiose delusions 3. Somatic delusions 4. Persecutory delusions

3 Bizarre ideas that focus on the body being incapacitated are known as somatic delusions and are sometimes observed in schizophrenia. Grandiose delusions are beliefs of being important. Ideas of reference or delusions of reference occur when a person believes or perceives that irrelevant, unrelated, or innocuous things in the world are referring to them directly or have special personal significance. Persecutory delusions are when a person believes (wrongly) that they are being picked on or threatened by someone/ something else. (Halter, 7 ed., p. 206)

A client asks the nurse why he has to take several chemotherapy agents at the same time. The nurse's response would be based on which principle? 1. The more medications that can be given together, the shorter the treatment period. 2. The cost is decreased because the medications are administered at the same time. 3. Multiple medications given together will attack the cancer cells at different levels. 4. One medication will interact with another to reduce incidence of side effects.

3 Combination drug therapy is important because different drugs inhibit cancer cell growth at various phases of cellular replication. This makes each of the medications more effective. Medications are given together because this is a more effective method of treatment, not because of cost or to reduce the side effects. (Lewis et al., 10 ed., p. 248)

A client who underwent cholecystectomy 3 days ago has a T-tube that has stopped draining. What is the best nursing action? 1. Flush the tube with 5 mL of normal saline solution. 2. Reposition the client. 3. Continue to monitor. 4. Assess for tube placement.

3 Continue monitoring, as T-tube drainage decreases after day 2 and may stop between days 3 and 5. T-tubes are not irrigated. Placement is not checked. Repositioning will not change the drainage. (Lewis et al., 10 ed., p. 1009)

A client has a history of atherosclerotic heart disease with a sustained increase in his blood pressure. What is important to discuss with this client before he uses an over-the-counter decongestant? 1. Urinary frequency and diuresis 2. Bradycardia and diarrhea 3. Vasoconstriction and increased arterial pressure 4. Headache and dysrhythmias

3 Decongestants should be avoided by clients with hypertension because these medications often contain pseudoephedrine and phenylephrine, which cause central nervous system stimulation with vasoconstriction and increased blood pressure. They also precipitate anxiety and insomnia. Decongestants do not cause urinary frequency, diuresis, or dysrhythmias. (Lehne, 8 ed., p. 981)

The nurse questions the use of which drug for the client with cerebral hemorrhage? 1. Gemfibrozil 2. Mannitol 3. Enoxaparin 4. Nitroprusside

3 Enoxaparin is a low-molecular-weight heparin. Thinning the blood of a client with cerebral hemorrhage could significantly worsen the bleed. Gemfibrozil is used to decrease cholesterol. Mannitol is an osmotic diuretic. Nitroprusside is used for a hypertensive crisis. (Lewis et al., 9 ed., pp. 1370-1373)

A neonate is being discharged home with a fiber-optic blanket for treatment of physiologic jaundice. What is important for the nurse to include in the discharge instructions? 1. Cover the infant's eyes during the treatment. 2. Reduce the daily number of formula feedings. 3. Encourage frequent feeding to increase intake. 4. Expect a constipated stool until jaundice clears.

3 Feedings and fluids should be encouraged to promote excretion of the bilirubin. It is not necessary to cover the neonate's eyes with use of the fiber-optic blanket, but there should be a covering pad between the infant's skin and the fiber-optic blanket. The stool would be loose, rather than constipated, while the jaundice is resolving. (Hockenberry & Wilson, 10 ed., p. 322)

The nurse is providing teaching to a family whose child has been recently diagnosed with hemophilia. Which of the following would the nurse include in this discussion? 1. Hemophilia is a genetic disease that is more common in females. 2. Hemophilia is correctable through transfusions and bone marrow transplantation. 3. Hemophilia is most often a sex-linked congenital disorder. 4. Hemophilia is preventable through genetic counseling.

3 Hemophilia is a sex-linked genetic disorder carried by females but manifested in males. The female is a carrier and the male exhibits the condition in a sex-linked disorder. Genetic counseling is important for a couple to identify the risks involved, but if a woman who is a carrier decides to have children, the risk of her children having the condition cannot be prevented. (Lewis et al., 10 ed., p. 626)

The nurse is making a home visit to a client with hepatitis A virus (HAV). Before assessing the client, the nurse will gather the equipment and perform what action next? 1. Wipe the bedside table with alcohol preps. 2. Place the supplies on a clean, convenient work area. 3. Maintain standard precautions before and after client contact. 4. Put on a gown, mask, and gloves.

3 Hepatitis A is transmitted via fecal contamination and oral ingestion. It is important to maintain standard precautions before and after client contact. The use of standard precautions should prevent transmission of HAV to the health care worker. Paper towels are used to create a clean area surface. Alcohol preps are not effective. The mask is not appropriate because hepatitis is not spread by respiratory secretions. (Lewis et al., 10 ed., p. 982)

A client with cirrhosis is receiving neomycin sulfate. The nurse understands that the purpose of this medication is to: 1. Decrease gastric acidity. 2. Acidify feces and trap ammonia in the bowel. 3. Decrease bacterial flora. 4. Reduce portal hypertension.

3 Neomycin sulfate decreases bacterial flora in the bowel, thus decreasing ammonia. Proton pump inhibitors and H2 blockers decrease gastric acidity in the treatment of cirrhosis. Lactulose acidifies feces in the bowel and traps ammonia, causing its elimination in the feces. Propranolol reduces portal venous pressure. (Lewis et al., 10 ed., p. 992)

The nurse is obtaining a health history from a client who reports having pain in the left arm. Which question by the nurse will elicit the most useful response from the client? 1. "Does the pain feel like pins and needles in your arm?" 2. "Does the pain radiate from your neck to your arms?" 3. "Can you describe the pain you are experiencing in your arm?" 4. "Is the numbness in your arm intermittent or constant?"

3 Open-ended questions are most helpful in obtaining accurate health history information because they elicit more detailed descriptions of the symptoms. Although the other options are applicable to the presenting symptom, they result in "yes" or "no" responses, and this does not encourage the client to provide detailed information about the problem. (Potter & Perry, 8 ed., p. 322)

The nurse is evaluating a teenager for hearing loss. In reviewing the client's history, the nurse knows that which finding is not associated with a hearing loss? 1. Listening to loud music on an iPod 2. Repeated chronic ear infections 3. Taking penicillin and cephalosporin medication 4. History of increased ear cerumen

3 Penicillin and cephalosporin medications are not ototoxic. Aminoglycosides are ototoxic. The other three options—listening to loud music on an iPod, repeated chronic ear infections, and a history of increased ear cerumen—are risk factors for hearing loss. (Lewis et al., 9 ed., p. 391; Lehne, 9 ed., p. 1056.)

A client with chronic kidney disease has an internal venous access site for hemodialysis on their left forearm. What action will the nurse take to protect this access site? 1. Irrigate with heparin and normal saline solution every 8 hours. 2. Apply warm moist packs to the area after hemodialysis. 3. Do not use the left arm to take blood pressure readings. 4. Keep the arm elevated above the level of the heart.

3 Protect the arm with the functioning shunt. No blood pressure readings should be taken from that arm, and there should be no needlesticks. The access is not irrigated with heparin. (Lewis et al., 10 ed., p. 1087)

The nurse is assessing a client with a tentative diagnosis of a brain tumor. What primary client complaint would the nurse anticipate? 1. Decreased appetite 2. Frequent insomnia 3. Recurrent headaches 4. Peripheral edema

3 Recurrent headaches that increase in frequency and severity are often the first complaint of a client with a brain tumor; the headaches usually correlate with the area of the brain involved. The headaches tend to be worse at night and may awaken the client. Headaches are described as dull and constant but occasionally throbbing. Nausea and vomiting can occur due to ICP. Mood and personality changes occur, especially with brain metastases. Muscle weakness, sensory losses, aphasia, and perceptual/spatial dysfunction are symptoms. Insomnia, decreased appetite, and peripheral edema are not relevant to a brain tumor. (Lewis et al., 9 ed., p. 1376)

The nurse understands that scaling around the toes, blistering, and pruritus is characteristic of what condition? 1. Eczema 2. Psoriasis 3. Tinea pedis 4. Pediculosis corporis

3 Scaling, itching, and redness are common signs of tinea pedis or athlete's foot. Eczema or atopic dermatitis in adults is characterized by reddened lesions in antecubital and popliteal space with pruritus or in children on cheeks, arms, and legs. Psoriasis is a benign condition of the skin where there are silvery scaling plaques on the skin, commonly the elbows, knees, palms, and soles of the feet. Pediculosis corporis is body lice and is a parasitic infection. (Lewis et al., 10 ed., p. 418)

What is the most important nursing intervention for the safety of a client with altered clotting mechanisms caused by hepatic cirrhosis? 1. Promote independence in the client's activities of daily living. 2. Administer antibiotics to decrease ammonia. 3. Implement bleeding precautions. 4. Increase vitamin supplements and nutritional intake.

3 The altered clotting mechanisms in a client with hepatic cirrhosis mean that the client has fewer clotting factors available to assist in the clotting process. Bleeding precautions should be in place. Although the client may be able to be independent in ADLs, for reasons of safety, supervision should be provided. Antibiotics and increased nutritional intake will not speed clotting times. (Lewis et al., 10 ed., pp. 994, 1008)

A newborn is suspected of having esophageal atresia with a tracheal esophageal fistula. What nursing assessment information would assist in validating the presence of a fistula? 1. Clammy skin and a croupy cough 2. Crying and chest retractions 3. Choking and coughing 4. Chin tug and circumoral pallor

3 The classic three Cs—choking, coughing, and cyanosis—plus frothy saliva and constant drooling are the characteristic signs of esophageal atresia with a tracheoesophageal fistula. The other options are an incomplete description of the signs and symptoms. (Hockenberry & Wilson, 10 ed., p. 1108)

A client in sickle cell crisis is admitted to the emergency department. What are the priorities of care in order of importance? 1. Nutrition, hydration, electrolyte balance 2. Hydration, pain management, electrolyte balance 3. Hydration, oxygenation, pain management 4. Hydration, oxygenation, electrolyte balance

3 The priorities for care of a client in sickle cell crisis are focused on providing fluid, oxygen, and pain control during the crisis to reduce sickling and prevent complications. Electrolyte management is not a priority, nor is nutrition. (Lewis et al., 10 ed., pp. 617-619)

The nurse is discussing the prevention of urinary tract infections with a female client. What would be important to include in the discussion? 1. Decrease fluid intake to decrease burning on urination. 2. Take warm sitz baths with a mild bubble bath. 3. Avoid spermicides with nonoxynol-9. 4. Drink only acidic fluids such as orange juice.

3 The use of nonoxynol-9 spermicides can be irritating to the urinary tract and lead to infections. UTIs in women can be prevented by urination before and after sexual intercourse. Fluid intake should be increased, and no soap should be added to the sitz bath. (Lewis et al., 10 ed., p. 1036)

The nurse is caring for a client who is being treated with chemotherapy for his lung cancer. The client has had two treatments in the last 2 days, and the nurse notes hyperkalemia and hyperuricemia on the latest serum laboratory values. The nurse understands that these are symptoms of: 1. Third-space syndrome 2. Syndrome of inappropriate antidiuretic hormone 3. Tumor lysis syndrome 4. Parathyroid deficiency

3 These two findings, hyperuricemia and hyperkalemia, are hallmark symptoms of tumor lysis syndrome, which often occurs at the onset of chemotherapy when a large number of tumor cells are destroyed. This process yields fatal biochemical changes of hyperkalemia, hyperuricemia, hypocalcemia, and hyperphosphatemia if not averted with adequate fluids. (Lewis et al., 10 ed., p. 263)

The nurse is discharging a client with bilateral cataracts following cataract surgery on one eye. What statement by the client would indicate to the nurse the need for additional teaching? 1. "I'll call if I have a significant amount of pain." 2. "I'll remember to wash my hands before changing the eye dressing." 3. "I'll be okay by myself at home today." 4. "I will have someone help me with my eye medications."

3 This client may experience visual impairment and difficulty with selfcare the day of surgery because the operative eye will be patched and the other eye still has a cataract. This client may need some special assistance at home until the vision improves, especially the first 24 hours while the operative eye is patched. Discomfort after cataract surgery is minimal, thus there should be no significant amount of pain. Proper hygiene is important to prevent wound/eye contamination during dressing changes. Eye drops may be difficult for the client to administer, so assistance is a good idea. (Lewis et al., 10 ed., p. 373)

A client is diagnosed with epididymitis related to bladder outlet obstruction. The nurse teaches which intervention to assist the client in recovery? 1. Walk briskly at least 30 minutes every day. 2. Limit oral intake to minimize nausea and vomiting. 3. Apply scrotal support to relieve edema and discomfort. 4. Use warm baths and compresses during acute inflammation.

3 To facilitate lymphatic drainage from the inflamed epididymis, the client should be taught to use a towel roll to act as a "scrotal bridge." Cool compresses should be used in the acute phase. There is no reason to limit oral intake. Walking is extremely painful, and bed rest is usually indicated to prevent spread of infection. (Lewis et al., 10 ed., p. 1284)

A sexually active female 17-year-old is diagnosed with trichomoniasis through vaginal discharge analysis. The nurse explains which pharmacologic intervention to minimize symptoms and the risk for reoccurrence? 1. Return to clinic each week for intramuscular injection of penicillin. 2. Perform a daily vaginal douche with a weak iodine solution. 3. Oral administration of metronidazole three times a day to client and her partner. 4. Application of trichloroacetic acid to lesions daily for 6 to 8 weeks.

3 Trichomoniasis is a protozoal infection transmitted through sexual intercourse. All sexual partners need to be treated with the oral administration of metronidazole, a systemic antiprotozoal. Males may be asymptomatic. Intramuscular injection of penicillin is prescribed for syphilis. Douching should be avoided because it destroys the normal vaginal flora and increases the risk for developing the problem. Trichloroacetic acid (TCA) or podophyllin is topically applied to genital warts. (Lewis et al., 10 ed., p. 1231)

A client who has had a stroke 1 week ago remains aphasic. The client is beginning to show functional improvement and demonstrates an ability to follow verbal directions. What will rehabilitation now include? 1. A right-leg brace 2. Ambulation training 3. Speech training 4. Vocational retraining

3 When a stroke occurs in the dominant hemisphere, the client experiences communication difficulties or aphasia. Speech retraining cannot begin until the client understands and can follow directions. The question is focusing on the client's ability to speak and his current problem with aphasia. Although wearing a leg brace and ambulation training will begin when the client has stabilized, the question is focusing on the client's aphasia. Vocational retraining will be part of the client's rehabilitation at a later date. (Lewis et al., 9 ed., p. 1394)

A nurse case worker suspects older adult neglect. Which assessment findings during a home visit would confirm this? (Select all that apply.) 1. Confusion and disorientation 2. Recent hip fracture 3. Poor nutrition and hygiene 4. Dirty dishes in the sink 5. Outdated prescription bottles 6. Missing hearing aids

3, 5, 6 Lack of assistive devices, medication mismanagement, and access to basic physiologic needs such as hygienic care, food, and water are characteristics of neglect in the older adult. A hip fracture is typically caused by osteoporosis in older adults, not neglect. Confusion and disorientation are signs of dementia. Dirty dishes in the sink is not a sign of neglect. (Halter, 7 ed., p. 540)

A client reports that her last menstrual period was November 10. She asks the nurse, "When will my baby be due?" What is the best answer? 1. July 3 2. August 30 3. Around the middle of September 4. Around the third week of August

4 According to Näegele's rule, count back 3 months from the date of the last menstrual period and add 7 days to determine the estimated date of conception. About 35% of all women deliver within 5 days of (either before or after) this date. (Lowdermilk et al., 11 ed., p. 302)

A 6-year-old client is admitted to the postoperative recovery area after a tonsillectomy. In what position will the nurse place the client? 1. Semi-Fowler's position, with the head turned to the side 2. Prone position, with the head of the bed slightly elevated 3. On the back, with the head turned to the right side 4. On the abdomen, with the head turned to the side

4 Before the child is fully awake, he or she should be placed on the abdomen with the head turned to one side to facilitate the drainage of secretions and to prevent aspiration. When alert, the child may sit up or assume a position of comfort. The other options are not appropriate because they do not allow for drainage of secretions from the mouth and throat after a tonsillectomy while the child is in early recovery. (Hockenberry & Wilson, 10 ed., p. 1174)

A client asks the nurse about an alternative remedy for hot flashes. Which dietary supplement is the client asking the nurse about? 1. Ginseng. 2. Valerian. 3. Feverfew. 4. Black cohosh.

4 Black cohosh is an herbal supplement used for treating symptoms of menopause, including hot flashes, vaginal dryness, palpitations, depression, irritability, and sleep disturbance. The nurse should determine whether the client's symptoms are caused by tamoxifen and other selective estrogen receptor modulators, if this is the situation, then black cohosh should not be used. Ginseng improves mood, boosts endurance, and may lower blood glucose. Feverfew is taken for migraine prophylaxis. Valerian has sedative properties that promote sleep and reduce anxiety. Burchum & Rosenthal, 9 ed., pp. 1321-1328)

The nurse is admitting a 5-month-old infant. The health care provider has ordered an IV solution of normal saline. There is also an order for potassium chloride (KCl) to be added to the solution. The infant's temperature is 101°F (38.8°C) rectally, and the pulse is 120 beats/min; the infant is irritable and has not voided. What is the priority nursing action? 1. Wait for 1 hour from admission time and then begin the infusion of normal saline with the KCl. 2. Feed the infant before adding the KCl to the infusing solution. 3. Consider the order a stat order and begin the infusion immediately. 4. Start the normal saline infusion and hold the KCl until adequate urinary output has been documented.

4 In infants (and adults), validation of renal function must be established before the delivery of IV KCl. This is necessary to prevent hyperkalemia and possible death. The key point in the question is that the infant has not voided, so the prudent nurse should hold the KCl until adequate urine output is documented. (Lewis et al., 10 ed., p. 277)

A client is being seen in the emergency department after an accident. He has no obvious physical injuries, and his blood pressure is 158/90 mmHg. He is crying loudly, wringing his hands, and pacing the floor. His respiratory rate is 32 breaths/ min, and he says he feels lightheaded. What is the best nursing response? 1. Have him lie down and begin O2 per nasal cannula at 4 L/min. 2. Put him on a stretcher and begin a head-to-toe assessment. 3. Perform a quick neurologic examination to determine his level of orientation. 4. Have him sit down and help him breathe into a paper sack.

4 The client is experiencing acute respiratory alkalosis from hyperventilation caused by anxiety. Rebreathing in a paper sack will help reestablish normal Paco2 levels. The other options do not address the origin of the problem and may further increase his anxiety. (Potter, 9 ed., p. 944)

A client with Parkinson disease is experiencing anorexia and vomiting. The client is taking levodopa. What will be the initial nursing activity? 1. Assess client's food preferences. 2. Monitor client's blood pressure. 3. Hold client's medication and notify the physician. 4. Administer client's medication with food.

4 The first side effect to be noticed may be gastrointestinal problems. Taking medication with meals may alleviate these symptoms, but high-protein meals should be avoided. The client may continue to take the medication, and attempts should be made to minimize the side effects. The client's food preferences and his blood pressure are not relevant to the situation. (Lewis et al., 9 ed., p. 1435)

A client is being admitted for problems with Ménière disease. What is most important for the nurse to assess to promote the client's safety? 1. Diet history 2. Screening hearing tests 3. Effect on client's activities of daily living (ADLs) 4. Frequency and severity of attacks

4 The nurse must assess the frequency and severity of attacks to plan best for the client's safety. Although hearing tests and diet may be of some significance, they will not protect the client immediately. After the client's immediate safety needs are met, the nurse will want to determine the effect that Ménière's disease has on the client's ADLs. (Lewis et al., 10 ed., p. 386)

The nurse is assessing the pulse of a client in atrial fibrillation. Based on the graphic below, where should the stethoscope be placed to correctly auscultate this client's pulse? (image with negative T waves that interrupt QRS)

4 The nurse needs to auscultate the apical pulse when a client is in atrial fibrillation. The point of maximal impulse is the correct location for auscultation of the apical pulse. It is located at the fifth intercostal space, at the midclavicular line. This is the apex of the heart. It is noted with a black dot on the correct area in the illustration. (Lewis et al., 10 ed., p. 667)

An infant born at 28 weeks' gestation weighs 4 lb 3 oz (1950 g). What does the initial nursing care of this infant include? 1. Place the infant in protective isolation because of the underdeveloped immune system. 2. Feed him a low-phenylalanine formula to increase digestion and utilization of calories. 3. Provide gavage feedings every 2 hours because of an inadequate sucking and swallow reflex. 4. Place the infant under a radiant heater to maintain regulation of body temperature.

4 The premature infant is at greater risk for chilling. A premature infant has less brown fat than does a term neonate, as well as a lower reserve of glycogen and lower reserve for breathing. A lower body temperature leads to greater oxygen consumption, a decrease in surfactant production, and a tendency to develop acidosis. The infant does not need to be in protective isolation due to an underdeveloped immune system. There is no indication in the stem of the question that the infant has PKU, for which a lowphenylalanine diet would be indicated. Gavage feedings provide nourishment to the neonate who is compromised by respiratory distress or who is too immature to have coordinated suck-swallow reflexes, or who is easily fatigued by sucking. (Hockenberry & Wilson, 10 ed., p. 341)

The nurse is preparing medications for a client. The medication order is for cefaclor 0.1 gm PO. The dose available in the unit is 125 mg/5 mL. How many milliliters will the nurse need to give? Answer: __________ mL

4 mL 1 gm 5 1000 mg; therefore, 0.1 gm 5 100 mg 125 mg : 5 mL :: 100 mg : x mL (Potter & Perry, 9 ed., pp. 618-620)

The nurse practitioner orders an enteral formula at a rate of 50 mL/hr. A can holds 250 mL. How many cans would the nurse need for the next 24 hours? Answer: ______ cans

5 cans 50 3 mL/hr 3 24 hours/day 5 1200 total mL for 24 hours 1200 mL/24 hr ÷ 250 mL/can 5 4.8 cans or 5 cans. (Potter & Perry, 9 ed., p. 617)

The nurse is teaching a client with hypertension about his antihypertensive medications, hydrochlorothiazide (HCTZ) and enalapril. What is important to include in this teaching? 1. "Stand up slowly to decrease problem with dizziness." 2. "Increase fluid intake because of increased loss of body fluids." 3. "When you begin to feel better, the doctor will decrease your medications." 4. "Stay out of the sunshine, and make sure you have adequate sodium intake."

1 A common side effect of a combination of antihypertensive and diuretic medications is postural hypotension. It is important to teach the client how to deal with it. The client should not increase intake of fluids, because the diuretics are being given to decrease excess fluid. The client should decrease his intake of sodium. When the client is feeling better, the medication is working and will probably not be decreased. (Lewis et al., 10th ed., p. 697)

The client returns to his room after a thoracotomy. What will the nursing assessment reveal if hypovolemia from excessive blood loss is present? 1. CVP of 3 cm H2O and urine output of 20 mL/hr 2. Jugular vein distention with the head elevated 45 degrees 3. Chest tube drainage of 50 mL/hr in the first 2 hours 4. Increased BP and increased pulse pressure

1 A low-range CVP reading and the decrease in urine output would be associated with hypovolemia caused by hemorrhage. Normal CVP is 2 to 6 cm H2O. The decrease in urine output is reflective of poor renal perfusion. Jugular vein distention is indicative of increased CVP, which does not occur with hypovolemia. Chest tube drainage is within the normal expectations. The blood pressure decreases with hemorrhage. (Lewis et al., 10th ed., p. 1564)

A client is experiencing a sickle cell crisis during labor and delivery. What is the best nursing action? 1. Maintain IV fluid infusion and assess adequacy of hydration. 2. Administer a high concentration of oxygen. 3. Insert a Foley catheter and monitor hourly urine output. 4. Provide continuous sedation for pain relief.

1 Adequate hydration is critical during stress periods for the client with sickle cell disease, and this is particularly true of a client in labor. Oxygen may or may not be ordered in low concentrations. A urinary retention catheter is not necessary at this time and would be a potential cause of infection. Although pain relief is important for both the sickling issues and the labor pains, continuous sedation would not be indicated because this would be detrimental to the fetus. (Lewis, 10 ed., pp. 617-619; Lowdermilk et al., 10 ed., pp. 721-722)

The nurse is working in the pediatric unit and receives a phone order from the doctor for a 10-year-old client who weighs 40 kg. The order is for ceftazidime 1.5 gm every 8 hours IV. The therapeutic dosage range is 90 to 150 mg/kg/24 hr. What would be the best nursing action? 1. Administer the medication because it is within the therapeutic dosage range. 2. Call the doctor to clarify the order because it is outside the therapeutic dosage range. 3. Call the hospital pharmacist and ask him or her to calculate the dosage. 4. Notify the nursing supervisor and request assistance.

1 Administer the medication ceftazidime as ordered. The therapeutic dosage is 90 to 150 mg/kg/day. Ordered: 1.5 gm every 8 hours 5 1.5 gm 3 3 doses 5 4.5 gm per day Minimum therapeutic dose: 90 mg 3 40 kg 5 3600 mg or 3.6 gm per day Highest safe dose: 150 mg 3 40 kg 5 6000 mg or 6 gm per day (Potter & Perry, 9 ed., p. 620)

Six hours after gastric resection, the client's NG aspirate is continuing to drain bright red fluid. What is the best nursing action? 1. Continue to monitor the amount of drainage and correlate it with any change in vital signs. 2. Reposition the NG tube and irrigate the tube with normal saline solution. 3. Call the physician immediately and notify of the continued bright red aspirate. 4. Irrigate the NG tube with iced saline solution and attach the tube to gravity drainage.

1 After gastric surgery, the aspirate is usually bright red at first, gradually darkening within the first 24 hours after surgery. Normally the color changes to yellow-green within 36 to 48 hours. This is a normal occurrence on the first postoperative day and should be correlated with the vital signs. The tube is in the correct position because it is draining gastric secretions. There is no indication to notify anyone or to irrigate the NG tube. (Lewis et al., 10 ed., p. 918)

The nurse is caring for a client who is categorized as HIV positive, acute infection. What would the nurse anticipate finding on the nursing assessment? 1. Fever, swollen lymph glands, nausea 2. Confusion, wasting syndrome, localized infections 3. Dyspnea, dementia, persistent fever 4. Night sweats, low-grade fever, generalized lymphadenopathy

1 An acute infection occurs when the primary condition is identified or the client has recently been infected. The client may be asymptomatic at this time or they may have flulike symptoms and early nonspecific changes characterized by fatigue, sore throat, fever, diarrhea, muscle and joint pain, diffuse rash, and swollen lymph glands. The other items are symptoms noted with chronic HIV infection. Chronic HIV infection can be either asymptomatic (fatigue, headache, low-grade fever, night sweats, persistent generalized lymphadenopathy), symptomatic (symptoms become worse, leading to persistent fever, frequent drenching night sweats, chronic diarrhea, recurrent headaches, and fatigue severe enough to interrupt normal routines with localized infections, lymphadenopathy, and nervous system problems), or AIDS (development of opportunistic infections, wasting syndrome, dementia). The symptoms noted in the remaining options are seen with chronic disease. (Lewis et al., 10 ed., pp. 220-221)

The nurse is assessing a client who is on a ventilator and has an endotracheal tube in place. What information confirms that the tube has migrated too far into the trachea? 1. Decreased breath sounds are heard over the left side of the chest. 2. Increased rhonchi are present at the lung bases bilaterally. 3. Client is able to speak and coughs excessively. 4. Ventilator pressure alarm continues to sound.

1 An endotracheal tube that is inserted too far—beyond the carina—is most likely to enter the right main stem bronchus. The volume of air from the ventilator is only delivered to the right lung; breath sounds are decreased or absent over the left lung. The pressure alarm indicates that the current pressure is not adequate to deliver the tidal volume prescribed. This may occur but does not confirm the migration of the tube. (Ignatavicius & Workman, 8 ed., p. 615)

What is an allergic reaction that can quickly deteriorate into shock and death? 1. Anaphylaxis 2. Graft-versus-host disease 3. Type III, immune complex formation 4. Delayed sensitivity

1 Anaphylaxis is a massive antigen-antibody response, causing a physiologic system shutdown and possible death. Graft-versus-host disease (GVHD) occurs when antibodies in the transplanted organ attack the host's antigens. An immune complex formation occurs when antigen-antibody complex is deposited in body tissue. A delayed sensitivity is a reaction that occurs after cells are sensitized to an antigen, as seen in the tuberculosis skin testing. (Lewis et al., 10 ed., pp. 197-199)

The clinic nurse observes that a 3-day-old baby girl is jaundiced. A bilirubin level is 11.4 mg/dL (194.99 umol/L). What causes this bilirubin level? 1. Physiologic jaundice 2. Hemolytic disease of the newborn 3. Erythroblastosis fetalis 4. Sepsis

1 Approximately 40% to 60% of all term babies develop jaundice between the second and fourth days of life. In the absence of disease or a specific cause, it is referred to as physiologic jaundice. It is most often due to the breakdown of excessive red blood cells and increased destruction of immature red cells. Hemolytic disease of the newborn caused by maternal/newborn blood group incompatibility is the most common cause of pathologic jaundice. Erythroblastosis fetalis and kernicterus are associated with pathologic jaundice. Sepsis does not normally cause an elevated bilirubin level. (Hockenberry & Wilson, 10 ed., p. 315)

A client is confined to bed with a fracture of the left femur. He begins receiving subcutaneous low-molecular-weight heparin (LMWH) injections. What is the purpose of this medication? 1. To prevent thrombophlebitis and pulmonary emboli associated with immobility 2. To promote vascular perfusion by preventing formation of micro emboli in the left leg 3. To prevent venous stasis, which promotes vascular complications associated with immobility 4. To decrease the incidence of fat emboli associated with long bone fractures

1 Because of the high risk of venous thromboembolism (VTE) after a femur or hip fracture, prophylactic anticoagulant drugs such as warfarin and low-molecular-weight heparin such as enoxaparin may be ordered to prevent thromboembolic complications in the immobilized client. It is not effective in preventing fat emboli or venous stasis or promoting vascular perfusion. (Lewis et al., 10 ed., p. 1468)

The nurse is about to conduct an admission assessment on an 85-year-old female client who is admitted after falling down in her home. When determining the amount of time to set aside for the interview, the nurse will consider which of the following? 1. Allow ample time to gather psychosocial data from the client. 2. Skip the psychosocial assessment; it is not important for the client with a physiologic problem. 3. Interview the client's daughter and son about the client's psychosocial background. 4. Ask the client whether she has any pressing or major issues she wants to talk about.

1 Because older adult clients may be starved for someone to listen to them, the nurse must allow ample time to gather psychosocial data. It would not be good nursing judgment to skip the psychosocial assessment. The nurse should interview the client and, if additional information is required, then the client's family can be interviewed. Although asking the client about pressing or major issues is a good answer, it does not address the importance of allowing sufficient time to actually discuss the pressing or major issues that the client may want to describe. (Potter & Perry, 8 ed., pp. 318-319)

The nurse is caring for a postoperative client who had a thyroidectomy. The client develops difficulty breathing from laryngospasms, muscular spasms, and twitching, Which medication should the nurse have available for emergency treatment in the client who has had a thyroidectomy? 1. Calcium chloride. 2. Potassium chloride. 3. Magnesium sulfate. 4. Propylthiouracil.

1 Calcium chloride or calcium gluconate should be available to treat tetany caused by accidental removal of the parathyroid glands during surgery. The parathyroid glands regulate calcium metabolism. Potassium chloride replaces the electrolyte potassium. Magnesium sulfate is used in the treatment of preeclampsia (pregnancyinduced hypertension). Propylthiouracil is an antithyroid medication used to block production of thyroid hormone. (Lewis et al., 10 ed., p. 1173)

What is the priority assessment information to obtain from a client who is being admitted with a tentative diagnosis of fractured hip? 1. Circulation and sensation distal to the fracture 2. Amount of swelling around the fracture site 3. Degree of bone healing that has occurred 4. Amount of pain that the fracture and healing are causing

1 Circulation and neurosensory status distal to the fracture are always priorities for clients with fractures. The amount of swelling is important, but the primary concern regarding swelling is circulatory and neurosensory deficits. The amount of bone healing cannot be assessed. There is concern regarding pain, but circulatory and neurologic checks are the priority actions. (Lewis et al., 10 ed., p. 1468)

What finding on the nursing assessment would be associated with a diagnosis of pneumonia in the older adult? 1. Acute confusion 2. Hypertension 3. Hematemesis in the morning 4. Dry hacking cough at night

1 Confusion in the older adult is related to hypoxemia, which occurs with pneumonia. Vasodilation and dehydration cause hypotension and orthostatic changes. Crackles are typically heard when fluid is in the alveolar area. The cough is generally productive. The breathing is rapid and shallow without the use of accessory muscles. Hemoptysis may occur but not hematemesis (blood from the gastrointestinal tract). (Ignatavicius & Workman, 8 ed., pp. 591-592)

A client with Cushing's syndrome is admitted to the medical- surgical unit. During the admission assessment, the nurse notes that the client has a flat affect but is irritable when questioned, has a poor memory, reports a loss of appetite, wants to sleep all the time, and doesn't care if she gets well. What collaborative action should the nurse take in response to this information? 1. Discuss with the health care provider a concern for depression. 2. Request a neurology consult for a CT scan. 3. Discuss with the dietitian a need for a nutritional consult. 4. Request a social service consult for home evaluation.

1 Cushing's syndrome develops because of an excess of cortisol, in this case from prolonged exogenous steroid administration. Depression and a marked change in personality are common. It is important that the client be taught how to deal with the emotional changes of the disease. (Lewis et al., 10 ed., 1174)

The nurse is caring for a client with excess fluid volume. Which action will best evaluate a change in the client's condition? 1. Obtaining the client's daily weight before breakfast each day 2. Measuring fluid intake and output and comparing with values from the previous day 3. Assessing the blood pressure and comparing it with previous readings 4. Auscultating the lungs for the presence of adventitious breath sounds

1 Daily weights are an accurate indicator of fluid retained or lost. One liter of fluid is equal to 2.2 pounds (1 kg). Measuring the intake and output does not take into consideration insensible fluid loss and is not as accurate an evaluation as is the daily weight. Changes in vital signs are less reliable because they do not reflect subtle changes in retention of fluid. Although adventitious sounds may be present if the client is overhydrated, the daily weights would have increased before the pulmonary changes occurred. (Lewis et al., 10 ed., p. 277.)

When caring for a client admitted for medically monitored detoxification from alcohol, the nurse would assess for which of the following signs and symptoms of withdrawal? 1. Anorexia, irritability, nausea, and tremulousness 2. Bradycardia, hypotension, diaphoresis, and fever 3. Vivid hallucinations, coarse irregular tremor 4. Severe craving, euphoria, profuse sweating, and paranoid ideation

1 Detoxification or controlled withdrawal from alcohol via a medical protocol includes regular assessment for withdrawal signs and symptoms and administration of prescribed medications. Signs and symptoms of alcohol withdrawal include anorexia, irritability, nausea, tremulousness, insomnia, nightmares, hyperalertness, tachycardia, increased blood pressure, diaphoresis, and anxiety. (Halter, 7 ed., p. 421)

What is significant about the development of proteinuria in a client with type 1 diabetes mellitus? 1. Chronic kidney disease may eventually develop. 2. It indicates that the client's diabetes is uncontrolled. 3. Serum creatinine will diminish as albuminuria increases. 4. Insulin maintenance dose should be lowered.

1 Diabetic nephropathy is the primary cause of end-stage kidney failure. A microscopic amount of albumin in the urine is one of the earliest indications of kidney abnormality and is asymptomatic, which is the purpose of annual albumin-to-creatinine ratios being collected on random urine samples for albumin. Control of hypertension is paramount to delaying the progression of nephropathy. Serum creatinine elevates along with the increased albumin in the urine. Measure of diabetic control is the HbA1c laboratory test. Insulin therapy should not be altered due to proteinuria. (Lewis et al., 10 ed., p. 1079)

The nurse is teaching self-care to an older adult client. What would the nurse encourage the client do for his dry, itchy skin? 1. Apply a moisturizer on all dry areas daily. 2. Shower twice a day with a mild soap. 3. Use a pumice stone and exfoliating sponge on areas to remove dry scaly patches. 4. Wear protective pads on areas that show the most dryness.

1 Dry skin should be moisturized daily and as needed, especially after the client takes a bath. The number of baths and showers should be limited. Exfoliation will remove the dry epidermal layer, but underlying areas also need moisturizing. Protective pads do nothing to provide moisture to dry areas. (Lewis et al., 10 ed., p. 409)

The nurse is taking the history of a client with heart failure caused by chronic hypertension. Which statement by the client is most concerning? 1. "I get short of breath after walking about half a block." 2. "My weight has dropped 15 pounds over the past 3 months." 3. "My legs get swollen in the evenings." 4. "Sometimes I get dizziness when I get up too quickly."

1 Dyspnea on exertion is a classic sign of left ventricular problems, regardless of the precipitating cause. Lower extremity edema is also characteristic, but it is not as significant as dyspnea on exertion. Dizziness and fainting on standing are indicative of postural (orthostatic) hypotension. (Lewis et al., 10 ed., p. 664)

The nurse is caring for a client with chronic hepatitis B virus (HBV). What will the teaching plan for this client include? 1. Use a condom for sexual intercourse. 2. Report any clay-colored stools. 3. Eat a high-protein diet. 4. Perform daily urine bilirubin checks.

1 HBV is spread by sexual contact, so it is important to teach patients to use a condom for sexual intercourse, and the partner should be vaccinated. There will be no bilirubin in the urine or stools; claycolored stools are expected, so they would not be reported. Client would not be told specifically to eat a high-protein diet. (Lewis et al., 10 ed., p. 982)

A client is being referred to the hospice nurse for care. The nurse explains to the client and the family that the primary goal of hospice differs from the goal of traditional care in what way? Hospice care: 1. Provides support to the family and to the client with a terminal illness 2. Is delivered only at home, so that no extraordinary means are initiated to prolong life 3. Provides a Medicare-supported pain regimen so pain medications are affordable 4. More readily recognizes advance directives related to the "right to die"

1 Hospice care provides compassion, concern, and support for the dying client and family. It may or may not be delivered in the home. Pain control is an issue for the hospice client and for any client with cancer. Advance directives are recognized in all health care settings. (Potter & Perry, 9 ed., pp. 761-762)

The nurse is assisting a client with his antiretroviral therapy. What can the nurse do to help the client take his medications as prescribed? 1. Assess the client's activities of daily living and his lifestyle routine to determine when he can most easily remember to take his medications. 2. Provide the client with brochures that explain the side effects of the medications and why it is so important for him to adhere to his medication schedule. 3. Plan for him to visit with other clients who use the same antiretroviral therapy and have them explain to the client how they handle their medications. 4. Emphasize to the client how important it is to take the medications on the schedule prescribed so that the virus will not get stronger.

1 It is important to identify the client's routines and discuss how he can adapt those routines to take his medications as prescribed. Discussing with the client the importance of taking the medications does not assist him to identify ways in which he can incorporate the medications into his daily routines. Talking to and working with another client is positive, but it still does not incorporate the medication routine into his own daily living routines. (Lewis et al., 10 ed., pp. 228-229)

The nurse is providing preoperative care for a client who is scheduled for cardiac surgery. During the preoperative preparation, what is an important nursing action? 1. Perform a thorough nursing assessment to provide an accurate baseline for evaluation after surgery. 2. Discuss with the client the steps of myocardial cellular metabolism and the anticipated surgical response. 3. Provide preoperative education regarding the mechanics of the cardiopulmonary bypass machine. 4. Discuss with the client and family the anticipated amount of postoperative chest tube drainage.

1 It is important to perform a thorough nursing assessment before the surgery. This provides a baseline for comparison of physiologic assessment data after surgery. Physiologic needs come first. The client and family do need to know about the chest drainage; however, this is client education. Providing a discussion about cellular metabolism and the mechanics of the cardiopulmonary bypass machine are not part of preoperative preparation. (Lewis et al., 10 ed., p. 739)

What is important to include in the discharge teaching plan for a 38-year-old client who has had a vaginal hysterectomy? 1. Use of birth control is no longer required. 2. Refrain from sexual intercourse for 2 months. 3. Take hormone replacement therapy. 4. Anticipate heavy vaginal bleeding.

1 It will no longer be possible to become pregnant, and birth control is no longer required. On her postoperative visit to the doctor, he or she will evaluate the healing of the cervical area and will advise her regarding sexual intercourse. There is no mention in the question that the client had her ovaries removed, so hormone replacement therapy is not necessary. Heavy vaginal bleeding might indicate hemorrhage. The client may experience a moderate amount of serosanguineous drainage immediately postoperative; once healed, she will not experience menses. (Ignatavicius & Workman, 7 ed., p. 1620)

The nurse is caring for a client who is at the beginning of her third trimester of pregnancy. The client has been admitted in preterm labor, and magnesium sulfate is being used. Contractions are occurring about every 4 to 5 minutes and lasting 1 to 11⁄2 minutes; blood pressure is 130/88 mmHg; respirations are 22 breaths/min; pulse is 98 beats/min. What nursing observation would cause the nurse the most concern? 1. Urinary output of a total of 240 mL for the past 8 hours 2. Presence of active 21 deep tendon reflexes 3. Complaints of hot flashes, nausea, and a headache 4. Blood pressure increase to 145/92 mmHg

1 Monitoring during the administration of magnesium sulfate includes urine output, which should be greater than 30 mL per hour, respiratory rate should be greater than 12 breaths/min, and deep tendon reflexes should be 21. The medication should be held or decreased if the urinary output drops to or below 30 mL per hour, as magnesium sulfate is excreted through the urine and toxicity can develop if urine output is not sufficient. Deep tendon reflexes are present, and the respiratory rate is normal. Hot flashes may occur after the medication is started, but it is not a primary concern at this time; however, the blood pressure is. The urine output is the priority. Magnesium sulfate will reduce the blood pressure once administration is started. (Lowdermilk et al., 11 ed., p.664)

The nurse is assessing a client who is at 36 weeks' gestation; this client has type 2 diabetes. The client says she is extremely upset that she will not be able to breastfeed her infant. The best nursing response would be based on what information? 1. There are no contraindications to diabetic mothers' breastfeeding their infants; it will be important for the mother to carefully monitor her blood glucose levels and insulin needs. 2. Because the mother's blood sugar level will be controlled by oral hypoglycemics, the medications are excreted in breast milk and would not be good for the baby. 3. Breastfeeding puts increased carbohydrate metabolism demands on the mother and makes it unrealistic to control her blood glucose level. 4. Offer the mother reassurance that there are some very good, nutritious infant formulas that are very close to breast milk and her infant should do well with them.

1 Mothers with diabetes can breastfeed, but they must frequently check their blood glucose levels, and insulin is recommended to maintain good blood glucose control. Oral hypoglycemics are not usually recommended for breastfeeding mothers. Breastfeeding does put an increased demand on the mother's metabolism, but this can be controlled with careful monitoring of blood glucose levels. Multiple formulas are available, but if the mother can breastfeed, that option should be made available to her. (Lowdermilk et al., 11 ed., p. 698)

The nurse is providing preoperative care for a client who will have a gastric resection. What will the preoperative teaching include? 1. An NG tube will be in place several days after surgery. 2. The client will be started on a low-residue, bland diet about 2 days after the surgery. 3. Explain the anticipated prognosis and implications that the client may have a malignancy. 4. A urinary retention catheter will be in place for 1 week after surgery.

1 NG tubes are left in place for several days after gastric resection. It is important to prevent the stomach from becoming distended and putting pressure on the suture line. A diet will be started after there is evidence of good bowel function. Diet will be clear liquids until client tolerance is determined. It is not a nursing responsibility to advise the client regarding prognosis and status of malignancy. A urinary retention catheter may or may not be in place; preferably, the client will be voiding. (Lewis et al., 10 ed., p. 918)

The nurse is planning care for a client scheduled for esophagogastroduodenoscopy (EGD) and a barium swallow. What will the nursing care plan include? 1. Anticipating the client will receive a clear liquid diet in the evening and then receive nothing by mouth (NPO status) 8 hours before the test. 2. Discussing with the client the NG tube and the importance of gastric drainage for 24 hours after the test. 3. Explaining to the client that he will receive nothing by mouth (NPO status) for 24 hours after the test to make sure his stomach can tolerate food. 4. Discussing the general anesthesia and explaining to the client that he will wake up in the recovery room.

1 NPO status before a barium swallow and an esophagogastroduodenoscopy (EGD) and a clear liquid diet the evening before the procedures are routine orders for these tests. There is no general anesthesia. The client can eat or drink as tolerated after procedure once the gag reflex returns, and there is no routine placement of NG tubes. (Lewis et al., 10 ed., p. 845)

A parent and an 8-month-old child come into a public health clinic for a well-child checkup. The parent tells the nurse the child has been crying more than usual. What information obtained during the nursing assessment would cause the nurse the most concern? 1. Crying when sucking on his bottle 2. Crying when placed in crib at night 3. On-and-off crying throughout the day 4. Crying when left at the child care center

1 Pain during feeding may indicate increased inner ear pain during sucking. With effusion in the middle ear space, negative pressure draws mucus into the middle ear in response to a child crying, or sucking on a nipple, resulting in increased pressure and pain. Crying when placed in a crib and on and off during the day is normal in childhood development. Separation anxiety is not an uncommon problem. (Hockenberry & Wilson, 10 ed., pp. 1179-1181)

The nurse is caring for a client who has been diagnosed with a bleeding duodenal ulcer. Which data identified on a nursing assessment would indicate a possible intestinal perforation and require immediate nursing action? 1. Increasing abdominal distention, with increased pain and vomiting 2. Decreasing hemoglobin and hematocrit with bloody stools 3. Diarrhea with increased bowel sounds and hypovolemia 4. Decreasing blood pressure with tachycardia and disorientation

1 Perforation is characterized by increasing distention and boardlike abdomen. There is frequently increasing pain with fever and guarding of the abdomen. Peritonitis occurs rapidly. The nurse should maintain the client NPO, keep the client on bed rest, and immediately notify the physician. Decreasing hemoglobin and hematocrit and decreasing blood pressure are associated with hemorrhage rather than perforation. Remember to select an answer that reflects what the question is specifically asking. (Lewis et al., 10 ed., p. 912)

A client has been diagnosed with pernicious anemia. What will the nurse teach this client regarding the medication he will need to take after he goes home? 1. Monthly vitamin B12 injections will be necessary. 2. Daily ferrous sulfate (in oral form) will be prescribed. 3. Coagulation studies are important to monitor the effect of medications. 4. He should reduce his intake of leafy, green vegetables to decrease vitamin K.

1 Pernicious anemia is caused by lack of intrinsic factor to effectively utilize vitamin B12 and is treated by monthly vitamin B12 injections. Ferrous sulfate is given for iron deficiency anemia. Coagulation studies are not necessary because the client is not receiving anticoagulants. Decrease in vitamin K is not necessary because the client is not receiving warfarin. (Lewis et al., 10 ed., pp. 612)

The nurse is assessing an IV site after the client has verbalized an increase of tenderness. The site is inflamed, streaks of inflammation are progressing up the inside of the client's arm, and the fluid is continuing to infuse at the prescribed rate. What is the best nursing action? 1. Remove the catheter and place warm packs on the area. 2. Lower the IV bag below the site to determine whether there is blood return in the line. 3. Determine what medications the client is receiving that may have caused the irritation. 4. Decrease the rate of the infusion to decrease the discomfort.

1 Phlebitis has developed at the site, and the catheter should be removed. If continued administration of fluids is necessary, the catheter should be restarted at another site. It does not make any difference whether the catheter is still in the vein. The catheter must be removed because of the inflammation. Assessment of medications causing the problem can be done at a later time after the current problem is resolved. The rate of infusion should remain the same. (Potter & Perry, 9 ed., p. 962)

The nurse is reviewing the health care provider's prescriptions for a new client returning from the post anesthesia care unit. The client is NPO, with a nasogastric tube and stable vital signs. Which prescription should the nurse question? 1. 20 mEq potassium IV push. 2. 1000 mL D5 1⁄2 NaCl to infuse at 125 mL/hr. 3. Assist client to dangle at bedside in morning. 4. Mefoxin 1 gm IV in 50 mL D5W over 30 minutes.

1 Potassium should never be administered by IV push. It is extremely irritating and painful at the catheter site, as well as lethal to the client when administered by IV push. It should be diluted in an IV solution and run over the time of the total infusion (1000 mL D5W with 40 mEq potassium over 8 hours), or small amounts should be given in less solution (potassium 10 mEq in 250 mL D5W to run over 3 hours). The other orders listed are all within acceptable limits for a postoperative client. (Lewis et al., 10 ed., p. 282)

The nurse is caring for an infant with an unrepaired tracheoesophageal fistula. In planning care, the nurse will identify which priority nursing goal? 1. To promote oxygen exchange 2. To prevent lung infection 3. To promote bonding 4. To replace fluids and electrolytes

1 Promoting life-saving oxygen exchange is a priority measure at this time. Prevention of infection will be appropriate after surgical repair. It is important to prevent pulmonary infection, especially aspiration, but oxygen exchange is still a priority. (Hockenberry & Wilson, 10 ed., p. 1110)

A client is receiving busulfan. The nurse would notify the physician regarding which assessment finding? 1. Persistent, nonproductive, dry cough 2. Hemoglobin 13 g/dL (130 g/L), hematocrit 38% (0.38 proportion of 1) 3. Nausea and vomiting 4. Low serum uric acid

1 Pulmonary toxicity (dry nonproductive cough, crackles, dyspnea, tachypnea) is an adverse effect of busulfan. It can lead to pulmonary fibrosis. Hemoglobin is within normal limits. The nausea and vomiting may be a side effect of the chemotherapy, and the client would be treated for this. Problems occur with a high (not low) serum acid level. (Lehne, 9 ed., p. 1226)

What nursing measure would be included in the plan of care for a client with acute kidney injury? 1. Observe for signs of a secondary infection. 2. Provide a high-protein, low-carbohydrate diet. 3. In-and-out catheterization for residual urine 4. Encourage fluids to 2000 mL in 24 hours.

1 Secondary infections are the cause of death in 50% to 90% of clients with acute renal failure. A low-protein diet is most often offered. Catheterizations are avoided. Fluids may be limited if the client is in acute kidney injury. (Lewis et al., 10 ed., pp. 1073-1075)

The nurse is caring for a client with hepatitis A. Which type of infection precaution is appropriate for this client? 1. Standard precautions 2. Droplet precautions 3. Contact precautions 4. Bloodborne precautions

1 Standard precautions are the appropriate type of infection precautions for all clients with hepatitis. Droplet precautions are not necessary for clients with hepatitis. Because hepatitis A is transmitted through the oral-fecal route, contact precautions are not necessary, except for the methods provided by standard precautions. Bloodborne precautions (part of standard precautions) are necessary for clients with hepatitis B and C (which are bloodborne), as well as for clients with hepatitis D and G. (Lewis et al., 10 ed., p. 982)

What will be important for the nurse to do when collecting a stool specimen for an occult blood (Hemoccult) test? 1. Samples should be taken from two areas of the stool. 2. Three separate stool samples will be required for accuracy of test. 3. The nurse should collect about 20 mL of stool sample. 4. Any red color on or near the specimen is considered positive.

1 Stool samples should be taken from different areas of the stool to more accurately reflect the presence of occult blood. The nurse only needs to collect a small sample of stool for the test. The test is done on the nursing unit and is not sent to the laboratory for further evaluation. Three separate samples will more accurately validate the presence of blood, but it is not required. Diets rich in red meat may cause a false-positive result. (Potter & Perry, 9 ed., p. 1156)

Ten days after delivery, a client is diagnosed with mastitis. The nurse would anticipate what assessment findings? 1. Tender, hard, inflamed area on the breast 2. Dimpled skin on the upper outer quadrant of the breast 3. Lack of milk production 4. Nipple burning during feeding

1 Swelling, erythema, and pain are found most often in the upper, outer quadrant of the breast. Dimpled skin (orange peel appearance) is a potential sign of breast cancer. Nipple burning is related to positioning and initiation of the let-down reflex during feeding. Milk production starts about 4 days after delivery and is not related to the development of mastitis. (Lowdermilk et al., 11 ed., p. 625)

A client has pain at the peripheral IV site. The nurse determines the IV is not infusing; assesses the site; and finds the area swollen, pale, and cool to touch. What is the best nursing action? 1. Discontinue the IV and apply warm, moist packs to the involved area. 2. Slow the IV infusion and see whether the swollen area decreases. 3. Notify the health care provider regarding the status of the IV. 4. Discontinue the IV and start another IV in the same vein, distal to the current site.

1 The IV is infiltrated and should be discontinued. A warm, moist pack can be applied for client comfort. The IV should be discontinued but not restarted distal to the previous site; it should be started proximal to or above the current infiltrat

The physician orders an IV piggyback of cefotetan 1 gm in 100 mL D5W to run over 30 minutes. The drop factor for the tubing is 10 gtt/mL. At what rate in drops per minute would you run the IV? 1. 33 gtt/min 2. 25 gtt/min 3. 12 gtt/min 4. 50 gtt/min

1 The calculation would be 100 mL/30 min x 10 gtt/mL = 33.0 or 33 gtt/min. (Potter & Perry, 9 ed., pp. 618-620)

The nurse is reviewing the chart of a client who recently had a cervical biopsy. The test results indicate Tis, N0, M0. How will the nurse interpret this information? 1. The cancer is in situ, which means it is localized and not invasive at this time. 2. The origin of the cancer is probably in the uterus, and further testing will be necessary. 3. The lymph nodes are involved, and the presence of distant metastasis cannot be determined. 4. There is no cancer present; the tissue was normal.

1 The cancer is in situ (Tis), which means the cancer is localized to the cervix; there is no evidence of either lymph node involvement (N0) or of invasive activity (M0). Because it is not invasive at this time, there is no immediate need for treatment. The client's history and risk factors will influence the decisions for further diagnostic testing or treatment. (Lewis et al., 10 ed., p. 241)

A client has had her blood pressure evaluated weekly for month. At the end of the month, the nurse averages out the weekly blood pressures at 150/96 mmHg. The client is 20 pounds (9.1 kg) overweight, and her cholesterol is 240 mg/dL (6.22 mmol/L). What is important information for the nurse to include in the teaching plan for this client? 1. Refer her to the doctor for further follow-up and medications. 2. Increase the fiber in her diet and begin a daily 30-minute workout. 3. Reduce her sodium intake and decrease the dietary calories that come from fat. 4. Reduce her cholesterol intake for 1 month and check her BP 3 times a week.

1 The client should be referred for further evaluation of blood pressure. The blood pressure is definitely elevated, the client is overweight, and she has an increased level of cholesterol. A multifocal approach is necessary to control the blood pressure. Because of the multiple risk factors, increasing fiber in the diet and exercise would not likely be sufficient to reduce the hypertension. Neither would dietary changes. This patient needs a multifocal approach. (Ignatavicius & Workman, 8th ed., pp. 710-712)

The nurse would explain to a patient with genital herpes that he or she is most contagious at what stage? 1. When vesicles rupture and release transudate 2. When superficial, painful ulcers appear 3. When yellow vaginal drainage is present 4. When pustules become inflamed and erythematous

1 The herpes simplex virus (HSV-2) is concentrated in the vesicles, and therefore the infection is highly contagious when this clear fluid is released. The lesions rupture and form shallow, moist ulcerations (not pustules) that eventually crust, allowing epithelialization of the erosions to occur. There is no yellow drainage. The primary infection is associated with local inflammation and pain and is often accompanied by systemic symptoms of fever, headache, malaise, myalgia, and regional lymphadenopathy. (Lewis et al., 10 ed., p. 1232)

Which nursing observations indicate that a male client with a kidney stone is experiencing renal colic? 1. Severe flank pain radiating toward the testicles 2. Stress incontinence with full bladder 3. Hematuria and severe burning on urination 4. Enuresis with hyperalbuminuria

1 The most characteristic symptom of renal colic is sudden, severe pain. The client may also exhibit nausea and vomiting, pallor, and diaphoresis during the acute pain episode. Hematuria and burning on urination are associated with UTIs, although there may be some bleeding with the passage of the renal stone. (Lewis et al., 10 ed., p. 1045)

What evaluation is important in the preoperative nursing assessment of a client with a severely herniated lumbar disk? 1. Movement and sensation in the lower extremities 2. Leg pain that radiates to both lower extremities 3. Reflexes in the upper extremities 4. Pupillary reaction to light

1 The movement and sensation should be evaluated before surgery to serve as a baseline for comparison during the postoperative recovery period. Movement of the legs and assessment of sensation should be unchanged compared with the preoperative status. Radiating leg pain is diagnostic of the condition, and assessing it before surgery is not as beneficial as determining movement and sensation. (Lewis et al., 10 ed., p. 1504)

The nurse is teaching a client with a pelvic inflammatory disease. The nurse instructs the client to sleep with her head elevated about 45 degrees. What is the rationale behind instructing the client to sleep in this position? 1. Assists to localize drainage in the lower abdomen 2. Decreases abdominal muscle tension 3. Makes coughing and deep breathing more effective 4. Prevents scarring of the fallopian tubes

1 The nurse teaches the client to maintain a semi-Fowler's position to prevent or decrease movement of the contaminated fluid to the upper abdomen and the area of the diaphragm. (Lewis et al., 10 ed., p. 1251)

The nurse is assessing a client who had a transurethral resection of the prostate (TURP) 6 hours ago. He has a urinary catheter with continuous bladder irrigation running. What nursing observations would indicate a complication is developing? 1. Catheter drainage of 50 mL in the past hour and increase in suprapubic pain 2. Dark, grossly bloody catheter drainage with pieces of tissue 3. Client states that he feels like he needs to void 4. Moderate amount of bloody discharge from around the catheter

1 The primary complication is the obstruction of the urinary catheter with clots or tissue. There should be a large amount of drainage from the catheter because the irrigating fluid is infusing into the bladder. The catheter drainage should be closer to 300 to 400 mL/ hr. It is not unusual for the drainage to be grossly bloody on the operative day, but it should begin to clear over the next 24 hours. It is common to have a feeling of needing to void with a catheter in place. (Lewis et al., 10 ed., p. 1268)

The nurse assigned to the care of newborn infants understands the importance of keeping these infants swaddled in a warm blanket to prevent heat loss. Why is this important in the care of the newborn? 1. Chilling leads to increased heat production and greater oxygen needs. 2. The newborn's metabolic rate is decreased. 3. Evaporation will affect the newborn's ability to feed. 4. The newborn will sleep more comfortably.

1 The priority is to prevent chilling, which leads to greater oxygen consumption and to an increased utilization of glucose and brown fat. Chilling also increases caloric needs, decreases surfactant production, and promotes a tendency to develop acidosis. Evaporation occurs when the newborn is wet with amniotic fluid. (Hockenberry & Wilson, 10 ed., p. 244)

The nurse is caring for a client who has had a right-sided stroke. What would be appropriate nursing care for this client? 1. Performing passive ROM exercises to affected side, active ROM on unaffected side 2. Placing food on the affected side of the client's mouth 3. Applying hot packs to the right leg to decrease muscle spasms 4. Turning client every 2 hours and maintaining position on the right side for 2 hours

1 The rehabilitation program for a client who has had a stroke includes actions that prevent deformity (passive range of motion on affected side and active ROM on the unaffected side) and prevent complications that may be associated with immobility. The client's affected side should be protected, and food should be placed on the unaffected side so that the client can control it. The client should be turned to his right (affected side) but for shorter periods of time. Hot packs would not be applied to decrease spasticity or spasms. (Lewis et al., 9 ed., p. 1393)

The nurse is monitoring a client who is experiencing an acute asthma attack. What observations would indicate an improvement in the client's condition? 1. Respiratory rate of 18 breaths/min 2. Pulse oximetry of 88% 3. Pulse rate of 110 beats/min 4. Productive cough with rapid breathing

1 The respiratory rate is within normal limits at 18 breaths per minute. The option for the pulse oximetry is too low. The pulse rate is too high to indicate improvement, and the productive cough with rapid breathing is not as significant as the decrease in respiratory rate. (Ignatavicius & Workman, 8 ed., pp. 550-556)

The nurse is providing discharge teaching to a 20-year-old mother who has had her first male child. Which statement by the mother demonstrates that she understands the discharge teaching regarding his circumcision? 1. "I will observe the whitish yellow drainage on his penis, but I will not remove it." 2. "I will bring him back to the clinic in 3 days to have the drainage removed." 3. "I will use antibiotic ointment on his penis with every diaper change." 4. "I will rub the area briskly with a washcloth to remove the discharge."

1 The whitish yellowish exudate around the glans penis is granulation tissue and is normal. It will usually disappear within 2 to 3 days. It is not an infection; therefore antibiotic ointment is not appropriate. Soap and water cleansing after each diaper change is appropriate. A small sterile petrolatum gauze dressing may be applied to the area during the first 2 to 3 days (Gomco and Mogen clamp). If a PlastiBell was used, keep area clean; application of petrolatum jelly is not necessary; plastic ring will dislodge when area has healed (5 to 7 days). (Hockenberry & Wilson, 10 ed., p. 274)

A 10-lb (4536-g) newborn of a mother with diabetes is admitted to the intensive care unit because of hypoglycemia. The baby's mother is concerned that he will have diabetes. The most appropriate response by the nurse is that the baby: 1. May have an increased risk of acquiring metabolic syndrome in childhood or early adulthood 2. Will not be at risk for developing diabetes 3. Will have to follow a diabetic diet to avoid complications 4. Will not need to be monitored closely during his early childhood years

1 There is evidence of an increased risk of acquiring metabolic syndrome (i.e., obesity, hypertension, dyslipidemia, and glucose intolerance) in childhood or early adulthood. Nursing care should focus on healthy lifestyle and prevention later in life. The hypoglycemia is transient. Hypoglycemia is a rebound response because the maternal hyperglycemia stimulates insulin production in the fetus, and the newborn is left with excessive insulin after the maternal glucose supply is ended at birth. The infant does have an increased chance of becoming diabetic during childhood, especially of symptoms of metabolic syndrome appear. (Hockenberry & Wilson, 10 ed., p. 396)

A client with AIDS has several cutaneous lesions identified as Kaposi sarcoma. How will the nurse care for these areas? 1. Gently cleanse the areas, keeping them dry and free of abrasions. 2. Place sterile, saline-soaked gauze over the areas. 3. Apply a topical corticosteroid cream. 4. Decrease infection by applying an antibiotic ointment.

1 There is no specific nursing care required for Kaposi sarcoma lesions. Gently cleansing the area and protecting it from abrasive trauma, which could open the lesions, would be appropriate. Dressings, steroid cream, and antibiotic ointment are not indicated. Standard precautions should be followed when caring for the lesions. (Lewis et al., 10 ed., pp. 222-223)

A nurse is caring for a client who received a penicillin injection about 15 minutes earlier. The client complains of itching around the mouth, and this rapidly progresses to severe dyspnea and respiratory distress. What are the priority nursing actions? 1. Anticipate need for possibility of endotracheal intubation, begin oxygen, call for assistance, and obtain emergency cart. 2. Place the client in supine position and assess for patent airway and presence of breath sounds. 3. Start oxygen at 6 L/min via nasal cannula; review chart for history of a penicillin allergy. 4. Place the client in semi-Fowler's position, perform a chin lift to open the airway, and assess for air movement.

1 This situation best describes an anaphylactic reaction to the penicillin injection. This is a rapidly occurring response that may result in a life-threatening occlusion of the airway (laryngeal edema, bronchospasm). The client may require an emergency tracheotomy or endotracheal intubation. Time should not be wasted on checking the chart or further assessment. The client is obviously in distress, and the presence of breath sounds will not be significant in determining the nursing actions. The chin lift is not an effective method for opening an airway that is occluded from edema or bronchospasm. Epinephrine is the drug of choice for a serious anaphylactic reaction. (Lewis et al., 10 ed., p. 201)

At the shift hand-off report, a nurse is told that one of her clients is becoming tolerant to his pain medication. What nursing observation would be in agreement with this conclusion? 1. The current medication order, which has previously been effective, is no longer providing adequate pain relief. 2. The client becomes irritable and confused before the next scheduled dose of medication. 3. Pain medication is being administered every 3 to 4 hours around the clock for adequate pain relief. 4. The client is sleeping and arouses with physical and verbal stimulation but is very lethargic.

1 Tolerance occurs when there is a decrease in the response to a drug after repeated drug administration. More of the analgesic is needed to maintain the same level of pain control. Requiring pain medication around the clock for adequate pain relief is not tolerance because the client is obtaining proper pain relief. Irritability and confusion before the next dose may be an adverse reaction to the medication, or another problem may be occurring. Sleepiness and lethargy may indicate that the client is overmedicated. (Burchum & Rosenthal, 9 ed., pp. 74-75)

The client returns to his room after liver biopsy. The nurse positions the client on his right side and assesses for bleeding. What is a priority nursing assessment? 1. Assess the vital signs. 2. Observe for frank bleeding. 3. Check pretest prothrombin time and partial thromboplastin time values. 4. Determine adequacy of urinary output.

1 Vital signs must be assessed first if the nurse is monitoring the client for bleeding or shock. The nurse would do all of the assessments with vital signs as the priority. (Lewis et al., 10 ed., p. 845)

An unknown chemical was splashed into a client's eyes. What is most important for the nurse to tell the client to do immediately? 1. Rinse the eye with a large amount of water or saline solution. 2. Put a pad soaked in sterile saline solution over the eye. 3. Go to the closest emergency department. 4. Have a coworker visually check the eye for a foreign body.

1 When an unknown solution has been splashed in the eyes, it is most important to remove as much of the solution as possible by rinsing the eyes with large amounts of water or normal saline solution, if available. Placing a pad soaked in sterile saline solution over the eye, going to the closest emergency department, and having a coworker visually check the eye for a foreign body do not address removing the excess solution from the eye to prevent further damage. (Lewis et al., 10 ed., p. 371)

A client is admitted to the inpatient psychiatric unit for medically monitored detoxification from alcohol. Which of the following actions would be included in the client's plan of care? 1. Encourage increased fluid intake. 2. Order a high-protein, high-fat diet. 3. Provide a high-sodium, low-carbohydrate diet. 4. Encourage ambulation and deep breathing.

1 When assisting in the medical treatment of alcohol withdrawal, the nurse should encourage intake of fluids. Alcohol depletes the body of fluid, and detoxification is usually smoother if the client takes fluids readily. A high-protein and high-carbohydrate diet would be encouraged, because alcoholic clients often have poor nutrition and become debilitated. (Halter, 7 ed., p. 421)

A client is receiving NPH insulin 20 units subcutaneously at 0700 hours daily. At 3 pm, the nurse finds the client apparently asleep. What priority action should the nurse perform to assess for a hypoglycemic reaction? 1. Feel the client and bed for dampness. 2. Observe the client for Kussmaul respirations. 3. Smell the client's breath for acetone odor. 4. Note if the client is incontinent of urine.

1 When clients are sleeping, the only observable symptom of hypoglycemia is diaphoresis. Kussmaul breathing and acetone odor to breath are indicative of hyperglycemia. Incontinence is not associated with hypoglycemia, and polyuria may be associated with hyperglycemia. (Lewis et al., 10 ed., p. 1146)

A patient with a pituitary tumor is treated with a transsphenoidal hypophysectomy. What would be a priority postoperative action? 1. Ensure that any clear nasal drainage is tested for glucose. 2. Maintain the patient flat in bed to prevent cerebrospinal fluid (CSF) leak. 3. Assist the patient with tooth brushing to keep the surgical area clean. 4. Encourage deep breathing and coughing to prevent respiratory complications.

1 With a transsphenoidal hypophysectomy the pituitary gland is removed. CSF leaks and epistaxis are common postoperative complications, and it is important that any clear fluid draining from the nose is tested for glucose. Postoperative care includes elevating the head of the bed at all times to a 30-degree angle, and the client should avoid sneezing, coughing, and tooth brushing for at least 10 days to prevent a CSF leak. (Lewis et al., 10 ed., p. 1159)

The nurse is assisting a client to ambulate. Upon standing at the bedside, the client becomes weak, says "I feel dizzy," and sits back down on the bed. What should be the nurses next action? 1. Lay the client down in bed. 2. Obtain a blood pressure. 3. Ask the client to try again. 4. Find additional help for ambulation.

1 With dehydration or fluid deficit, orthostatic hypotension may occur. After the client is safely back in bed, the nurse should assess for other symptoms of fluid volume deficit: decreased blood pressure, weight loss, imbalance of urine output and fluid intake, and dry skin and mucous membranes. Ambulation should be avoided until the symptoms have subsided. (Lewis et al., 10 ed., p. 282)

In taking the health history of a client with severe painful osteoarthritis, which question should the nurse ask about their condition? Select all that apply. 1. Do you utilize and complementary or alternative therapies for your arthritis? 2. Do you have any joint deformities from your arthritis? 3. Did your symptoms begin during childhood? 4. Have you ever taken corticosteroids or immunosuppressive medication for your arthritis? 5. Have you ever developed kidney stones from your condition?

1, 2 Osteoarthritis has a gradual onset and affects weight-bearing joints with pain that is more pronounced after exercise. The onset of osteoarthritis is gradual, not sudden, and commonly occurs until after age 50. The client will usually complain of increased stiffness in the morning and after periods of inactivity, with improvement after activity. Joint pain generally worsens with joint use, and in the early stages of osteoarthritis, joint pain is relieved by rest. Alternative therapies are often used and need to be evaluated as to any joint deformities from the condition. The development of kidney stones would be secondary to gout, and the use of steroids and immunosuppressive medication are for rheumatoid arthritis. (Lewis et al., 10 ed., p. 1517)

The nurse notes that a client is quite suspicious during an assessment interview and believes that her family is under investigation by the CIA. What would be appropriate nursing interventions with this client? Select all that apply: 1. Use active listening skills to seek information from the client. 2. Encourage the client to describe the problem as she sees it. 3. Ask the client to tell you exactly what she thinks is happening. 4. Tell the client that she is delusional and you can help her. 5. Explain to the client that most people are not investigated by the CIA or FBI. 6. Reassure the client that you are not with the CIA.

1, 2, 3 The client is demonstrating paranoid behavior, which necessitates a matter-of-fact approach that is nonjudgmental and accepting of the client's statements and shows the nurse's willingness to listen attentively to the issue. Telling the client that she is delusional, explaining that most people are not investigated by the CIA or FBI, reassuring the client that you are not with the CIA do not help the paranoid client gain trust to talk with the nurse. (Halter, 7 ed., p. 465)

The nurse would identify which of the following clients to be at an increased risk for the development of a fecal impaction? Select all that apply: 1. Post barium enema 2. Obese client in traction 3. Poorly hydrated older adult 4. Client receiving opioid medications 5. Three days after colostomy 6. Acute appendicitis

1, 2, 3, 4 A barium enema procedure can lead to fecal impaction caused by barium left in the colon. An obese client in traction who is immobile and a poorly hydrated older adult (decreased motility and fluid intake) may also experience fecal impaction. These three conditions can contribute to the development of an impaction that may require manual removal. Opioid medications are central nervous system depressants, thus slowing peristalsis and contributing to constipation and fecal impaction. Acute appendicitis would be a condition in which no rectal enemas or manipulation would be indicated. (Lewis et al., 10 ed., pp. 934-935)

The nurse is preparing discharge teaching for a client with a diagnosis of gastroesophageal reflux disease (GERD). What would be important for the nurse to include in this teaching plan? Select all that apply. 1. Elevate the head of the bed. 2. Decrease intake of caffeine products. 3. Discuss strategies for weight loss if overweight. 4. Increase fluid intake with meals. 5. Take omeprazole at bedtime. 6. Eat a bedtime snack of milk and protein.

1, 2, 3, 5 Each of these actions will help either neutralize the acid in the stomach or decrease the physiologic reflux. Proton pump inhibitors will decrease the amount of acid produced in the stomach. Increased fluids with meals will exacerbate the problem, as will eating before going to bed. (Lewis et al., 10 ed., pp. 901-902)

The nurse received handoff for a client returning from a cardiac angiogram and begins the initial assessment. The right femoral groin dressing has a dime-sized area of blood. What additional actions should the nurse perform? Select all that apply. 1. Assess peripheral pulses in both legs and feet. 2. Mark the dressing with a pen, circling the bloody drainage. 3. Hold pressure on the dressing site for 20 minutes. 4. Assess blood pressure. 5. Place the client in a high-Fowler position.

1, 2, 4 A cardiac catheterization includes the insertion of a large sheath into the femoral artery. There is a risk of hemorrhage after the procedure along with a loss of circulation distal to the insertion site. Priority care includes frequent assessment of circulation to the extremity. Peripheral pulses, color, and sensation should be evaluated in both feet as a comparison. Observation of the insertion site for hematoma and bleeding is also important. The nurse should mark the site, and if it enlarges, pressure should be applied. Monitor of the ECG and vital signs in important for evaluation of instability. The head of bed should remain flat until the groin site has developed hemostasis. (Lewis et al., 10 ed., pp. 677, 729)

The nurse is documenting information regarding an IV insertion. What information is important to include? Select all that apply. 1. Time and date of insertion. 2. Type of catheter and size. 3. Name of vein used. 4. Status of fluid infusing. 5. Protective measures used. 6. Who ordered the IV and at what time.

1, 2, 4 The specific name of the vein is not necessary, but the general location of the site is important. Standard precautions are used for everyone, and it is not necessary to chart that they were used. The order and time for the IV should be on the client's chart; it is not necessary to repeat it in the documentation. The question did not ask for all the

The nurse is monitoring a client after thrombolytic therapy has been initiated. Shortly after the infusion is started, the client becomes confused, disoriented, cool, and clammy. The heart rate progressively increases to 120 and blood pressure drops to 60/40. What actions should the nurse take? Select all that apply. 1. Stop the thrombolytic 2. Apply oxygen 3. Raise the head of the bed 4. Call for assistance 5. Reorient the client

1, 2, 4 The symptoms the client is demonstrating support that he/she is hemorrhaging, the most common complication of thrombolytic therapy. The nurse needs to support oxygenation and perfusion by applying oxygen, lowering the head of the bed, and increasing fluids. The thrombolytics should be stopped and additional help given if needed since the client is very unstable. (Lewis et al., 10 ed., p. 724)

The nurse is caring for a client with a history of heart failure. Which statements by the client require additional inquiry? Select all that apply. 1. "I've noticed that I've gained 3 lbs. This week." 2. "I sleep best in my recliner chair." 3. "I've noticed that the swelling in my feet seems less." 4. "I cannot make it through the grocery store without resting." 5. "I often have to use the restroom at night."

1, 2, 4, 5 Symptoms of heart failure include weight gain, orthopnea, fatigue, edema, and nocturia. Additionally the client can experience tachycardia, skin changes, behavioral changes, and chest pain. (Lewis et al., 10 ed., pp. 742-743)

The nurse is performing a dressing change on a client who has a Staphylococcus infection in an abdominal incision. Which infection-control precautions will the nurse implement? Select all that apply. 1. Wear clean gloves to remove the old dressing. 2. Put on a gown when entering the room. 3. Wear a face shield. 4. Dispose of the gown and mask in container outside the client's door. 5. Leave all extra dressing supplies in the room. 6. Carefully cleanse the stethoscope and scissors before taking them out of the room.

1, 2, 5 Contact precautions would require the nurse to wear clean gloves to remove the old dressing, put on a gown when entering the room, and leave all extra dressing supplies in the room. A face shield is not necessary unless splattering of fluids is anticipated. The gown and mask should be disposed of in the client's room; they should not be worn outside the room and should be disposed according to hospital policy. The stethoscope and scissors should not be taken out of the client's room. (Potter & Perry, 9 ed., p. 458)

The nurse is evaluating a client's response to hemodialysis. Which laboratory values will indicate the dialysis was effective? Select all that apply. 1. Serum potassium level decreases from 5.4 to 4.6 mEq/L. (5.4 to 4.6 mmol/L) 2. Serum creatinine level decreases from 1.6 to 0.8 mg/dL. (111.44-70.72 umol/L) 3. Hemoglobin increases from 10 to 12 g/dL. (100-120 g/L) 4. White blood cells increase from 5000 to 8000/mm3. (5.00-8.00 3 109/L) 5. BUN decreases from 110 to 90 mg/dL. (39.27-32.13 mmol/L)

1, 2, 5 Primary action of hemodialysis is to clear nitrogenous waste products. The creatinine and BUN provide a measure of how effective the dialysate was in removing the waste products. Electrolytes are altered with a decrease in potassium. Hemoglobin, white blood cells, and sedimentation rate are not affected, these cells are too large to diffuse through the pores of the dialysate membrane. (Lewis et al., 10 ed., p. 1084)

The nurse is planning to educate colleagues on best practices in decreasing central line infections. What practices should be included in staff education? Select all that apply. 1. Follow agency policy for dressing and tubing changes. 2. Use clean technique when changing caps and dressings. 3. Report sites that are reddened. 4. Change dressings that have moisture. 5. Gauze dressings need changing less frequently

1, 3, 4 Central lines have a high rate of infection in hospitalized clients. The Centers for Disease Control and Prevention offers guidelines for prevention of intravascular catheter-related infections. The nurse should follow the agency policy for dressing and tubing changes, which will guide the nurse when in practice. Sterile technique should be used, not clean technique, and gauze dressings need to be changed more frequently, since they are porous. Moisture increases the risk of infection. All signs of infection should be reported, including temperature, increased white blood cells, and redness. (Lewis et al., 10 ed., p. 217)

The nurse recognizes which of the following conditions as an oncologic emergency? Select all that apply. 1. Cardiac tamponade 2. Leukopenia 3. Syndrome of inappropriate antidiuretic hormone 4. Hypercalcemia 5. Hypophosphatemia 6. Tumor lysis syndrome

1, 3, 4, 6 Metabolic emergencies, including SIADH, hypercalcemia, and TLS; infiltrative emergencies, including cardiac tamponade; and obstructive emergencies are life-threatening complications of cancer or cancer therapy. Leukopenia is an expected side effect. Hypophosphatemia is not common. (Lewis et al., 10 ed., p. 263)

A client is scheduled for a total hip replacement, and he has a history of using several herbal and vitamin products. What would the nurse advise the client to discontinue at least 2 weeks before surgery? Select all that apply. 1. Garlic 2. Vitamin C 3. Ginger root 4. St. John's wort 5. Ma huang 6. Black cohosh

1, 3, 5 Garlic and ginger root can prolong bleeding time by suppressing platelet aggregation. Ma huang is ephedra and affects the blood pressure. St. John's wort, vitamin C, and black cohosh are safe to continue before surgery. (Burchum & Rosenthal, 9 ed., pp. 1321-1328)

The nurse is instructing a client on dietary restrictions for the management of gout. The instructions include elimination of which of the following foods? Select all that apply. 1. Asparagus 2. Almonds 3. Chicken 4. Grapefruit 5. Red wine 6. Salmon

1, 3, 5 Purine-rich foods (e.g., shellfish such as crab and shrimp; vegetables such as lentils, asparagus, and spinach; meats such as beef, chicken, and pork) will not cause gout but can trigger an acute attack if a person is susceptible to gout. Other foods listed are not considered high in purine. (Lewis et al., 10 ed., p. 1532)

The nurse understands that the following clinical findings are indications for dialysis. Select all that apply. 1. Volume overload 2. Blood urea nitrogen level of 18 mg/dL (6.43 mmol/L) 3. Potassium level of 6.8 mEq/L (mmol/L) 4. Glomerular filtration rate of 25 mL/min 5. Metabolic acidosis 6. Creatinine level of 5.0 mg/dL (442.0 umol/L)

1, 3, 5, 6 Indications for dialysis include volume overload, weight gain, hyperkalemia, metabolic acidosis, and rising BUN (10-20 mg/dL [3.57-7.14 mmol/L]) and serum creatinine (0.5-1.5 mg/dL [44.2- 132.6 umol/L]) levels, along with decreased GFR rate (less that 15 mL/min). A potassium level of 6.8 is hyperkalemia, and a BUN of 18 mg/dL (6.43 mmol/L) is within normal range. (Lewis et al., 10 ed., p. 1084)

Which of the following statements are correct about latex allergy? Select all that apply. 1. Typical reactions include skin redness, urticaria, and rhinitis. 2. Latex allergy involves only type I allergic reactions. 3. The more frequent the exposure to the latex, the more likely a person will develop an allergy. 4. Hand lotions should be applied before putting on gloves to reduce exposure. 5. Wash hands with mild soap after removing gloves. 6. Persons should wear a medic alert bracelet and carry an epinephrine pen.

1, 3, 5, 6 It is important for the nurse to recognize symptoms of latex allergy—skin rash, hives, flushing, and itching; nasal, eye, and sinus symptoms; asthma and (rarely) anaphylaxis. The nurse should also be aware of latex-containing products—gloves, blood pressure cuffs, stethoscopes, tourniquets, IV tubing, syringes, electrode pads, oxygen masks, tracheal tubes, colostomy and ileostomy tubes, urinary catheters, anesthetic masks, and adhesive tape. The use of nonlatex gloves and powder-free gloves, along with the elimination of oil-based hand creams or lotions when wearing gloves, can reduce exposure. Always wash hands after removing gloves. Individuals with latex allergy should wear a medic alert bracelet if latex sensitive. The more frequent and prolonged the exposure to latex, the greater the likelihood of developing latex allergy. There are two types of latex allergy: type IV allergic contact dermatitis (delayed reaction) and type I allergy reaction (immediate response). (Lewis et al., 10 ed., pp. 203-204)

The nurse is caring for a client with venous blood pooling in the lower extremities caused by chronic venous insufficiency. The nurse would identify what assessment data that would correlate with this diagnosis? Select all that apply. 1. Stasis dermatitis 2. Diminished peripheral pulses 3. Peripheral edema 4. Gangrenous wounds 5. Venous stasis ulcers 6. Skin hyperpigmentation

1, 3, 5, 6 Long-term impairment of venous return leads to chronic venous insufficiency that is characterized by leathery, brawny appearance from erythrocyte extravasation to the extremity, persistent edema, stasis dermatitis, and pruritus. Venous leg (stasis) ulcers characteristically form near the ankle on the medial aspect, with wound margins that are irregularly shaped with tissue that is a ruddy color. Gangrenous wounds and diminished peripheral pulses are associated with arterial occlusive disease. (Lewis et al., 10th ed., pp. 826-828)

The nurse is preparing discharge teaching for a woman newly diagnosed with SLE. What will be important for the nurse to include in the teaching plan? Select all that apply. 1. Wear sunscreen and protective clothing when in direct sunlight. 2. Avoid nonsteroidal antiinflammatory drugs to prevent bleeding episodes. 3. Plan activities that encourage range of motion in extremities. 4. Advise the client that pregnancy is contraindicated. 5. Observe fingertips for changes in circulation. 6. Help the client prioritize self-care activity.

1, 3, 5, 6 The client with SLE is photosensitive and needs protection from sunlight. The client needs to keep joints mobilized because of the invasion of the lupus erythematosus cells into the joints. This condition also affects the circulation in the fingertips, and Raynaud's phenomenon is characteristic of the disease. Fatigue is a problem, and the client needs to prioritize activities of daily living. NSAIDs are frequently used to reduce the musculoskeletal discomforts. Although individual disease progression and course of therapy need to be considered in consultation with health care providers, there is no specific contraindication to pregnancy. The woman should be advised regarding individual risk, but she can carry and deliver a healthy infant. (Lewis et al., 10 ed., pp. 1538-1542)

What are the best nursing actions in caring for a young client with appendicitis before surgery? Select all that apply: 1. Maintain bed rest. 2. Offer full liquids to maintain hydration. 3. Keep patient still and position with right leg flexed 4. Position on left side, apply a warm K-Pad to the abdomen. 5. Administer morphine intravenously to relieve pain. 6. Keep the client NPO and maintain a peripheral IV for fluid replacement.

1, 3, 6 Before surgery, keep the patient still, minimize movement, and offer to flex the right leg, which may increase comfort. Keep the client NPO because of the impending surgery; initiate IV fluid replacement. Maintain the client on bed rest; do not apply any type of heat to the abdomen. Narcotics are given cautiously if at all because the analgesic may mask the symptoms of rupture. (Lewis et al., 10 ed., p. 942)

The nurse is assessing a child with a tentative diagnosis of appendicitis. The nursing assessment is most likely to reveal what characteristics concerning the pain? Select all that apply. 1. Colicky, cramping abdominal pain located around the umbilicus 2. Tenderness in the left lower quadrant, associated with decreased bowel sounds 3. Nausea, vomiting, and anorexia after onset of pain 4. Gnawing pain radiating through to the lower back, with severe abdominal distention 5. Sharp pain with severe gastric distention, frequently associated with hemoptysis 6. Tenderness at McBurney's point

1, 3, 6 Colicky, cramping abdominal pain located around the umbilicus often noted as "referred pain" for its vague periumbilical localization is characteristic of appendicitis. The most common point of tenderness is over the area known as McBurney's point. Typically, nausea, vomiting, and anorexia follow onset of pain. Diarrhea, poor feeding, lethargy, and irritability may accompany peritonitis. Tenderness in the right lower quadrant (not the left) that occurs during palpation or percussion is called Rovsing's sign. Gastric distention and gnawing radiating pain are not common signs of appendicitis; gnawing pain is more characteristic of ulcers. Hemoptysis is not seen in appendicitis but in pulmonary edema. Remember, all the items in an option have to be correct if it is the correct answer. (Hockenberry & Wilson, 10 ed., p. 1079)

In planning discharge teaching for the client who has undergone a gastrectomy, the nurse includes what information regarding dumping syndrome? Select all that apply. 1. Symptoms may include nausea, vomiting, weakness, and abdominal cramping. 2. The client should eat three to four small meals per day. 3. Consumption of fluids should be very limited with the meal. 4. The client should increase the amount of complex carbohydrates and fiber in the diet. 5. Activity will decrease the problem; it should be scheduled about 1 hour after meals. 6. You may need to take a multivitamin with calcium and iron supplements.

1, 3, 6 Dumping syndrome is not uncommon after a combination bariatric surgery. Common symptoms include nausea, vomiting, abdominal cramping, weakness, palpitations, diaphoresis, and dizziness. Precautions such as limiting the amount of fluids taken with a meal should be implemented. The intake of foods high in iron, calcium, and B12 may not prevent the vitamin or mineral deficiencies, because the problem is with the absorption of these elements; supplements may be necessary. The client should plan to eat six small meals a day to decrease distention of the remaining stomach and limit the ingestion of carbohydrates, which may cause hyperglycemia. Rest after a meal, rather than activity, is helpful because it may prevent gastric contents from emptying too rapidly into the small intestine, thus generating the symptoms. (Lewis et al., 10 ed., pp. 917-918)

Which of the following would be appropriate discharge instructions for the client that has just been diagnosed with polycythemia vera? Select all that apply. 1. "You can expect to have repeated phlebotomies." 2. "Take an iron supplement daily." 3. "Low-dose aspirin may be prescribed by your health care provider." 4. "A warm bath may be used to decrease generalized pruritus." 5. "Avoid crowds due to increased risk of infection secondary to your low white blood cell (WBC) count." 6. "Try to keep well hydrated by drinking at least 2 liters of fluid per day."

1, 3, 6 Phlebotomy is the main method of treatment to reduce the hematocrit. Generally, the client will have 300 to 500 mL of blood removed each time, aimed at keeping the hematocrit level between 45% and 48%. Iron supplementation is to be avoided because it raises the hematocrit, and low-dose aspirin is often prescribed to help prevent clot formation. Warm baths may precipitate pruritus, not relieve it. Cool baths may be beneficial, and alpha-interferon is sometimes prescribed for severe itching. There is no risk of infection with crowds, as the WBC count is increased instead of decreased, and the client should maintain normal hydration, which helps decrease clot formation. (Lewis et al., 10 ed., p. 621)

The nurse would anticipate that Rho(D) immune globulin would be administered in which of the following situations? Select all that apply. 1. After chorionic villus sampling 2. History of Rh exchange transfusion in a previous pregnancy 3. Following a spontaneous abortion 4. Current pregnancy of an Rh-negative infant 5. Positive indirect Coombs' test at 28 weeks 6. Following delivery of an Rh-positive infant

1, 3, 6 Rho(D) immune globulin is administered to Rh-negative women whose indirect Coombs tests are negative. Women who are sensitized are not given Rho(D) immune globulin; this would be a woman with a history of Rh exchange transfusion in a previous pregnancy and a positive indirect Coombs' test at 28 weeks' gestation (a negative indirect Coombs test means sensitization has not occurred). Also, it is not necessary to give Rho(D) immune globulin if the infant is Rh negative. It is given within 72 hours of an Rhnegative mother's delivery of an Rh-positive infant, after amniocentesis, chorionic villus sampling, ectopic pregnancy, miscarriage, elective abortion, abruptio placentae, placenta previa, or trauma at 28 weeks' gestation (with a negative indirect Coombs' test result). (Hockenberry & Wilson, 10 ed., p. 325)

The nurse is caring for a client who had a stroke (brain attack) 3 months ago and is taking warfarin 5 mg by mouth (PO). The client tells the nurse she has started taking some herbal and vitamin supplements. She gives the nurse a list of the supplements she is taking. What supplements would cause concern for the client who is on warfarin? Select all that apply. 1. Garlic 2. Cyanocobalamin (vitamin B12) 3. St. John's wort 4. Vitamin E (alpha tocopherol) 5. Saw palmetto 6. Ginkgo biloba

1, 4, 6 Garlic, ginkgo biloba, and vitamin E may interfere with platelet aggregation and increase the risk for bleeding in clients who are taking warfarin. (Burchum & Rosenthal, 9 ed., pp. 1321-1324)

The nurse is caring for a client with thyroid disease who is experiencing a "racing heart," weight loss, exophthalmos, and heat intolerance. What additional actions should the nurse take? Select all that apply. 1. Evaluate if the client is receiving a beta-blocker. 2. Assess for hypotension. 3. Request increased calories with three balanced meals a day. 4. Apply lubricating eye drops throughout the day. 5. Place a circulating fan in the room.

1,4,5 Beta-blockers are used effectively for symptomatic relief of thyrotoxicosis. When the client feels palpitations or "heart racing," the nurse should evaluate for tachycardia and cardiac changes. The client with hyperthyroidism will experience hypertension, not hypotension. Because of the weight loss, a high-calorie diet of 4000 to 5000 cal/day is recommended with six full meals each day. Exophthalmos is the protrusion of the eyeballs from the orbits from fat deposits and fluid in the orbital tissues and ocular muscles from hyperthyroidism. When the eyelids do not close, corneal ulcers and loss of vision can occur. Eyedrops are helpful to prevent dryness. A fan in the client's room will provide comfort. (Lewis et al., 10 ed., pp. 1164-1166)

The nurse is caring for a client who began showing signs of diabetes insipidus 4 hours ago and was treated with IV fluids and one dose of nasal desmopressin (DDAVP). How will the nurse know the treatment is effective? Select all that apply. 1. Urine output will decrease. 2. Blood pressure will lower. 3. Glucose level will normalize. 4. Sodium level change from 128 mEq/L to 134 mEq/L. 5. Urine specific gravity of 1.029.

1,5 Desmopressin (DDAVP) alleviates polyuria by acting as antidiuretic hormone (ADH). In the case of diabetes insipidus, urine output will decrease and urine specific gravity should return to a normal level of 1.010 to 1.030. The blood pressure should increase because more water is being retained in the bloodstream, which will lower sodium levels. Glucose is not affected by diabetes insipidus. (Lewis et al., 10 ed., p. 1161)

A client is receiving IV antibiotic therapy. The order is for methicillin 750 mg IV. The nurse has a vial on hand that contains 1 gm. The instructions for reconstitution say to add 1.5 mL sterile water. Reconstituted solution will contain 500 mg methicillin per milliliter. How much will the nurse give? Answer: __________ mL

1.5 mL The dosage calculation cannot be made from the amount of solution added to the vial. The ratio of mg per mL after reconstitution is 500 mg/mL. (Potter & Perry, 9 ed., pp. 618-620)

A client has been diagnosed with cholecystitis. What menu selection would be appropriate for this client? 1. Eggs, bacon, whole grain toast, and decaffeinated tea 2. Fresh fruit, oatmeal, and decaffeinated coffee 3. Roast beef sandwich with Swiss cheese and cranberry juice 4. Cottage cheese, avocado, bagel, and tea

2 A low-fat diet is appropriate for the client with cholecystitis. Eggs, bacon, cheese, and avocados are high in fat and should be avoided. Other foods to avoid include whole milk, cream, butter, ice cream, fried foods, rich pastries, gravies, and nuts. (Lewis et al., 10 ed., p. 1008)

The nurse is caring for a newborn with an unrepaired meningocele. What is the highest priority goal for care? 1. Maintaining a patent airway 2. Preventing trauma to the sac 3. Providing nourishment to prepare for surgery 4. Encouraging long-term rehabilitation planning

2 A meningocele is a saclike cyst of meninges filled with spinal fluid that protrudes through a defect in the bony part of the spine. Trauma and/or infection could lead to permanent central nervous system damage. The other responses are important, but preventing trauma to the sac is the highest priority. (Hockenberry & Wilson, 10 ed., p. 1638)

Which position is best for the client who has undergone a traditional abdominal cholecystectomy? 1. Side-lying position, to prevent aspiration 2. Semi-Fowler's position, to facilitate breathing 3. Supine, to decrease strain on the incision line 4. Prone, to reduce nausea

2 A semi-Fowler's position improves lung expansion. The incision for a cholecystectomy is high and may interfere with respiratory exchange. The other positions would probably interfere with respirations. (Lewis et al., 10 ed., p. 1011)

The nurse is assessing a client after beginning external radiation. What is a nursing observation that confirms the presence of early side effects of the radiation? 1. A gradual weight loss and GI disturbances 2. Skin erythema followed by dry desquamation 3. Vertigo when sitting up quickly 4. Excoriation and blisters on the affected skin

2 Abnormal skin pigmentation, erythema, and dry desquamation may develop within a few days of beginning the radiation treatment. Wet desquamation may occur with progression of the radiation treatment, but the skin does not have blisters. Vertigo may be a sign of orthostatic hypotension associated with hypovolemia. The weight loss occurs, but it is not due to the radiation; it is most often due to the malignancy. (Lewis et al., 10 ed., pp. 250-256)

The nurse is caring for an infant who is experiencing respiratory distress and being treated with continuous positive airway pressure (CPAP). The nurse knows that for this treatment to be most effective, the infant must be: 1. Intubated with respiration maintained by controlled ventilation 2. Able to breathe spontaneously 3. Frequently stimulated to maintain respiratory rate 4. Suctioned frequently to maintain alveolar ventilation

2 CPAP only works when the infant is breathing on his own. When the airway is opened for a breath, the CPAP increases the pressure in the airway, which increases airflow to the lungs and oxygenation. CPAP is not used when a child requires controlled ventilation. Stimulating the infant may be appropriate, but the child must be able to breathe spontaneously for this to be effective. The child is not suctioned unless an excessive amount of mucus must be removed. (Hockenberry & Wilson, 10 ed., pp. 373-374)

The nurse received handoff for a client returning from a right leg amputation. What should be included in the plan of care? 1. Applying ice packs to the residual limb for 72 hours 2. Having the client lie on his or her abdomen for 30 minutes three or four times a day 3. Wrapping the residual limb with elastic bandages from proximal to distal ends 4. Managing client's pain with antiinflammatory medications

2 Client should lie on the abdomen for 30 minutes three or four times a day and position the hip in extension while prone. Also, to prevent flexion contractures, clients should avoid sitting in a chair for more than an hour. The residual limb is wrapped from distal to proximal. Ice packs are not used on the residual limb after surgery because the cold restricts blood flow. Antiinflammatory medications may be used for pain relief but not to prevent edema. (Lewis et al., 10 ed., p. 1488)

The nurse is conducting discharge dietary teaching for a client with diverticulosis who is recovering from an acute episode of diverticulitis. Which statement by the client would indicate to the nurse that the client understood his dietary teaching? 1. "I will need to increase my intake of protein and complex carbohydrates to increase healing." 2. "I need to progress my diet from liquids to soft, low-fiber foods until the diverticulitis is completely resolved." 3. "I will not put any added salt on my food, and I will decrease intake of foods that are high in saturated fat." 4. "Milk and milk products can cause a lactose intolerance. If this occurs, I need to decrease my intake of these products."

2 Constipation increases problems with diverticula. Upon discharge, the client should continue fluids, progressing to soft foods that are low in fiber. A diet high in fiber is recommended once the acute diverticulitis is resolved completely. The other options do not have any specific relevance to diverticula disease. (Lewis et al., 10 ed., p. 964)

A client is scheduled for an abdominal hysterectomy. Preoperative teaching includes which of the following? 1. A nasogastric tube will be left in to control vomiting after surgery. 2. A douche and an enema may be done the evening before surgery. 3. There will be a moderate amount of bloody vaginal drainage after surgery. 4. Ambulation will be delayed for 48 hours because of the extensive nature of the procedure.

2 Douching and some method of bowel preparation are part of the preoperative measure to cleanse the field of bacteria and pathogens. Vomiting is not a usual postoperative problem. If this occurs, it would most likely be controlled by an antiemetic. Vaginal drainage occurs with a vaginal, not an abdominal, hysterectomy. Ambulation is encouraged to prevent venous stasis. (Lewis et al., 10 ed., p. 1257)

The nurse is administering medications to a client who has no allergy band on his arm. The nurse tells the client she has his penicillin medication. The client states that the last time he had penicillin, it made his mouth tingle and his hands itch. What would the best nursing action be? 1. Administer the medication because there is no indication that the client is allergic to penicillin. 2. Hold the medication and contact the physician regarding the client's statement about his previous experience with penicillin. 3. Hold the medication and review the client's chart to determine whether there is a penicillin allergy noted. 4. Notify the nursing supervisor regarding the client's statement and request further evaluation of the client.

2 Even though there is no allergy band, if the client provides information that may indicate a previous reaction to a medication (especially penicillin), hold the medication. With each exposure to an allergen, the reaction could become worse (type I reaction). Notifying the supervisor is not necessary; there is enough information for the nurse to make this decision. (Lewis, 10 ed., pp. 201-202)

The newborn's mother is concerned about the shape of the baby's head after delivery. She states that the baby looks like a "cone head." What is the most appropriate response by the nurse? 1. "You don't need to worry about it. It is perfectly normal after birth." 2. "It is molding caused by the pressure during birth and will disappear in a few days." 3. "I will report it to the physician and recommend a diagnostic scan." 4. "It is a collection of blood related to the trauma of delivery and will absorb in a few weeks."

2 Explaining that the "cone-head" appearance is molding caused by pressure during birth and that it will disappear in a few days provides the most appropriate response. The nurse reassures the mother that molding is normal and caused by the pressure during delivery and will disappear in 1 to 2 days. The nurse should not tell a concerned mother not to worry, which is a type of false reassurance. The condition does not require a diagnostic scan. The collection of blood related to trauma that will take several weeks to absorb describes cephalhematoma. (Hockenberry & Wilson, 10 ed., p. 253)

A client has been diagnosed with disseminated intravascular coagulopathy (DIC). The nurse will anticipate administering which of the following fluids? 1. Packed red blood cells (PRBCs) 2. Fresh frozen plasma (FFP) 3. Volume expanders, such as D10W 4. Whole blood

2 Fresh frozen plasma contains all coagulation factors including V and VIII. DIC results when the body can no longer create clotting factors; thus, fresh frozen plasma is the best answer. Packed red blood cells will be used to increase oxygenation, but this is second-line treatment in this situation. Whole blood is used less frequently, but it does not provide adequate clotting factors. Volume expanders will not help increase clotting factors. (Lewis et al., 10 ed., pp. 630-631)

A 7-year-old boy is in the emergency department with a greenstick fracture of the ulna. How will the nurse explain the fracture to the parents? 1. The bone is broken across the growth plate. 2. There is a splintering of the bone on one side. 3. There is a separation of the bone at the fracture site. 4. The bone is broken into several fragments.

2 Greenstick fracture refers to splintering of the bone, not a complete fracture. The name comes from the splintering effect in attempts to break a "green stick." It is a common fracture in children. A comminuted fracture (bone is broken into several fragments) has multiple bone fragments and is more common in adults. In a nondisplaced fracture, such as the greenstick fracture, the periosteum is intact across the fracture, and the bone is still in alignment. (Lewis et al., 10 ed., p. 1468)

Herpes zoster has been diagnosed in an older adult client. What will the nursing management include? 1. Apply antifungal cream to the areas daily. 2. Maintain client on contact precautions. 3. Instruct on the need for sexual abstinence. 4. Closely inspect the perineal area for lesions.

2 Herpes zoster is considered infectious and contact precautions should be used with an older adult client. Antiviral medications would be given instead of antifungal agents. Lesions are usually along the sensory dermatomes (waist, neck, face) and not in the perineal area, which is HSV-2. There is no need for sexual abstinence, although a condom should be worn if contact may occur with the lesions. (Lewis et al., 10 ed., p. 415)

A client's Pap test reveals epithelial cells characteristic of adenocarcinoma of the cervix. The nurse understands that which of the following is a major risk factor for cervical cancer? 1. Long-term use of oral contraceptives 2. Recurrent outbreaks of human papilloma virus (HPV) 3. Grand multiparity with history of preterm labor 4. Alternative therapy for treatment of menopausal symptoms

2 Increased risk for cervical cancer is associated with a history of HPV (genital warts) infection, STDs, HSV-2, multiple sex partners, first intercourse at early age, and abnormal Pap tests. The use of alternative therapy and estrogen therapy for menopause, oral contraceptive use, and multiple pregnancies do not increase the risk. (Lewis et al., 10 ed., p. 1234)

A client begins receiving methotrexate for severe symptoms of rheumatoid arthritis. What is the most important information for the nurse to give this client regarding the medication? 1. Take extra fiber and fluids to counteract the constipating effect. 2. It is important to have periodic laboratory work done. 3. Take the drug on an empty stomach. 4. Hirsutism and menstrual changes sometimes develop as side effects.

2 Laboratory work will need to be done periodically during administration to monitor for the development of anemia, leukopenia, thrombocytopenia, and/or hepatic toxicity. Hirsutism and menstrual changes occur with long-term corticosteroid use. Methotrexate should be given 1 hour before or 2 hours after meals to prevent vomiting when given by mouth (PO). Antiemetics are given concurrently with the medication. (Lewis et al., 10 ed., p. 1525)

A client is receiving magnesium sulfate to help suppress preterm labor. The nurse should watch for which sign of magnesium toxicity? 1. Headache 2. Loss of deep tendon reflexes 3. Palpitations 4. Dyspepsia

2 Magnesium toxicity causes central nervous system depression; this would be observed as loss of deep tendon reflexes, paralysis, respiratory depression, drowsiness, lethargy, blurred vision, slurred speech, and confusion. Headache may be an adverse effect of calcium channel blockers, which are sometimes used to treat preterm labor. Palpitations are an adverse effect of terbutaline, which is also used to treat preterm labor. Dyspepsia may occur as an adverse effect of indomethacin, a prostaglandin synthetase inhibitor used to suppress preterm labor. (Lowdermilk et al., 11 ed., p. 664)

What is the type of skin cancer that is most difficult to treat? 1. Dysplastic nevi 2. Malignant melanoma 3. Basal cell epithelioma 4. Squamous cell epithelioma

2 Malignant melanoma is the most difficult to treat; it involves extensive full-thickness skin resections and has the poorest prognosis. Dysplastic nevi are thought to be a precursor of malignant melanoma, although they are not considered malignant in the initial stage. Basal cell epithelioma and squamous cell epithelioma are easier to treat and do not metastasize as does melanoma. (Lewis et al., 10 ed., pp. 411-413)

The nurse applies a nitroglycerin patch on a client who has undergone cardiac surgery. What nursing observation indicates that a nitroglycerin patch is achieving the desired effect? 1. Chest pain is completely relieved. 2. Client performs activities of daily living without chest pain. 3. Pain is controlled with frequent changes of patch. 4. Client tolerates increased activity without pain.

2 Nitroglycerin is used to prevent angina so that the client can perform the normal activities of daily living without chest pain. Sublingual nitroglycerin or translingual spray is used to treat immediate-onset chest pain. (Lehne, 9 ed., p. 591)

The doctor has indicated that ampicillin and gentamicin are to be given piggyback in the same hour, every 6 hours (12-6-12-6). How would the nurse administer these drugs? 1. Combine the drugs into 100 mL NS and administer. 2. Give each drug separately, flushing between drugs. 3. Retrograde both drugs into the tubing. 4. Give one drug every 4 hours and the other every 6 hours.

2 Only one antibiotic should be administered at a time; therefore if the medications are given during the same hour, the IV tubing will need to be flushed between administrations. Both drugs should be administered at the time ordered. (Potter & Perry, 9 ed., pp. 667-670)

Which client is at highest risk for retinal detachment? 1. A 4-year-old with amblyopia 2. A 17-year-old who plays physical contact sports 3. A 33-year-old with severe ptosis and diplopia 4. A 72-year-old with nystagmus and Bell palsy

2 Participating in physical contact sports puts this person at the highest risk for retinal detachment because trauma is a leading cause. The other pathologies (amblyopia, ptosis, diplopia, nystagmus, and Bell palsy) will affect eye function but have minimal likelihood of causing retinal detachment. (Lewis et al., 10 ed., p. 376)

A teacher notifies the school nurse that many of the students in her third-grade class have been scratching their heads and complaining of intense itching of the scalp. The nurse notices tiny white material at the base of a student's hair shaft. What condition does this assessment reflect? 1. Tinea capitis 2. Pediculosis capitis 3. Dandruff 4. Scabies

2 Pediculosis capitis (head lice) is characterized by tiny white nits (eggs) that attach to the base of the hair shaft and are highly contagious. Tinea capitis is characterized by a red, scaly, rash with central clearing in the well-defined margins. Dandruff is often mistaken for head lice, but dandruff can be easily removed from the hair shaft. Nits adhere to the hair shaft and are not easy to remove. Scabies form burrows under the skin and cause intense nighttime itching. (Lewis et al., 10 ed., p. 416)

A client has a diagnosis of right-sided empyema. Thoracentesis is to be performed in the client's room. The nurse will place the client in what position for this procedure? 1. Prone position with feet elevated 2. Sitting with upper torso over bedside table 3. Lying on left side with right knee bent 4. Semi-Fowler's position with lower torso flat

2 Positioning over the bedside table allows the ribs to separate, which assists the physician in positioning the needle into the pleural cavity. If the client is unable to assume a sitting position, he or she is placed on the affected side with head of bed slightly elevated. The area containing the fluid should be dependent. (Ignatavicius & Workman, 8 ed., p. 511 )

A client with diabetes receives a combination of regular and NPH insulin at 0700 hours. At what point in the day should the client be educated about peak incidence of hypoglycemia? 1. 12 p.m. to 1 p.m. (1200-1300 hours). 2. 9 a.m. and 5 p.m. (0900 and 1700 hours). 3. 10 a.m. and 10 p.m. (1000 and 2200 hours). 4. 8 a.m. and 11 a.m. (0800 and 1100 hours).

2 Regular insulin (a short-acting insulin) peaks in 2 to 3 hours, and NPH (an intermediate-acting insulin) peaks in 4 to 10 hours. Hypoglycemia would most likely occur between 9 a.m. and 5 p.m. (0900 and 1700 hours). (Lewis et al., 10 ed., p. 1126)

The nurse is providing discharge instructions to a client who has had a splenectomy. The teaching is based on the knowledge that splenectomy clients have: 1. Decreased leukocytes 2. Increased platelets 3. Decreased hemoglobin 4. Increased eosinophils

2 Removing the spleen will lead to an increase in peripheral RBC, WBC, and platelet counts. In addition, after splenectomy, immunologic deficiencies may develop (IgM); however, IgG and IgA remain normal. There is always a lifelong risk for infection after a splenectomy. (Lewis et al., 10 ed., p. 647)

The nurse is assessing a primigravida client who is at 26 weeks' gestation; the client's blood type is AB negative, her serology is negative, and she has a history of one miscarriage at 20 weeks. On the basis of this information, what will the nurse anticipate being ordered for this client? 1. An amniocentesis at 30 weeks' gestation 2. Administration of Rho(D) immune globulin at 28 weeks' gestation 3. A blood test on the father to determine his blood type 4. Fetal blood sampling to determine fetal blood type

2 Rho(D) immune globin is given to all Rh-negative women in the 28th week of gestation. The blood type of the father may be determined, but paternity is not an important issue in this situation. An amniocentesis is not necessary at this time, and obtaining fetal blood to determine blood type is more risky than administering the medication to the mother. (Lowdermilk et al., 11 ed., p. 494)

The nurse is caring for a client who is experiencing a severe anaphylactic reaction caused by an allergy to peanuts. After administering subcutaneous epinephrine and beginning oxygen administration, what would be the next most important nursing action? 1. Administer analgesics to relieve the pain. 2. Start an IV for fluid administration. 3. Insert a catheter to determine urinary output. 4. Obtain a history of possible reactions to penicillin.

2 Shock is a common problem in anaphylactic reactions; therefore it is important to establish an IV for fluid and medication administration. There should be no pain, and there is no reason the client cannot void on his or her own. A history can be taken at a later time. (Lewis et al., 10 ed., p. 202)

The nurse is assisting a client immediately before a colonoscopy. The nurse will direct the client and help him move into what position? 1. Prone 2. Sims' lateral 3. Slight Trendelenburg 4. Flat with lithotomy stirrups

2 Sims' lateral position is most commonly used for best access and visualization during the procedure and for the client's comfort. Lithotomy position with stirrups is used for gynecologic examinations and prostate surgery. (Ignatavicius & Workman, 8 ed., p. 1095)

A nurse is caring for a client who is receiving a blood transfusion. The transfusion was started 30 minutes ago at a rate of 100 mL/hr. The client begins to complain of low back pain and headache and is increasingly restless. What is the first nursing action? 1. Slow the infusion and evaluate the vital signs and the client's history of transfusion reactions. 2. Stop the transfusion, disconnect the blood tubing, and begin a primary infusion of normal saline solution. 3. Stop the infusion of blood and begin infusion of normal saline solution from the Y connector. 4. Recheck the unit of blood for correct identification numbers and cross-match information.

2 Stop the blood infusion and disconnect the line to decrease the further infusion of red blood cells. Begin administration of normal saline solution with new tubing. The remainder of the blood should be returned to the blood bank for evaluation regarding the reaction. (Lewis et al., 10 ed., pp. 650-652)

The nurse is caring for a client who is doing well after a craniotomy. What will the bowel care for this client include? 1. An enema every other day to avoid the Valsalva maneuver 2. High-fiber diet and stool softeners to prevent constipation 3. Low-residue diet to decrease stool formation and prevent constipation 4. Daily checking for impaction caused by loss of bowel innervations

2 Straining at defecation or the use of the Valsalva maneuver may exacerbate increased ICP. The nurse promotes normal bowel movements that prevent straining by encouraging a high-fiber diet and stool softeners when needed. Enemas are discouraged, and it is important to prevent constipation so that an impaction does not occur. (Lewis et al., 9 ed., p. 1368)

The nurse understands clamping a chest tube may cause what problem? 1. Atelectasis 2. Tension pneumothorax 3. Bacterial infections in the pleural cavity 4. Decrease in the rate and depth of respirations

2 Tension pneumothorax occurs when air enters the pleural space with each inspiration, becomes trapped there, and is not expelled during expiration (i.e., one-way valve effect). Pressure builds in the chest as the accumulation of air in the pleural space increases. This can lead to a mediastinal shift. Atelectasis occurs when the atmospheric pressure enters the pleural cavity. This procedure has nothing to do with an infection or pulmonary consolidation. (Ignatavicius & Workman, 8 ed., pp. 623-624)

To evaluate the progress of the client's systemic lupus erythematosus(SLE), the nurse evaluates which data? 1. Increased serum complement fixation, which correlates with reduction of "butterfly" rash 2. Increasing levels of C-reactive protein (CRP) and erythrocyte sedimentation rate (ESR) 3. Overall bone marrow proliferation, which correlates with symptoms of inflammation 4. Presence of antinuclear antibodies (ANA), which correlates with a diminishing immune process

2 The ESR and the CRP are indicators of inflammation in the body. Neither is diagnostic of SLE, but the level of inflammation is an index to the progress of the condition. Presence of ANA is characteristic of SLE, but it does not indicate progression. Complement fixation does not indicate progression, nor does absence or presence of the butterfly rash. (Lewis et al., 10 ed., p. 1540)

What should the nurse take into consideration when giving medication to an older adult client? 1. The serum albumin level of an older adult is lower, thus decreasing drug metabolism. 2. The older adult client metabolizes and excretes at a decreased rate. 3. Medication affects the older adult client during the early hours of the morning. 4. Medication has an increased effect on the respiratory system of the older adult client.

2 The ability to metabolize and excrete medication decreases with the aging process. For example, at the age of 80 years, the ability to metabolize medications decreases approximately 50% compared with a 30-year-old. Medications do not have an increased effect in the morning, nor do they specifically affect the respiratory system of the older adult. (Burchum & Rosenthal, 9 ed., pp. 92-93)

A multigravida client comes into the emergency department complaining of abdominal pain. She is at 30 weeks' gestation. On assessment, the nurse observes complete dilation and effacement of the cervix with the perineal area bulging and the infant's head crowning. The mother states she is feeling a strong urge to push. What is the best nursing action? 1. Prepare the client for an emergency cesarean delivery. 2. Place even gentle pressure on infant's head and support it through the birth canal. 3. Have the client hold her legs together and take her to the labor and delivery unit. 4. Have the client take two deep breaths and push hard with the next contraction.

2 The birth of this infant is imminent. The infant is probably small because of prematurity, the mother is multigravida, and the mother has not received any analgesics. The nurse should place hands at the perineum to apply light pressure to the fetal head to prevent rapid expulsion. Do not attempt to take the mother to labor and delivery; notify labor and delivery and newborn nursery units of the impending birth of premature infant. If the mother pushes hard with the next contraction, the infant may progress too rapidly. There is no need for a cesarean delivery; no attempt should be made to slow or prevent the delivery because this would put the infant in further jeopardy. (Lowdermilk et al., 11 ed., p. 462)

A client is going to begin external radiation therapy for his lung cancer. Which comment by the client would indicate to the nurse the need for additional teaching? 1. "I will shower with a mild soap and check my skin for areas of redness." 2. "I am looking forward to swimming laps again for my exercise." 3. "I am going to eat small meals and increase the protein and fiber in my diet." 4. "I will use only unscented emollient creams to the dry skin areas on my chest."

2 The client should avoid swimming during the treatment period because of the irritating chemicals in swimming pools. The other options—showering with a mild soap, eating small nutritious meals, and using an emollient cream—are correct for external radiation. (Lewis et al., 10 ed., p. 255)

The nurse is discussing testicular self-examination with a male client. What information is important for the nurse to include in the discussion? 1. The best time to perform the examination is 24 hours after sexual intercourse. 2. The examination should be conducted at the same time each month. 3. The client should perform this self-examination every 3 to 4 months. 4. When the scrotum is pulled up tight against the body, the testes are easier to palpate.

2 The examination should be done at the same time each month to develop a regular routine. After a shower, when the scrotum is warm and the testicles are descended away from the body, is a good time to perform the examination. (Lewis et al., 10 ed., p. 1205)

A client's eye has been anesthetized for an ophthalmology examination. What instructions will be important for the nurse to give the client? 1. Do not watch television for at least 24 hours. 2. Do not rub the eye for 15 to 20 minutes. 3. Irrigate the eye every hour to prevent dryness. 4. Wear sunglasses when in direct sunlight for the next 6 hours.

2 The eye has been anesthetized, therefore there is no feeling or sensation in it for 15 to 20 minutes. It would be very easy to rub the eye and cause damage. Not watching television for a day would have no effect on the safety. Irrigating the eye every hour is not necessary. Because there is no effect on the client's tolerance of direct sunlight, sunglasses would be optional. (Lilley, 8 ed., p. 916)

At 9:00 a.m. a 24-hour (composite) urine collection is started. What instructions will the nurse provide to the client? 1. Place the first voided specimen in the container and continue to collect the urine until 9:00 a.m. the following day. 2. Discard the first morning specimen, collect urine for the next 24 hours, and make sure to void before the collection is completed at 9:00 a.m. the following day. 3. Discard the first morning specimen because it may contain concentrated abnormal components. 4. Collect all urine from 9:00 a.m. onward in separate containers that are labeled for time and amount of voiding.

2 The first specimen is discarded before the collection is started. Collection will continue until the completed time frame; the client should void before the collection is completed. (Lewis et al., 10 ed., p. 1024)

The nurse is caring for a client with leukemia who is experiencing bleeding into the knee joints. What is the best nursing care for this client regarding joint mobility and activity? 1. Encourage short walks around the room every 2 hours. 2. Keep the joint immobilized and maintain bed rest for the client. 3. Gently put the legs through passive range of motion every 4 hours. 4. Keep the legs wrapped with elastic bandages and immobilized in splints.

2 The knee joint should remain immobilized during the active bleeding phase; walking and passive range of motion may increase the bleeding. The legs are not wrapped, but compression dressings may be placed on the knee. (Lewis et al., 10 ed., p. 628)

The nurse is responsible for documenting the first meconium stool the newborn passes. If the newborn does not have a stool in the first 24 hours of life, the nurse should first: 1. Insert a rectal thermometer to facilitate the process. 2. Inspect the anal area for an opening. 3. Monitor the vital signs for a rise in temperature. 4. Increase oral feeding to stimulate passage of stool.

2 The lack of passage of a meconium stool requires further assessment; it may be a sign of imperforate anus. The first assessment the nurse should perform is to visually inspect the anal area for an opening. Inserting a rectal thermometer could tear the anal mucosa, and if an imperforate anus is present, all oral feedings will be stopped. (Hockenberry & Wilson, 10 ed., p. 253)

Which statement best describes the problem of regulation of body temperature in a 3-lb (1361-g) premature infant? 1. The surface area of the premature infant is relatively smaller than that of a healthy term infant. 2. There is a lack of subcutaneous fat, which furnishes insulation. 3. There are frequent episodes of diaphoresis, causing loss of body heat. 4. There is a limited ability to produce body proteins.

2 The premature infant's temperature-regulating mechanism is poorly developed at birth. Heat production is low, and heat loss is high because of the greater body surface area relative to weight and the infant's lack of subcutaneous fat. No diaphoresis should be occurring, and the question has nothing to do with protein metabolism. The statement "The surface area of the premature infant is relatively smaller than that of a healthy term infant" describes the body surface area of the infant, and although true, it does not have any implication for maintaining emperature. (Hockenberry & Wilson, 10 ed., p. 341)

The nurse is administering propranolol to a client who is being treated for hypertension. What is the desired response to this medication? 1. Vasodilation occurs, resulting in a decrease in the cardiac afterload. 2. The cardiac rate is decreased, with a resulting decrease in the cardiac output. 3. Cardiac output is decreased, and the arterial BP rises. 4. Pericardial fluid is decreased, thus decreasing the cardiac workload.

2 The primary action of the beta-blocker, propranolol, is to slow the cardiac rate. The medication is effective in the treatment of hypertension or dysrhythmias that result in tachycardia. With a decrease in cardiac rate, there is also a decrease in cardiac output. The betablockers do not cause vasodilation. A decrease in cardiac output would cause a decrease in arterial BP, not an increase. Beta-blockers do not have an effect on pericardial fluid. (Lewis et al., 10th ed., pp. 691-692)

A client has extensive burns with eschar on the anterior trunk. What is the nurse's primary concern regarding eschar formation? 1. It prevents fluid remobilization in the first 48 hours after burn trauma. 2. Infection is difficult to assess before the eschar sloughs. 3. It restricts the ability of the client to move about. 4. Circulation to the extremities is diminished because of edema formation.

2 The primary concern would be watching for infection, because the eschar makes it difficult to visually examine the healing skin. Removal of the eschar enhances healing and prevents infection, which occurs because of the moist, enclosed area under the eschar. Eschar formation will not prevent fluid remobilization. It might restrict mobility if the eschar were involving the arm, legs, or joint areas. Circulation to the extremities would not be affected by eschar on the anterior trunk; that would be the case if the eschar were on the extremities. (Lewis et al., 10 ed., p. 443)

After a tonic-clonic (formerly grand mal) seizure, what nursing action is the highest priority? 1. Loosen or remove constricting clothing and protect client from injuring himself or herself. 2. Maintain a patent airway by turning the client on his side and suctioning, if necessary. 3. Remain with the client and administer anticonvulsant medications as ordered by the physician. 4. Describe and record events before the onset of the seizure, during the seizure, and after the seizure.

2 The priority after a grand mal seizure is to maintain a patent airway. The question is asking for a nursing intervention after the seizure is over. The clothes should be loosened so that they do not constrict the client. The nurse may need to remain with the client, and the events of the seizure need to be recorded, but the priority of this question is the airway. (Lewis et al., 9 ed., p. 1426)

The nursing assessment of an infant reveals expiratory grunting, substernal retractions, and a temperature of 99°F (37.2°C). What is the first nursing action? 1. Place the infant in Trendelenburg position. 2. Begin administration of 40% humidified oxygen via hood. 3. Increase the temperature of the environment. 4. Perform a complete assessment for congenital anomalies.

2 The priority here is the respiratory distress. The first nursing action is to increase the inspired oxygen; it would be appropriate to determine whether the infant has any mucus in the airway. Do a quick but thorough assessment of the infant and advise the supervisor or doctor regarding the infant's status. Trendelenburg position would be contraindicated because it would make breathing more difficult. Although performing an assessment is important, respiratory distress is a priority. Increasing the temperature of the neonate's environment could lead to an increased pulse rate and may lead to more oxygen consumption as an attempt of the body to cool itself. (Hockenberry & Wilson, 10 ed., p. 373)

The nurse is assessing a client who has just had a lumbar puncture. What nursing observation would cause the nurse the most concern? 1. Client tells the nurse he has a headache. 2. Nurse observes clear fluid oozing from the puncture site. 3. Client states he has less strength in his arms. 4. Client has difficulty voiding from supine position.

2 The spinal needle is inserted at L3-L4. If there is any oozing after the procedure, it could be spinal fluid. This would increase the risk of headache and infection. Headache is not uncommon. Remaining in the supine position should help prevent the headache. Weakness of the upper muscles is not relevant to the lumbar puncture, and many clients have difficulty voiding while confined to bed. (Lewis et al., 9 ed., pp. 1350-1351)

Which client is most likely to have iron deficiency anemia? 1. A client with cancer receiving radiation therapy twice a week 2. A toddler whose primary nutritional intake is milk 3. A client with a peptic ulcer who had surgery 6 weeks ago 4. A 15-year-old client in sickle cell crisis

2 The toddler will need to eat a balanced diet and may require an iron supplement. A diet based primarily on milk products will not cove the iron needs of a toddler. A client in sickle cell crisis may experience anemia, but it is due to the increased destruction of red blood cells, not poor iron intake. The client receiving radiation therapy may also develop anemia; however, it is not due to poor nutritional intake but to bone marrow suppression. The client who has had gastric surgery may develop anemia as a result of a lack of adequate utilization of vitamin B12. (Lewis et al., 10 ed., p. 610)

A client is found to be comatose with a blood glucose level of 50 mg/dL (2.8 mmol/L). What action should the nurse implement first? 1. Infuse 1000 mL of D5W over a 12-hour period. 2. Administer 50% glucose intravenously. 3. Check the client's urine for the presence of sugar and acetone. 4. Encourage the client to drink orange juice with added sugar.

2 The unconscious, hypoglycemic client needs immediate treatment with 50% intravenous glucose (highly concentrated). Administering 1000 mL of D5W over 12 hours does not provide enough glucose to treat the problem. Trying to give oral fluids to an unconscious client should never be done because it increases the risk for aspiration. Urine sugar does not need to be evaluated if the serum blood glucose is available. (Lewis et al., 10 ed., p. 1180)

The nurse encourages a client in labor to assume a side-lying position. What is the purpose of this position? 1. Prevents prolapse of the cord 2. Enhances venous return 3. Relaxes the pelvic musculature 4. Promotes crowning

2 The weight and pressure of the uterus on the vena cava decreases the venous return to the mother's heart. This will precipitate a drop in blood pressure and decreased blood supply to the fetus. In a side-lying position, pressure is reduced and adequate cardiac output is promoted. (Lowdermilk et al., 11 ed., p. 440)

A client returns to the clinic to receive evaluation of his routine purified protein derivative (PPD) test for tuberculosis screening. The test result is positive. What is the best nursing interpretation of this information? 1. This is a serious type II reaction and could indicate that he has active tuberculosis; he will need further evaluation immediately. 2. The positive results indicate the client has been exposed to the tuberculosis bacilli and has had a delayed type IV response. 3. The client's immune system has been compromised, which allows the immune system to build up antibodies against the pathogen. 4. An autoimmune response has occurred, and the client will need further evaluation to determine appropriate treatment.

2 Type IV (cell-mediated, delayed hypersensitivity) reaction is a delayed response that occurs 24 to 72 hours after exposure to the allergen (e.g., PPD). The client has been sensitized to tuberculosis. There is no indication of active TB, and it is not a type II reaction. The client's immune system has not been compromised; rather, it has responded normally with the production of antibodies after exposure to the allergen. This does not represent an autoimmune response. (Lewis et al., 10 ed., pp. 199-200)

A client has developed stomatitis while receiving chemotherapy. What would be an appropriate intervention to suggest for the pain associated with the stomatitis? 1. Use lemon-flavored glycerin swabs. 2. Apply antacid coating solutions and viscous lidocaine. 3. Brush oral plaques off with a soft toothbrush. 4. Have client swish mouth with a weak hydrogen peroxide solution.

2 Ulcerations in the mouth (stomatitis) can occur when a client is receiving chemotherapy medication. Alleviation of the pain can be achieved by administering systemic or local analgesics and coating agents such as antacids. Frequent saline rinses are encouraged. Lemon-glycerin swabs may irritate the mucosa and lead to further pain, as would trying to remove oral plaques. (Lewis et al., 10 ed., pp. 251-254)

A client in kidney failure is to have a serum blood urea nitrogen level determined. What will this diagnostic test measure? 1. Concentration of the urine osmolarity and electrolytes 2. Serum level of the end products of protein metabolism 3. Ability of the kidneys to concentrate urine 4. Levels of C-reactive protein to determine inflammation

2 Urea is an end product of protein metabolism. In kidney failure, the kidneys cannot clear all of the urea from the blood, and the creatinine and BUN level will be elevated. The C-reactive protein is a diagnostic test used in assessing clients with inflammatory bowel disease, rheumatoid arthritis, autoimmune diseases, and pelvic inflammatory disease (PID). A specific gravity test of the urine would assess the ability of the kidneys to concentrate urine. The urine osmolarity (concentration of particles in urine) and electrolytes assess fluid balance. The kidneys play an important role in the balance of electrolytes and fluids. (Lewis et al., 10 ed., p. 1024)

What is the priority assessment finding for a client who has sustained burns on the face and neck? 1. Spreading, large, clear vesicles 2. Increased hoarseness 3. Difficulty with vision 4. Increased thirst

2 When there is evidence of burns around the face, the airway should be carefully assessed. Increased respiratory rate and hoarseness may be the first sign of respiratory complications. Large, clear vesicles are expected on burns of second degree or worse and are not a sign of a complication. Difficulty with vision may be of concern, but it is not life threatening like respiratory distress. Increased thirst is common in the first few hours after a burn because of fluid shift into the extravascular space. Intubation is often needed within 1 to 2 hours of the injury in burns of the face and neck. (Lewis et al., 10 ed., p. 438)

The nurse is preparing a teaching plan for a family with a child who has been diagnosed with sickle cell anemia and crisis. What will the nurse include in the teaching regarding the pathophysiology of sickle cell crisis? 1. It results from altered metabolism and dehydration. 2. Tissue hypoxia and vascular occlusion cause the primary problems. 3. Increased bilirubin levels will cause hypertension. 4. There are decreased clotting factors with an increase in white blood cells.

2 When there is inadequate hydration, the sickled cells begin to clump together, which leads to vascular occlusion. Tissue hypoxia occurs as a result of the decreased oxygen-carrying capacity of the sickled red blood cells and from the vascular occlusion. Increased bilirubin from the hemolysis of the RBC does not cause hypertension, but jaundice and an increased incidence of cholelithiasis may occur. With sickle cell anemia, there is not a decrease in clotting factors. (Lewis et al., 10 ed., pp. 616-617)

The nurse is assigned to care for a newly admitted client with acute pancreatitis. Admitting assessment includes midepigastric pain of an 8 out of 10, low-grade fever, and elevated amylase and lipase levels with hypocalcemia and hyperglycemia. What should be the nurse's priority action? 1. Deliver proton pump inhibitor. 2. Place nasogastric (NG) tube. 3. Administer IV calcium gluconate. 4. Administer oral analgesic.

2 With the management of acute pancreatitis, the pain is better controlled when the client is NPO and an NG tube with gastric suction is initiated. Pain medicine should be IV because patient is NPO. Although a PPT and IV calcium may be used, they are not the priority. (Lewis et al., 10 ed., p. 1001)

The parents of a client with hemophilia are taking their child home. Which statement indicates a need for further education regarding hemophilia? 1. "We should ensure that our child has regular dental appointments." 2. "We need to wrap our child's limbs daily to prevent bleeding." 3. "We should help our child select activities that minimize the risk of injury." 4. "We should not give our child aspirin."

2 Wrapping limbs does not prevent bleeding and does not stop bleeding during acute episodes. Regular dental appointments are necessary to prevent dental problems in a child with hemophilia. Children with hemophilia should be encouraged to participate in activities that are not contact sports and have minimal risk of injury. Aspirin, NSAIDs, and other blood thinners should be avoided. (Lewis et al., 10 ed., p. 628)

The nurse is teaching a client about home care and treatment of venous stasis ulcers on his leg. What should be included in the nurse's instructions? Select all that apply: 1. Dressings do not need to be changed frequently because there is minimal drainage. 2. Healing will be facilitated by wearing leg compression devices. 3. When the client is in the sitting position, he should keep his legs elevated. 4. Avoid standing for prolonged periods of time. 5. Cool packs can be applied to the ulcers to decrease inflammation. 6. Soak the affected extremity in warm water every evening.

2, 3, 4 Healing of venous stasis ulcers is dependent on relieving the venous congestion in the extremity. Compression devices and elevation of the extremity are the most effective methods. The client should avoid standing for long periods because this increases venous stasis. Moist cool and/or warm packs are not used, but moist environment dressings are utilized. Dressings need to be changed as frequently as necessary because there may be excessive drainage. (Ignatavicius & Workman, 8th ed., pp. 734-735)

Which are signs and symptoms of cystitis. Select all that apply. 1. Increased bladder capacity 2. Frequency 3. Dysuria 4. Nocturia 5. Urgency 6. Polydipsia

2, 3, 4, 5 Classic signs of cystitis include frequency, urgency, decreased bladder capacity, nocturia, and dysuria caused by the inflammatory process. Polydipsia (excessive thirst) is associated with diabetes mellitus. (Lewis et al., 10 ed., p. 1034)

The nurse is caring for a client who begins to exhibit the cardiac rhythm shown in the illustration below. As the nurse observes, the rhythm remains the same. What is the best nursing actions? Select all that apply. 1. Call an emergency code and begin resuscitation. 2. Assess apical pulse, comparing to distal pulse. 3. Apply oxygen. 4. Assess for chest pain or shortness of breath. 5. Have the client cough. 6. Assess the blood pressure.

2, 3, 4, 6 The rhythm is premature ventricular beats or contractions (PVCs). Treatment is related to the cause, which may be hypoxia or from electrolyte replacement. Assessment of the client's hemodynamic status is important to determine whether treatment with drug therapy is needed. Drug therapy includes beta-blockers, procainamide, or amiodarone. (Lewis et al., 10 ed., p. 769)

The nurse is preparing discharge teaching for a client with aplastic anemia. What will be important to include in the teaching plan? Select all that apply. 1. Take your iron with meals every day and decrease the amount of green, leafy vegetables in your diet. 2. Establish a balance between rest and activity; avoid excessive fatigue. 3. Rest and supplemental oxygen may be required during periods of dyspnea. 4. Drink a glass of wine in the evening to help increase your appetite. 5. Notify your health care provider if you begin to experience frequent bruising. 6. Increase your intake of dairy products (milk and cheese) and protein.

2, 3, 5 Because clients who are anemic experience chronic fatigue, it is important to balance rest and activities to avoid problems with tachycardia and dyspnea. If a client has a problem with dyspnea during activities, supplemental oxygen may be necessary. The health care provider should be notified if bruising occurs, because this could indicate further problems in the hematologic system. Iron should be taken on an empty stomach, but if significant GI upset occurs, it can be taken with food. All alcoholic beverages should be avoided. There is no indication to increase the intake of dairy products and protein. A balanced diet should be followed. (Lewis et al., 10 ed., pp. 608)

The nurse is assessing a client who had a fractured femur repairedwith an external fixator device. Which assessment findings should the nurse report to the health care provider? Select all that apply. 1. Increase in pulse rate in affected leg 2. Paresthesia distal to area of injury 3. Toes on affected leg cool to touch and edematous 4. Reports that pins are hurting 5. Reports of leg pain unrelieved by analgesics or repositioning 6. Client angry and calling loudly to the nurse every 10 minutes

2, 3, 5 Paresthesia, edema, and leg pain unrelieved by analgesics are classic indicators of the development of compartmental syndrome. With a femur fracture, there is some degree of edema postoperatively that may leave the toes on the affected leg cool to touch. An increase in pulse rate is not an indication of a problem; a decrease in pulse strength is. The pins usually do not cause undue pain, and frequently the client is angry regarding the immobility and does not use effective coping measures; neither would need to be reported. (Lewis et al., 10 ed., p. 1483)

In the immediate postoperative phase for a client with a mandibular fracture repair, what are the priority nursing concerns? Select all that apply. 1. Postoperative bleeding 2. Postoperative pain 3. Client positioning 4. Client's inability to speak 5. Respiratory distress 6. Location of scissors and wire cutters

2, 3, 5 Two potential life-threatening problems in the immediate postoperative period are airway obstruction and aspiration of vomitus. Because the patient cannot open the jaws, it is essential that an airway is maintained. Place the patient on the side with the head slightly elevated immediately after surgery. The wire cutter or scissors may be used to cut the wires or elastic bands in case of an emergency. Although pain is an important consideration for any postoperative patient, it is not a life- threatening priority, nor are the patient's inability to speak or bleeding. (Lewis et al., 10 ed., p. 1468)

The nurse understands that a client may experience pain during peritoneal dialysis because of which of the following? Select all that apply. 1. Warming the dialysate solution before administration 2. Too-rapid instillation of the dialysate 3. Infiltration of solution into the bloodstream 4. Increased intraabdominal pressure 5. Too-rapid outflow rate of the dialysate solution

2, 4 Rapid instillation of dialysate fluid and accumulation of the fluid within the abdomen can lead to pain and discomfort. Warming the dialysate solution helps in the clearance and may diminish any cold sensation of fluid entering the body. It will also assist to diminish any cold sensation of fluid entering the body. The dialysate fluid does not infiltrate and enter the circulatory system. Rapid outflow of dialysate does not cause pain. (Lewis et al., 10 ed., pp. 1084-1086)

Which nursing interventions will assist in reducing pressure points that may lead to pressure ulcers? Select all that apply: 1. Position the client directly on the trochanter when side-lying. 2. Avoid the use of donut-type devices. 3. Massage bony prominences. 4. Elevate the head of the bed no more than 30 degrees when possible. 5. When the client is side-lying, use the 30-degree lateral inclined position. 6. Avoid uninterrupted sitting in any chair or wheelchair.

2, 4, 5, 6 Elevating the head of the bed to 30 degrees or less will decrease the chance of pressure ulcer development from shearing forces. When placing the client in a side-lying position, use the 30 degrees lateral inclined position. Do not place the client directly on the trochanter, which can create pressure over the bony prominence. Avoid the use of donut-shaped cushions because they reduce blood supply to the area, which can lead to the extension of the area of ischemia. Bony prominences should not be massaged, because this increases the risk for capillary breakage and injury to underlying tissue leading to pressure ulcer formation. (Ignatavicius & Workman, 8 ed., pp. 436-444)

Which of the following are appropriate nursing actions when measuring visual acuity using a Snellen chart? Select all that apply. 1. Position the client 30 feet (9 meters) away from the chart. 2. Have the client first read the chart with both eyes open. 3. Record visual acuity as the largest line that the client can read correctly. 4. Test each eye individually with the opposite eye covered. 5. Repeat the test with the client wearing corrective lenses. 6. Use a picture chart if the client is unable to read.

2, 4, 5, 6 The nurse should position the client 20 feet (6 meters) away from the chart, not 30 feet (9 meters). Record the smallest line that can be correctly read, not the largest. Each eye is tested separately with the opposite eye covered. The eye test is repeated with the client wearing corrective lenses. You may use an "E" chart or a picture chart if the client is unable to read. (Lewis et al., 10 ed., p. 355)

The nurse is caring for a client scheduled for a bone scan. What should be included in the preprocedure teaching? Select all that apply. 1. Maintain NPO (nothing by mouth) status for 8 hours before the procedure. 2. The procedure will involve intravenous injection of radioisotopes. 3. The procedure will involve a small incision where bone tissue is removed for biopsy. 4. The client will have to lie supine for about an hour during the scan. 5. Avoid stimulants such as caffeine for 24 hours before the procedure. 6. Increase intake of fluids after the procedure is completed.

2, 4, 6 The procedure will include an IV injection of radioisotopes that are given 2 hours before procedure; ensure client empties bladder before the procedure; the procedure takes about 1 hour; the client will lie supine during the procedure; fluid intake should be increased after the procedure to promote excretion of radioisotopes. Food or fluids are not limited before the scan, and no biopsy of the bone will be taken. (Lewis et al., 10 ed., p. 1458)

A client had a left modified radical mastectomy 48 hours ago. What would be important for the nurse to include in a discharge teaching plan for this client? Select all that apply. 1. Massage wound site with essential oils once incision has healed. 2. Avoid needle-sticks in the left arm. 3. Begin active exercises, such as pendulum arm swings, immediately. 4. Avoid abduction and external rotation of the upper arm. 5. Elevate arm on pillows to prevent edema. 6. Take blood pressure readings from the right arm.

2, 5, 6 Important teaching to include in the discharge plan of care for a client who has undergone a mastectomy includes the avoidance of needle-sticks in the arm on the side of the mastectomy and having blood pressure readings taken from the opposite arm. These measures are done to avoid any type of trauma that could lead to the development of lymphedema. Begin finger, wrist, and hand exercises to facilitate muscle contraction and to help prevent edema. Active exercises, such as pendulum swings and finger wall-climbing, are started after the incision has healed. As the area heals, abduction and external rotation will help improve the range of motion. (Lewis et al., 10 ed., p. 1214)

The nurse is caring for a client postoperative thyroidectomy. What action should the nurse prioritize? Select all that apply. 1. Have the client speak every 5 to 10 minutes if hoarseness is present. 2. Support the head with pillows and avoid flexion of the neck. 3. Check the breath sounds for stridor. 4. Assess for tingling in the toes, fingers, and around the mouth or muscular twitching. 5. Assess every 4 hours for the first 24 hours for signs of hemorrhage. 6. Place with head of bed flat, in a side-lying position in case of vomiting.

2,3,4 It is anticipated that the client will be hoarse for 3 to 4 days after surgery. Increased hoarseness can be a sign of edema, but other signs of edema are more frequently seen, such as laryngeal stridor. Serum levels of calcium are important to monitor because of possible damage to the parathyroids during surgery and can be assessed by signs of tetany like tingling and twitching. Increased hoarseness can also be a sign of hypocalcemia, but assessment every 5 to 10 minutes is excessive. Hemorrhage after surgery is a great concern, and assessments should take place every 2 hours for the first 24 hours. Positioning should be semi-Fowler's, supporting the head with pillows to avoid flexion and tension on the suture lines. (Lewis et al., 10 ed., p. 1168)

The nurse is creating a plan of care about exercise for a client newly diagnosed with diabetes. What should be included in the plan? Select all that apply. 1. Exercise needs to be vigorous and daily. 2. Properly fitting footwear is important. 3. Exercise is best done after meals when glucose levels are rising. 4. It is important to monitor glucose levels before, during, and after exercise. 5. Exercise-induced hypoglycemia may occur several hours after exercise.

2,3,4,5 Exercise for clients with diabetes mellitus should include regular and moderate activity with properly fitting footwear to prevent injury. Exercise sessions should include a warm-up and a cooldown period. The best time for exercise is after meals and should be individualized by the health care provider. Monitoring glucose is important before, during, and after exercise. The client needs to know that hypoglycemia can occur several hours after the exercise is completed. (Lewis et al., 10 ed., p. 1134)

The client who has undergone a traditional cholecystectomy has a T-tube in place after surgery. What is the purpose of the T-tube in the care of this client? 1. To remove bile leaking from the incision 2. To provide a means of wound irrigation 3. To drain bile from the common bile duct 4. To prevent rupture of the inflamed gallbladder

3 A T-tube is used after a common bile duct exploration to drain bile and maintain patency of the duct until healing can occur. A T-tube should never be irrigated by the nurse. (Lewis et al., 10 ed., p. 1007)

A client is receiving chemotherapy with several antineoplastic agents. Which nursing observation is considered a common side effect of chemotherapy? 1. Slow, slurred speech 2. Increased leukocytes on complete blood count 3. Stomatitis and oral ulcers 4. Sinus dysrhythmias with bradycardia

3 A common side effect of chemotherapy is stomatitis. It may be manifested as inflammation of the gums and ulcerations in the mouth. There is a decrease in leukocytes, making the client less resistive to infection. Dysrhythmias are not common in cancer therapy; they may occur with electrolyte imbalances secondary to chemotherapy. The slowed speech may occur with hypercalcemia as a complication involving the parathyroid gland. (Lewis et al., 10 ed., pp. 250-256)

The nurse is assessing a client whose condition is being stabilized after experiencing a ST-segment-elevation myocardial infarction. Which assessment is most indicative of inadequate renal perfusion? 1. Increasing serum blood urea nitrogen (BUN) level 2. Urine specific gravity of less than 1.010 3. Urine output of less than 30 mL/hr 4. Low urine creatinine clearance

3 A sustained low cardiac output decreases renal perfusion and results in oliguria and impaired renal function. Oliguria is marked by output of less than 30 mL/hr. Increased BUN, changes in specific gravity (osmolarity), and creatinine clearance will be affected if the client develops renal failure. (Lewis et al., 10 ed., p. 1070)

The nurse is caring for a child who had a long-leg plaster cast applied for a femur fracture. What action is a priority as the cast is drying? 1. Use only the fingertips when moving the cast. 2. Keep the client and cast covered with blankets. 3. Perform frequent neurovascular checks distal to fracture. 4. Place a heat lamp directly over the cast.

3 After cast application, observe for signs of compartment syndrome by performing neurovascular checks distal to the end of the cast. Palms of the hand should be used in turning the client. Heat should not be applied to a damp cast. (Lewis et al., 10 ed., p. 1472)

Which statement about the results of a contraction stress test (oxytocin challenge test) is considered accurate? 1. Negative if no fetal heart rate accelerations occur with accompanying fetal movements 2. Nonreactive if no late decelerations occur in more than half of the contractions 3. Positive if late decelerations occur in more than half of the contractions 4. Reactive if the fetal heart rate accelerates with accompanying fetal movement

3 An oxytocin challenge test, or contraction stress test, is conducted by giving an infusion of oxytocin to the mother and evaluating the fetal response to subsequent uterine contractions as plotted on a fetal monitor. The test result is negative if there are no late decelerations; it is positive if there are late decelerations in more than one-half of the contractions. If the CST result is positive and there is no acceleration of FHR with fetal movement (nonreactive NST result), the positive CST (nonreassuring; abnormal) result is an ominous sign, often indicating late fetal hypoxia. A negative CST (reassuring) result with a reactive NST result is desirable. (Lowdermilk et al., 11 ed., p. 649)

The nurse is caring for a client being discharged after experiencing infective endocarditis. What is most important to include with the discharge teaching? 1. Begin an exercise regimen as soon as possible, progressively increasing intensity each day. 2. Monitor urinary output daily and report a change in color or quantity. 3. Continue antibiotic therapy until the prescription is completed. 4. Track and monitor heart rate and blood pressure daily upon arising.

3 Antibiotics (usually administered by IV piggyback [IVPB]) are indicated for infective endocarditis. This may continue at home with the assistance of a home care nurse to administer the IVPBs, or the client may be changed to oral antibiotics. The continued antibiotic is critical to the prevention of vegetation growth on the valves. The other options are not specific for managing a client with infective endocarditis. (Lewis et al., 10 ed., p. 780)

A client with chronic kidney disease has been prescribed calcium carbonate. What is the rationale for this particular medication? 1. Diminishes incidence of gastric ulcer formation 2. Alleviates constipation 3. Binds with phosphorus to lower concentrations 4. Increases tubular reabsorption of sodium

3 Clients with chronic kidney disease have hypocalcemia and hyperphosphatemia. Clients are prescribed calcium-based phosphate binders, such as calcium acetate or calcium carbonate to improve excretion of phosphorus. (Lewis et al., 10 ed., p. 1079)

What statement would indicate to the nurse that the client understands the discharge teaching regarding his cirrhosis? 1. "I will decrease vitamin B intake." 2. "I need to continue Tylenol daily." 3. "I will weigh myself every day in the morning." 4. "I can eat my regular diet."

3 Daily weight measurement is essential to monitor for volume overload. Clients with cirrhosis need increased vitamin B, especially B6 (pyridoxine). Acetaminophen is hepatotoxic. The diet should be high in carbohydrates, include adequate amounts of protein to build tissue, and be moderate to low in fat intake. (Lewis et al., 10 ed., p. 994)

Pneumonia is a common problem in children with spastic cerebral palsy. The nurse understands that this occurs because: 1. There is an associated dysfunction of the respiratory center in the central nervous system. 2. The immunologic system is immature and does not produce adequate antibodies to fight infection. 3. Decreased mobility leads to stasis of secretions in the respiratory passages. 4. There is a weakness of the voluntary muscles that control respiration.

3 Decreased mobility over an extended period leads to stasis of secretions. There is no associated dysfunction in the CNS regarding the respiratory system. The immune response is mature but may become impaired as a result of the chronic illness. (Hockenberry & Wilson, 10 ed., p. 1149)

A client is admitted with duodenal ulcers. What will the nurse anticipate the client's history to include? 1. Recent weight loss 2. Frequent acetaminophen use 3. Burning pain 2 to 5 hours after a meal 4. Episodes of vomiting

3 Duodenal ulcers are characterized by high gastric acid secretion and rapid gastric emptying. Food buffers the effect of the acid; consequently, pain increases when the stomach is empty. Pain does not characteristically occur immediately after eating but 2 to 5 hours after a meal because the presence of food helps buffer the acid. The client does not usually have bouts of nausea unless bleeding or obstruction is a problem. Duodenal ulcers are associated with aspirin and NSAID use but not acetaminophen. (Lewis et al., 10 ed., p. 911)

In preparing a pediatric client for an appendectomy, the nurse would question which doctor's orders? 1. Penicillin 600,000 units IV piggyback, now. 2. Obtain signed consent form from parents. 3. Administer enemas until clear. 4. 500 mL Ringer's lactate solution at 50 mL/hr.

3 Enemas or laxatives are not administered before surgery in clients with an acute abdomen. If gastric motility is stimulated, there is an increased danger of appendiceal rupture. All other orders are appropriate before an appendectomy. (Lewis et al., 10 ed., pp. 942-943)

A primigravida client is experiencing Braxton Hicks contractions. Which statement is true concerning these contractions? 1. They are intensified by walking about. 2. They are confined to the low back. 3. They do not increase in intensity or frequency. 4. They result in cervical effacement and dilation.

3 False labor contractions decrease when the client is walking, are not concentrated in one part of the uterus, and do not increase in intensity and frequency. True labor is characterized by cervical effacement or dilation. (Lowdermilk et al., 11 ed., p. 287)

During the shift handoff report, the nurse learns that one of the assigned clients is in first-degree heart block. What action should the nurse take? 1. Count the radial pulse for 1 full minute. 2. Determine the cardiac rate at the point of maximum impulse. 3. Evaluate an ECG or monitor strip. 4. Take hourly pulse checks and correlate with blood pressure.

3 First-degree heart block can only be evaluated with an ECG or monitor tracing because the distinguishing factor is a prolonged P-R interval; all beats are being conducted. Other options are appropriate (determine cardiac rate, counting radial pulse for a full minute, and hourly pulse checks with blood pressure assessment) for this client; however, they do not assess first-degree block. (Lewis et al., 10 ed., p. 767)

A client in the acute phase of Guillain-Barré syndrome is admitted with weakness and numbness of the lower extremities, along with continual pain that worsens at night. What would be a priority nursing diagnosis? 1. Fear related to uncertain outcome and seriousness of the problem 2. Acute pain related to paresthesias, muscle aches, and cramps 3. Risk for ineffective breathing pattern related to progression of the disease 4. Risk for aspiration related to dysphagia

3 Guillain-Barré is an acute and rapidly progressing condition affecting the peripheral nervous system characterized by an ascending level of paralysis leading to a serious complication of respiratory failure. This necessitates constant monitoring of the respiratory system. Although other autonomic dysfunctions can occur (requiring the other assessments), such as orthostatic hypotension, hypertension, heart block, bowel and bladder dysfunction, facial flushing, diaphoresis, and lower brainstem involvement (cranial nerves), respiratory problems are the priority. (Lewis et al., 9 ed., p. 1467)

What is an important nursing action when assisting the doctor with a pelvic examination? 1. Instruct the client to bear down and hold her breath during the procedure. 2. Explain to the client that she will not feel any pain. 3. Have the client empty her bladder before the examination begins. 4. Lubricate the speculum well before handing it to the doctor.

3 Having the client void before the examination will make the procedure less painful and more accurate. There is no need for the patient to bear down or hold her breath. The patient should relax as much as possible during the procedure. The level of pain, if any, is highly dependent on the patient and her pain tolerance. Lubricant on the speculum may interfere with the Pap smear. The client should not douche or have intercourse before a pelvic examination, especially if any specimens are to be obtained, because changes in the normal flora and pH could occur from douching and the presence of semen. (Ignatavicius & Workman, 8 ed., p. 1454)

A client has just received 250 mL of packed cells and is now receiving 1000 mL of D5W at 150 mL/hr. The client tells the nurse that he feels dizzy and has a headache. The nurse observes the distended jugular veins with the client in a semi-Fowler's position. What should be the nurse's initial response? 1. Notify the health care provider of the client's symptoms. 2. Check vital signs. 3. Reduce the D5W infusion to keep vein open rate. 4. Lay the client flat.

3 Headache and dizziness in a client receiving IV fluid are frequently signs of fluid overload from the increase in circulating volume, which increases cerebral vascular pressure. After decreasing the IV rate, the nurse should continue with the assessment of the vital signs. If other assessment findings (increased blood pressure, lethargy, bounding pulse, weight gain, adventitious breath sounds) confirm the problem, the physician needs to be notified. If the increase in circulating volume continues, it can cause pulmonary edema. (Lewis et al., 10 ed., p. 709)

A client with acute kidney injury develops severe hyperkalemia. What prescription would the nurse anticipate? 1. Furosemide 2. Calcium carbonate 3. 50% glucose and regular insulin 4. Epoetin alfa

3 Hyperkalemia can develop into an emergency situation (cardiac arrest). It is important to quickly move the potassium back into the cells by administering 50% glucose and regular insulin, usually in conjunction with some type of base to correct the acidosis, such as sodium bicarbonate or calcium gluconate given intravenously. Insulin assists in the movement of potassium into the cells and helps to reduce the serum potassium level. Calcium carbonate is used for the treatment of hyperphosphatemia that occurs with chronic kidney disease. Procrit is used for the treatment of anemia caused by a decrease in erythropoietin production by the kidneys. A diuretic, such as Lasix, may lead to a loss of potassium, but the rate is too slow. (Lewis et al., 10 ed., p. 1)

A nurse is urgently called to a homebound neighbor's house. The neighbor is found unconscious and has a history of insulindependent diabetes. After determining there is no functioning glucometer available, what should the nurse's next action be? 1. Administer 10 units of regular insulin subcutaneously. 2. Arouse the client to drink 4 to 6 ounces of orange juice. 3. Administer glucagon 1 mg subcutaneously. 4. Find a phone to call EMS.

3 Hypoglycemia can be quickly reversed with effective treatment. Treatment should not be delayed, because permanent brain death can occur. If monitoring equipment is not available or the client has a history of fluctuating blood glucose levels, hypoglycemia should be assumed and treatment initiated. (Lewis et al., 10 ed., p. 1146)

A client with portal hypertension and ascites has had a paracentesis to relieve respiratory compromise. What medication will the nurse anticipate the client will receive? 1. D10W 2. Morphine 3. IV salt-poor albumin 4. Furosemide

3 IV salt-poor albumin is given to replace protein lost in the ascites fluid and to restore oncotic pressure in the vascular bed. D10W is an IV fluid, not a medication. Furosemide will diminish the ascitic fluid. The client has respiratory compromise and would not be given a narcotic pain medication (morphine). (Lewis et al., 10 ed., p. 991)

What is the primary purpose of giving lactulose to a client with advanced liver disease? 1. To ensure regular bowel movements 2. To prevent bowel obstruction 3. To decrease ammonia levels in the blood 4. To promote clotting

3 In a client with end-stage liver disease, lactulose is used to decrease ammonia levels in the blood, thus improving cognition and alertness. The ammonia is eliminated through the regular bowel movements that the medication promotes, preventing obstructions. Lactulose is not involved in blood clotting. (Lewis et al., 10 ed., p. 992)

A client and her husband are positive for the sickle cell trait. The client asks the nurse about the chances of her children having sickle cell disease. The nurse understands that this genetic problem will reflect what pattern in the client's children? 1. One of her children will have sickle cell disease. 2. Only the male children will be affected. 3. Each pregnancy carries a 25% chance of the child being affected. 4. If she has four children, one of them will have the disease.

3 In autosomal recessive traits, when both parents have the trait, there is a 25% chance with each pregnancy that the child will have the disease. When only males are affected, it is an X-linked recessive disease, such as hemophilia. Because there is a 1 in 4 chance of transmitting the trait, it is possible that none of the children would be affected. Frequently, there is a negative family history of the disease. (Lewis et al., 10 ed., p. 616)

Clients with COPD usually receive low-dose oxygen via nasal cannula. The nurse understands that which problem may occur if the client receives too much oxygen? 1. Hyperventilation 2. Tachypnea 3. Hypoventilation or apnea 4. Increased snoring

3 In clients with chronic high Pco2 levels (COPD), the administration of oxygen at a flow rate that increases the Pao2 may cause apnea and require the use of a bag valve mask resuscitator to ventilate the client. When the Pao2 increases significantly, it can decrease the client's stimulus to breathe and may cause carbon dioxide narcosis. (Ignatavicius & Workman, 8 ed., pp. 515, 563)

While talking with a client with a diagnosis of end-stage liver disease, the nurse notices the client is unable to stay awake and seems to fall asleep in the middle of a sentence. The nurse recognizes these symptoms to be indicative of what condition? 1. Hyperglycemia 2. Increased bile production 3. Increased blood ammonia levels 4. Hypocalcemia

3 In end-stage liver disease, the liver cannot break down ammonia byproducts of protein metabolism. The increased ammonia levels in the serum cross the blood-brain barrier, causing uncontrolled drowsiness and confusion. Hyperglycemia is characterized by polyphagia, polydipsia, and polyuria, along with fatigue, weight loss, excessive thirst, and abdominal pain. Hypocalcemia is characterized by tetany symptoms. Increased bile production does not cause neurologic symptoms; it is related more to digestion. (Lewis et al., 10 ed., p. 990)

The nurse is instructing a student nurse concerning the differences in the joints affected in osteoarthritis and rheumatoid arthritis. Which of the following is an important fact to include? 1. In rheumatoid arthritis, weight-bearing joints are affected first. 2. In osteoarthritis, only the small joints of the fingers are affected. 3. In rheumatoid arthritis, there is usually bilateral joint involvement. 4. In osteoarthritis, joint destruction is due to changes in the synovial fluid.

3 In rheumatoid arthritis, small joints typically first (proximal interphalangeal [PIPs], metacarpophalangeals [MCPs], metatarsophalangeals [MTPs]), wrists, elbows, shoulders, knees. Usually bilateral, symmetric joint involvement. In osteoarthritis, weightbearing joints of knees and hips are most often affected, although small joints of the hands and feet and cervical/lumbar spine, often asymmetric, may also be involved; and joint destruction is caused by long-term use and weight bearing. (Lewis et al., 10 ed., p. 1525)

Clients with liver disease frequently develop a problem with jaundice. What would the nurse identify as the physiologic cause of jaundice? 1. Increased levels of ammonia 2. Increased alanine aminotransferase (ALT) level 3. Bilirubin levels above 4 mg/dL (68.4 umol/L) 4. Increased red blood cell production

3 Increased levels of bilirubin (greater than 2.0 mg/dL [34 umol/L]) cause a discoloration of the skin called jaundice. The bilirubin value needs to be two to three times the normal level for jaundice to be manifested. Normal value of total bilirubin is 0.2 to 1.3 mg/dL (5-21 umol/L). Jaundice occurs because of an alteration in normal bilirubin metabolism or flow of bile into the hepatic or biliary system. Increased ammonia and ALT levels do not cause jaundice; they are problems associated with the malfunctioning liver. Hemolytic jaundice is due to an increased RBC production. (Lewis et al., 10 ed., pp. 977, 986)

Which nursing observations support the identification of the early development of a chronic subdural hematoma in a 3-month-old infant? 1. Closed posterior fontanel; open anterior fontanel 2. Retinal hemorrhages and hemiparesis 3. Increased irritability and vomiting 4. Papilledema and regressive behavior

3 Irritability and vomiting are common signs of increased ICP in the infant; the symptoms are often delayed in the infant because of the open fontanels. Retinal hemorrhage, paresis, and papilledema are indications of more acute hematoma formation. (Hockenberry & Wilson, 10 ed., p. 989)

The nurse is evaluating a central venous line before administering the client's chemotherapy. What observation would cause the nurse the most concern? 1. Nurse is unable to withdraw blood into line. 2. Dressing was changed 24 hours ago. 3. Inflammation and exudate are present at the insertion site. 4. Fluid infusing is D5W and 0.45% normal saline.

3 Irritation at the insertion site is a problem in a client who is immunocompromised. Checking the IV site and surrounding area for signs of infection is a priority nursing action. Frequently, central lines are long and narrow, and withdrawing blood may not occur. Transparent dressings are changed on an as-needed basis. Opaque (gauze) dressings are usually changed every 48 hours. (Lewis et al., 10 ed., pp. 295-297)

In the recovery room, the best immediate postoperative position for an infant who has had a cleft lip repair is: 1. Prone with the head turned to one side 2. Left Sims' position 3. Supine with the head turned to the side 4. Trendelenburg position to facilitate drainage

3 It is important that the child be positioned in such a manner that he does not traumatize the incisional area and that the airway is maintained, which would be supine with the head turned to the side. Trendelenburg position would compromise respiration. The prone position would cause trauma to the incisional area and is to be avoided in infants. Sims' position is not appropriate for an infant because the infant should rest in a supine position. (Hockenberry & Wilson, 10 ed., pp. 307)

A client asks the nurse about an alternative remedy for hot flashes. Which dietary supplement is the client asking the nurse about? 1. Ginseng. 2. Valerian. 3. Feverfew. 4. Black cohosh.

3 Kava is an herbal supplement that is used to relieve anxiety, promote sleep, and relax muscles and is known to be hepatotoxic. Monitoring liver function studies would be an appropriate action. Ma huang (ephedra) is an herbal supplement that can cause hypertension and stimulates the central nervous system. (Burchum & Rosenthal, 9 ed., pp. 1328-1329)

Which of the following signs and symptoms would the nurse assess for in a client with possible lithium toxicity? 1. Hypotension, bradycardia, polyuria 2. Tachycardia, hypertension, convulsions 3. Diarrhea, ataxia, seizures, lethargy 4. Urinary frequency, vomiting, fever

3 Lithium toxicity is a serious problem for clients with bipolar disorder. Symptoms include diarrhea, confusion, ataxia, slurred speech, hypotension, seizures, oliguria, coma, and death. (Halter, 7 ed., p. 240)

The mother of a 15-month-old child who is immunosuppressed asks about continuation of the childhood vaccines. Which immunizationsare not recommended to be given to the childduring immunosuppression? 1. Diphtheria, tetanus, and pertussis (DTaP); hepatitis B 2. Haemophilus influenzae B 3. Varicella; measles-mumps-rubella (MMR) 4. Inactivated polio; diphtheria, tetanus, and pertussis (DTaP)

3 Live viruses are usually not administered when a client is immunosuppressed. Frequently, measles-mumps-rubella (MMR) vaccine and varicella vaccine are not given to the immunocompromised client; they may be administered when the client has a more competent immune system. Childhood vaccinations are encouraged when the client is not immunocompromised (in remission or not on immunosuppressant medications). (Hockenberry & Wilson, 9 ed., p. 1455)

Which client is at the highest risk for developing chronic kidney disease? 1. Client with severe acute glomerulonephritis 2. Client with placenta previa and hemorrhage at delivery 3. Client with poorly controlled long-term hypertension 4. Client who received IV aminoglycosides for an infection

3 Long-term hypertension and diabetes are the leading causes of chronic kidney disease. The client with placenta previa and hemorrhage is at risk for developing acute kidney injury. Acute glomerulonephritis may decrease renal function but seldom to the point of chronic kidney disease, as most clients recover. Aminoglycosides can be nephrotoxic and cause damage (acute kidney injury), but chronic kidney disease is not common. (Lewis et al., 10 ed., p. 1083)

An 8-year-old child is admitted after an accident where he sustained a closed head injury. The child is alert and oriented but very lethargic. There is clear fluid draining from the child's nose. What is the best nursing action? 1. Gently suction the fluid from the nasal area. 2. Turn from side to side only. 3. Keep head of bed elevated. 4. Encourage participation in games to play in bed.

3 Most CSF leaks resolve spontaneously. The child should be maintained on bed rest until drainage ends. Nonsteroidal antiinflammatory drugs are not contraindicated. The child may turn and assume a position of comfort, and there are usually no dietary restrictions. (Hockenberry & Wilson, 10 ed., pp. 991-992)

Which instruction should be included in discharge teaching for the client with a new prescription for simvastatin? 1. Flushing occurs in almost all individuals. 2. Sedation is common but will decrease with time. 3. Liver enzyme levels should be monitored every few months. 4. Watch closely for occurrence of postural hypotension.

3 Most of the "statin" drugs used for hyperlipidemia are hepatotoxic. Liver enzyme levels should be determined as a baseline before administration of the drug is started and then checked periodically throughout therapy. (Lewis et al., 10th ed., pp. 709-711)

The nurse is infusing dialysate during peritoneal dialysis. What is a nursing action to make the client more comfortable at this time? 1. Increase the rate of flow. 2. Raise the head of the bed. 3. Turn the client from side to side. 4. Refrigerate the fluid before infusion.

3 Movement of the client will help disseminate the fluid throughout the abdomen. The fluid should be instilled within the prescribed period of time; therefore, the dialysate flow should not be decreased. Raising the head of the bed will make the client more uncomfortable and increase intraabdominal pressure. The solutions should be infused at body temperature. (Lewis et al., 10 ed., p. 1085)

The nurse is administering nitroglycerin intravenously to a client experiencing chest pain of an 8 on a 1 to 10 scale. What assessment changes would cause the nurse to decrease the infusion rate? 1. Pain drops from an 8 to a 4. 2. Heart rate increases from 110 to 115 beats per minute. 3. Blood pressure drops from 110/65 (80) to 89/44 (59) mmHg. 4. Client verbalizes his head is pounding.

3 Nitroglycerin is a vasodilator. It dilates the coronary arteries, thereby increasing myocardial blood supply; vasodilation of the peripheral circulation decreases the pressure against which the heart must pump (decreases afterload) and, by dilating the venous system, allows blood to pool in the venous system (decreases preload). It is important to continue the delivery of nitroglycerin until the client is pain free, since pain means coronary ischemia in the cardiac client. A headache is anticipated, as is tachycardia. Hypotension is expected with a vasodilator, but a drop this significant since the mean arterial pressure is below 60 mmHg. (Lehene, 9 ed., p. 593)

While discussing her diagnosis of hypertension, a client asks the nurse how long she is going to have to take all of the medications that have been prescribed. On what principle is the nurse's response based? 1. The client will be scheduled for an appointment in 2 months; the doctor will decrease her medications at that time. 2. As soon as her blood pressure (BP) returns to normal levels, the client will be able to stop taking her medications. 3. To maintain stable control of her BP, the client will have to take the medications indefinitely. 4. The nurse cannot discuss the medications with the client; the client will need to talk with the doctor.

3 Noncompliance with blood pressure medications is a common problem in the treatment of hypertension. The client must understand that the only way to keep her blood pressure under control is to continue to take her medications, potentially for the rest of her life. She will not be able to discontinue the medications unless there is a significant change in her condition as a result of weight loss, an exercise program, and/or decreased stress. Patients usually require follow-up and adjustments at monthly intervals until the goal BP is reached. Antihypertensives control BP but do not cure hypertension, therefore the medication cannot be stopped once the target reading is reached. (Ignatavicius & Workman, 8th ed., pp. 712-713, 717-718)

Four hours after aortic-femoral bypass graft surgery, the nurse assesses the client and is unable to palpate pulses in the operative leg. The client complains of pain in the leg. What is the first nursing action? 1. Massage the leg and apply warm towels. 2. Elevate the leg and recheck the pulse. 3. Call the physician immediately. 4. Help the client ambulate.

3 Occlusion to the aortic/femoral bypass graft is considered a medical emergency, and physician notification is imperative. No other nursing options would alleviate the problem. Massaging the leg and having the client ambulate would be contraindicated. If the pulses cannot be palpated and the client is experiencing pain, the nurse should not wait to call the physician. (Ignatavicius & Workman, 8th ed., pp. 723-724)

A client admitted with a pheochromocytoma returns from the operating room after adrenalectomy. Which assessment is most concerning? 1. Glucose of 70 mg/dL. 2. Potassium of 3.4 mEq/L. 3. Blood pressure of 169/98 mm Hg. 4. Sodium of 146 mEq/L.

3 Pheochromocytoma is a tumor in the adrenal medulla that produces excess catecholamines (epinephrine and norepinephrine). An excess of these catecholamines can cause severe hypertension. Surgery (adrenalectomy) alleviates the elevated blood pressure most of the time. In 10% to 30% of clients, hypertension remains and must be monitored and treated. Electrolyte imbalances and blood sugar are not typically affected. (Lewis et al., 10 ed., pp. 1107, 1181)

During the night, a client with a diagnosis of acute coronary syndrome is found to be restless and diaphoretic. What is the best nursing action? 1. Check his temperature and determine his serum blood glucose level. 2. Turn the alarms low and promote sleep by decreasing the number of interruptions. 3. Check the monitor to determine his cardiac rhythm and evaluate vital signs. 4. Call the physician to obtain an order for sedation.

3 Restlessness and diaphoresis may be indicative of decreased cardiac output, frequently originating from a dysrhythmia. Checking temperature and blood glucose levels is not a priority. Turning the alarm sound to low and reducing interruptions to facilitate sleep will be done when all physical problems are resolved. Physiologic needs must be addressed first. It is important to obtain critical assessment data before calling the doctor. (Lewis et al., 10 ed., pp. 742)

On the first postoperative day after a right lower lobe (RLL) lobectomy, the client deep breathes and coughs but has difficulty raising mucus. What nursing observation would indicate the client is not adequately clearing secretions? 1. Chest x-ray film showing right-sided pleural fluid 2. A few scattered crackles on RLL on auscultation 3. Increase in Paco2 from 35 to 45 mmHg 4. Decrease in forced vital capacity

3 Retained secretions may cause hypoventilation; this results in an increase in the Paco2. The other options do not as effectively reflect a problem with clearing mucus. Pleural fluid is not removed via coughing; the fluid is in the pleural space, not in the lung. Although the Paco2 is within the normal limits, there is still an increase noted, which is due to the hypoventilation. The nurse cannot easily measure the forcedvital capacity at the bedside. (Ignatavicius & Workman, 8 ed., pp. 579, 581)

The nurse is caring for a client with an acute onset of shortness of breath and a respiratory rate of 28 breaths per minute. Arterial blood gasses are pH 7.20; Paco2 47, HCO3 2 24. What is the priority plan of care? 1. Slow the respiratory rate with relaxation and sedation. 2. Improve the pH by administering sodium bicarbonate. 3. Determine a cause of the shortness of breath with further assessment. 4. Intubation to maintain respiratory effort.

3 Sudden shortness of breath with tachypnea is always a concern and a priority problem. The ABGs show that the client is in a respiratory acidosis that is uncompensated (from the sudden onset of symptoms). Further assessment should include a pulse oximetry reading, blood pressure, heart rate, auscultation of lungs, and assessment for tracheal shift and bilateral lung expansion. Anxiety level is another important assessment and should be addressed. Sedation would be contraindicated, since it would slow the respirator rate causing further retention of Paco2. Sodium bicarbonate is more commonly used in metabolic acidosis, and intubation is not warranted unless the client lacks a breathing response or more severe respiratory acidosis. (Lewis et al., 10 ed., pp. 288, 1616)

A client comes to the outpatient clinic with impetigo on his left arm. What information would the nurse give this client? 1. Apply antibiotic ointment to the crusted lesions. 2. Wash the lesions with soap and water and then apply a steroid ointment. 3. Soak the scabs off the lesions and apply an antibiotic ointment. 4. Wash the lesions with hydrogen peroxide and apply an antifungal cream.

3 Teaching should include the use of warm saline or aluminum acetate soaks followed by soap and water removal of crusts and application of a suitable antibiotic ointment, such as mupirocin (Bactroban). Hydrogen peroxide has little ability to reduce bacteria in wounds and can actually inflame healthy skin cells that surround a lesion, increasing the amount of time the wounds take to heal. Impetigo is caused by group A beta-hemolytic streptococcus or Staphylococcus species, which are bacterial and would not be treated by an antifungal cream. If lesions are on the face, a systemic antibiotic also may be given. (Hockenberry & Wilson, 10 ed., p. 226)

Each newborn should be screened before discharge for phenylketonuria (PKU), which can lead to mental retardation if not treated. The nurse knows that the blood screening test will have the most reliable results if the baby: 1. Is kept NPO for 6 hours before the blood is drawn 2. Has the heel wrapped in a warm towel for 30 minutes 3. Has been fed breast milk or formula at least 24 hours before the test 4. Is held by the mother and remains perfectly still

3 The Guthrie blood-screening test measures the amount of phenylalanine in the blood. It is most reliable if blood is drawn at least 24 hours after the newborn has ingested a source of protein. Breast milk and formula are both sources of protein. The heel stick is usually how the screening test is collected, so warming the extremity may increase blood flow. The infant would be held by a health care person, not the mother, for the blood collection. (Hockenberry & Wilson, 10 ed., p. 269)

The nurse is caring for an older adult client with edema, tachycardia, hypertension, and jugular venous distention. Which nursing action should the nurse prioritize to evaluate the client's fluid status? 1. Measure the intake and output (I&O). 2. Check for thirst and skin turgor. 3. Evaluate changes in daily weight. 4. Evaluate vital signs every 4 hours.

3 The assessment findings demonstrate fluid volume excess. The priority assessment for a client with fluid problems is to obtain the daily weight. Weight gain and loss are the most accurate measurements of fluid gain and loss. Checking tissue turgor on an older adult is not accurate because many older adults have poor turgor. Thirst is too nonspecific and would be a proper assessment for dehydration rather than fluid volume excess. The I&O is important, but it is not as accurate in evaluating the amount of fluid retained as is the daily weight. (Lewis et al., 10 ed., p. 277)

A client is walking down the hall, and he begins to experience vertigo. What is the most important nursing action when this occurs? 1. Have the client sit in a chair in a brightly lit room. 2. Administer meclizine PO. 3. Help the client sit or lie down. 4. Assess whether the problem is vertigo or dizziness.

3 The client experiencing vertigo is severely imbalanced and at high risk of falling, thus client safety is the priority. He should be assisted to sit or lie down immediately or he may fall. The client should be safely sitting or lying down before any treatment or further assessment can continue. Although sitting in a chair is not a bad option, the fact that the rest of the statement includes sitting in a well-lit room would not be prudent. The preferred action during an acute attack would be to have the client lie down in a quiet, darkened room. (Lewis et al., 10 ed., p. 386)

A client with chest pain is on a cardiac monitor. The monitor is showing ventricular tachycardia at a rate of 150 beats/min with multiple PVCs. The client is awake and coherent, and oxygen is being administered at a rate of 6 L/min via a nasal cannula. What is the nurse's next action? 1. Immediately defibrillate. 2. Administer adenosine IV push. 3. Assess the blood pressure. 4. Auscultation lung sounds.

3 The client is having chest pain; it is a priority to evaluate their blood pressure to determine whether they are tolerating the rhythm. Adenosine is given for supraventricular tachycardia. Defibrillation will be necessary if the client loses consciousness. Lung sounds are not a priority at this time. (Lewis et al., 10 ed., p. 769)

The nurse prepares a liquid medication and then finds that the client no longer needs the medication. What is the most appropriate nursing action? 1. To keep the count correct, record that the dose was taken. 2. Charge for the dose because it must be paid for. 3. Record the medication as "not taken" and discard the poured dose. 4. Pour the medication back into the container.

3 The dose should be recorded as "not taken," and the poured dose should be discarded. If the dose is a controlled substance, the discarded dose must be witnessed. Checking the medication administration record or the medication order for changes before preparing the medication helps eliminate

What is the first step the nurse should take to ensure that the right medication is being given to a client? 1. Check the client's ID band. 2. Read the information insert for directions as to correct administration. 3. Check the order with the medication administration sheet. 4. Check the expiration date on the medication.

3 The first step in drug delivery to a client is to check the order with the medication administration sheet for possible discontinuance or a change in dose, route, or time. The question asks for steps to ensure the right medication is being given, not for client identification. (Potter & Perry, 9 ed., p. 635)

When teaching a family and a client about the use of a hearing aid, the nurse will base the teaching on what information regarding the hearing aid? 1. It provides mechanical transmission for the damaged part of the ear. 2. It stimulates the neural network of the inner ear to amplify sound. 3. It amplifies sound and directs it into the ear canal. 4. It will assist the client to interpret the incoming sounds more effectively.

3 The hearing aid amplifies sound but does not change the overall ability to interpret incoming sound. Sensorineural hearing loss is the inability of the client to interpret the sounds. A hearing aid is used for clients with conductive hearing loss or a mix of conductive and sensorineural loss. The hearing aid does not stimulate the neural network to amplify sound, such as a cochlear implant. (Lewis et al., 10 ed., p. 391)

A new employee at a facility needs to receive the hepatitis vaccine. Which statement reflects accurate understanding of the immunization? 1. "I need to receive six shots—once a month until I show positive antibodies to hepatitis." 2. "Once I receive the hepatitis vaccine, I will be immune to all types of hepatitis." 3. "I will receive three injections over a period of months, which should protect me from hepatitis B." 4. "The hepatitis vaccine is an oral vaccine with live attenuated virus."

3 The hepatitis vaccine is used to protect health care workers and other individuals from hepatitis B. The series consists of three intramuscular injections, the first two given at least 1 month apart and the third given 4 to 6 months later to provide long-lasting protection from hepatitis B only. (Lewis et al., 10 ed., p. 982)

A young woman comes into the emergency department with menorrhagia. What is the priority concern for the nurse caring for this client? 1. The chronic bleeding will cause anemia. 2. The client is at a high risk for development of an infection. 3. The woman may be pregnant and is aborting the fetus. 4. The increased bleeding will promote later problems with infertility.

3 The nurse must rule out pregnancy/aborting first because of the potential for shock or damage to a fetus. Once aborting is ruled out or treated, the nurse addresses the issues of anemia and infection. Infertility would be the last concern, although it should be assessed. (Lewis et al., 10 ed., p. 1246.)

After delivery, a neonate is transferred to the nursery. The nurse is planning interventions to prevent hypothermia. What is the common source of radiant heat loss? 1. Low room humidity 2. Cold weight scale 3. Cool bassinette walls 4. Variable room temperature

3 The nurse understands that the common sources of radiant heat loss include cool bassinettes and bassinettes placed close to windows or areas of drafts. Low room humidity promotes evaporative heat loss. When the infant's skin has direct contact with a cooler object, such as a cold weight scale, conductive heat loss may occur. Convective heat loss occurs with a cool room temperature. (Hockenberry & Wilson, 10 ed., p. 343)

The nurse understands what major difference between benign and malignant tumors? Malignant tumors: 1. Are encapsulated and immovable 2. Grow at a faster rate than benign tumors do 3. Invade adjacent tissue and metastasize 4. Cause death, whereas benign tumors do not

3 The primary difference between benign and malignant tumors is the ability of the malignant tumor to invade adjacent tissues and metastasize. Benign tumors tend to be encapsulated, and both types of tumors can lead to death. As benign tumors expand, they can adversely affect organ function. The growth of malignant and nonmalignant tumors varies, depending on the characteristics and location of the tumor. (Lewis et al., 10 ed., p. 240)

The nurse is admitting a postoperative client after removal of an acoustic neuroma. What would be most important to include in the postoperative nursing care for this client? 1. Determining when the client will begin chemotherapy 2. Evaluating hearing status 3. Assessing for clear, colorless nasal discharge 4. Encouraging the client to discuss problems with hearing loss

3 The removal of an acoustic neuroma may result in a cerebrospinal fluid (CSF) leak that would be a significant risk of infection, which would be noted by clear, colorless nasal discharge. This symptom requires immediate assessment and intervention. The tumor is benign; therefore the client will not begin chemotherapy. The nurse will need to evaluate the hearing loss in the affected ear as well as the effect of hearing loss on the client's body image and ability to perform ADLs; however, these are secondary to the assessment of a possible CSF leak. Hearing loss may also occur after this procedure, but this would not be as much a priority as the possible CSF leak. (Lewis et al., 10 ed., p. 387)

Combined therapy of radiation and chemotherapy can have a significant therapeutic impact on the survival of an individual with cancer. The nursing priority for these clients includes measures to: 1. Monitor for acute renal tubular necrosis 2. Control nausea and vomiting 3. Prevent infection 4. Maintain hydration and nutrition

3 The statistics indicate that infection is the most common cause of morbidity in clients with cancer. Good handwashing, monitoring white blood cell counts, checking temperatures (watching for elevations), and providing protective isolation when needed (when clients are severely immunosuppressed) are the primary measures to prevent infection. (Lewis et al., 10 ed., p. 262)

The nurse is preparing to administer an intramuscular injection to an infant who is 8 months old. Which muscle would be the most appropriate injection site? 1. Deltoid 2. Dorsogluteal 3. Vastus lateralis 4. Ventrogluteal

3 The vastus lateralis is the preferred site for an infant because the muscle is not located near any vital nerves or blood vessels. It is the best choice for IM injections for children younger than 3 years of age. The deltoid muscle has too little muscle mass in a young infant 2 or toddler and is unable to hold large injected volumes of medication. (Hockenberry & Wilson, 10 ed., p. 917)

On auscultation, the nurse hears wheezing in a client with asthma. Considering the pathophysiology of asthma, what would the nurse identify as the primary cause of this type of lung sound? 1. Increased inspiratory pressure in the upper airways 2. Dilation of the respiratory bronchioles and increased mucus 3. Movement of air through narrowed airways 4. Increased pulmonary compliance

3 The wheezing is due to narrowing of the airway caused by bronchospasm. Increased mucus production hinders the airway as well; this also results in trapping of air in the alveoli. Increased pulmonary compliance indicates the lungs have good recoil and expansion. (Ignatavicius & Workman, 8 ed., pp. 550-551)

A 3-year-old child had a myringotomy about a week ago. The mother calls the nurse to report that one of the tubes fell out. She found the tube on the child's pillow. After the nurse makes an appointment for the child to be seen in the clinic, what would be important to tell the mother? 1. Observe for any purulent or bloody drainage from the ear. 2. Rinse the tube in soapy water and keep it. 3. Do not allow any water to get into the child's ears. 4. Do not allow the child to play outside.

3 There may still be an opening in the eardrum where the tube was placed. It is important to continue to keep all water out of the child's ear to prevent infection. Observing for drainage is important for the mother to do, but not allowing water to get into the ears is more important. The tube is usually thrown away. There is no need to wash it, and the child can play outside if he is comfortable. (Hockenberry & Wilson, 10 ed., pp. 1181-1182)

An older adult client has an open wound over the coccyx that extends through the dermis and subcutaneous tissue, exposing the deep fascia. The wound edges are distinct, and the wound bed is a pink-red color. There is no bruising or sloughing. The nurse would correctly document this ulcer as what stage? 1. Stage I 2. Stage II 3. Stage III 4. Stage IV

3 This is classified as a stage III pressure ulcer because of the fullthickness tissue loss extending to the deep fascia. Subcutaneous fat may be visible, but bone, tendon, or muscle is not exposed. Slough may be present but does not obscure the depth of tissue loss. There may be undermining and tunneling. A stage I pressure ulcer is characterized by intact skin with nonblanchable redness of a localized area usually over a bony prominence. Darkly pigmented skin may not have visible blanching; its color may differ from the surrounding area. A stage II pressure ulcer is characterized by partial-thickness loss of dermis presenting as a shallow, open ulcer with a red-pink wound bed without slough, which also may present as an intact or open/ruptured serum-filled blister. A stage IV pressure ulcer is characterized by full-thickness tissue loss with exposed bone, tendon, or muscle. Slough or eschar may be present on some parts of the wound bed and often includes undermining and tunneling. (Lewis et al 10 ed., p. 173)

While a client's wife is visiting, she observes the client's chest drainage system and begins to nervously question the nurse regarding the amount of bloody drainage in the system. What is the best response from the nurse? 1. "Your husband has been really sick; this must be a very difficult time. Let's sit down and talk about it." 2. "I have checked all of the equipment, and it is working fine; you do not need to worry about it." 3. "The system is draining collected fluid from around the lungs. The drainage is expected and does not mean that he is bleeding." 4. "The chest tube is draining the secretions from his chest; it is important for him to deep breathe frequently."

3 This is important information to explain to the client's wife regarding the bloody drainage in the chest tube collection system. After the nurse has explained the reason for the drainage, it would then be appropriate to sit down and talk with the wife. Checking the equipment may be appropriate, but telling the wife not to worry is a communication block (false reassurance). Having the client breathe deeply does not answer the question or address the wife's concern. (Ignatavicius & Workman, 8 ed., pp. 578-579)

When preparing a client for electroconvulsive therapy (ECT), the nurse would include which of the following actions? 1. Provide orientation to time. 2. Assess vital signs for 30 minutes to 1 hour. 3. Remove dentures and maintain NPO status. 4. Encourage problem solving in social settings.

3 To prepare a client for ECT, the nurse must do the following: check the client's record for routine preoperative information; institute and maintain NPO status for 6 hours before the treatment because the client will be receiving a general anesthetic agent— short-acting barbiturate (methohexital) and muscle paralyzing agent (succinylcholine); have the client remove dentures; and administer preoperative medication. (Halter, 7 ed., p. 271)

The night-shift nurse notes at the end of her shift that a client who had a mastectomy has a total of 90 mL of serosanguineous drainage from the incision over a 24-hour period. What is the best nursing action? 1. Report amount of drainage to the physician. 2. Start frequent blood pressure checks and observe for hemorrhage. 3. Continue to monitor the drainage. 4. Reinforce packing at the wound site.

3 Up to 100 mL of serosanguineous fluid would be an acceptable amount of drainage over a 24-hour period in a client who has had a mastectomy. Drains are usually removed when there is less than 25 mL in a 24-hour period. There is no indication of hemorrhage or the need to perform frequent blood pressure checks. If the nurse observes a greater amount of fluid in the drains, then it would be important to notify the physician. (Ignatavicius & Workman, 8 ed., pp. 1472-1474)

Which of the following circumstances is most likely to cause uterine atony leading to postpartum hemorrhage? 1. Hypertension 2. Cervical and vaginal tears 3. Urine retention 4. Endometritis

3 Urine retention is a common cause of uterine atony and can lead to postpartum hemorrhage. Urine retention causes a distended bladder to displace the uterus above the umbilicus and to the side, which prevents the uterus from contracting. The uterus needs to continue contracting if bleeding is to stay within normal limits. Cervical and vaginal tears can cause postpartum hemorrhage, but in the postpartum period, a full bladder is the most common cause of uterine bleeding. Neither endometritis, an infection of the inner lining of the endometrium, nor maternal hypertension causes postpartum hemorrhage. (Lowdermilk et al., 11 ed., p. 486)

A client with hypertension asks the nurse what type of exercise she should do each day. What is the nurse's best response? 1. "Exercise for an hour, but only three times a week." 2. "Walk on the treadmill for 45 minutes every morning." 3. "Begin walking and increase your distance as you can tolerate it." 4. "Exercise only in the morning and stop when you get tired."

3 When any client begins exercising, it should be gradually, with increasing activity as the client tolerates it. A complication of hypertension is heart failure, which may first be seen as dyspnea on exertion. The client should exercise as tolerated and stop when she gets tired or begins to have shortness of breath, regardless of the amount of time she has already exercised. (Lewis et al., 10th ed., p. 689).

A week after a right knee arthroplasty, a client continues to report moderate knee pain with activity. What recommendations should the nurse make? Select all that apply. 1. Rotate warm and ice packs to the incisional site around the clock. 2. Increase the amount of narcotic pain medication. 3. Perform isometric and active range of motion to the extremity every hour. 4. Take pain medicine before physical therapy sessions. 5. Progress physical activity as directed by the therapist.

3, 4, 5 The emphasis after a knee arthroscopic surgery should be on pain management and physical therapy. Postoperative pain is reduced with movement, so it should be encouraged. Isometric and active range of motion will increase muscle strength and flexion of the joint. Pain medicine should be reduced over time to encourage activity. Increased used of narcotics will decrease the mental alertness and the activity level. (Lewis et al., 10 ed., p. 1491)

The nurse understands that the following are general adverse effects of antineoplastic drugs. Select all that apply. 1. Urinary retention 2. Infertility 3. Stomatitis 4. Bone marrow depression 5. Extravasation 6. Nausea

3, 4, 6 Adverse effects of antineoplastic drugs can be classified as acute, delayed, or chronic. Acute toxicity includes nausea, vomiting, arrhythmias, and allergic reactions. Delayed side effects include stomatitis, alopecia, and bone marrow depression. Chronic toxicity involves organ damage. Common urinary problems include cystitis and nephrotoxicity. Extravasation is not an adverse effect but a complication of an infiltrated IV running a chemotherapy medication that is a vesicant. (Lewis et al, 10 ed., pp. 250-256)

A client is prescribed levothyroxine daily. What should the nurse include in the discharge teaching? Select all that apply. 1. Taper the dose, never stop abruptly. 2. Take it at bedtime to avoid the side effects. 3. Call the health care provider if you experience palpitations or nervousness. 4. Decrease the intake of juices and fruits with high potassium and calcium contents. 5. Regular follow-up care will be required.

3, 5 Levothyroxine increases the metabolic rate of body tissues. Some serious side effects include cardiovascular collapse, dysrhythmias, and tachycardia. Because of these side effects, clients should be instructed not to take the medication if their pulse is greater than 100 beats/min and to notify their provider of headaches, nervousness, chest pain, palpitations, or any unusual symptoms. Therapy will be lifelong, and regular follow-up care is needed to monitor serum blood levels. The medication should be taken in the morning before food, and there are no dietary limitations. (Lewis et al., 10 ed., p. 1170)

An older adult client is taking digoxin 0.25 mg once a day and furosemide 40 mg daily. She states having increasing lethargy and nausea over the past 2 days, but she is still able to take her medication. Her blood pressure is 150/98 mmHg; pulse is 110 beats/min and irregular; respiratory rate is 18 breaths/min. What laboratory information is most important for the nurse to evaluate? 1. Hemoglobin, hematocrit, and white blood cell count 2. Arterial blood gases and acid-base balance 3. Blood urea nitrogen (BUN) and serum creatinine levels 4. Serum electrolyte level.

4 A low potassium level can precipitate digitalis toxicity. The client has been taking her medications and not eating regularly. She also has an irregular heartbeat and increased lethargy along with nausea, all of which can be signs of digitalis toxicity. She is taking a diuretic, which increases the excretion of potassium, and a low potassium level can cause digitalis toxicity. (Lehne, 9 ed., p. 524)

A client is experiencing a lack of logical thought progression, resulting in disorganized and chaotic thinking. The nurse understands this to be: 1. Delusions of grandeur 2. Ideas of reference 3. Depersonalization 4. Associative looseness

4 According to Eugen Bleuler's classic symptoms of schizophrenia, associative looseness is a lack of logical thought progression resulting in disorganized and chaotic thinking. Grandiose delusions are beliefs of being important. Ideas of reference or delusions of reference occur when a person believes or perceives that irrelevant, unrelated, or innocuous things in the world are referring to him or her directly or have special personal significance. Depersonalization is characterized by a change in how an affected individual perceives or experiences his or her sense of self. The usual sense of one's own reality is temporarily lost or changed. A feeling of detachment from, or being an outside observer of, one's mental processes or body occurs such as the sensation of being in a dream. (Halter, 7 ed., p. 213)

After administering diuretics to a client with ascites, which of the following nursing actions most ensures safe care? 1. Monitoring serum potassium for hyperkalemia 2. Assessing the client for hypervolemia 3. Weighing client weekly 4. Documenting accurate intake and output

4 Accurate intake and output measurements are essential for clients receiving diuretics. Hypokalemia, not hyperkalemia, is a frequent occurrence with diuretic therapy. Hypovolemia is a greater risk with an increased urine output. Clients should be weighed daily. (Lewis et al., 10 ed., p. 994)

A client has sustained a third-degree burn. What would the nurse expect to find during an assessment of the burn? 1. Area reddened, blanches with pressure, no edema 2. Blackened skin and underlying structures 3. Thick, clear blisters, underlying skin edematous and erythematous 4. Dry white, charred appearance, damage to subcutaneous tissues

4 All of the skin is destroyed in a full-thickness or third-degree burn. Often it has a dry appearance and may be white or charred and usually requires skin grafting to repair. An area reddened that blanches with pressure is indicative of a superficial first-degree burn (partial thickness). Characteristics of a full-thickness fourth degree burn include blackened skin into underlying muscle and bone structures. Thick, clear blisters, underlying skin edematous and erythematous are characteristics of a deep second-degree burn (partial thickness). (Lewis et al., 10 ed., p. 432)

The nurse is admitting a client from the post anesthesia care unit. Postoperative prescriptions include D5 1⁄2 NS with 40 mEq/L of KCl @ 100 mL/hr. The current liter of lactated Ringer's solution has 450 mL left in the bag. What should the nurse's next action be? 1. Finish the current liter of fluid at 100mL/hr. 2. Assess urine output. 3. Change the solution to D5 1⁄2 NS with 40 mEq/L KCl at 100 mL/hr. 4. Assess the IV site.

4 Although the cost-effective decision is to finish the current liter of fluid, at the prescribed rate it will take 4 hours to infuse. Because there is a loss of potassium from urinary and GI loss with surgery, it is important to begin potassium replacement upon arrival to the unit. The initial assessment of the IV site should take place prior to the infusion of potassium. Determination of kidney function and urine output is also important to establish prior to the infusion of potassium, but IV assessment is a higher priority, since it can cause injury to the client if an infiltration is present. (Lewis, et al., 10 ed., p. 282)

The nurse is caring for a client is being treated with Buck's traction. What is a priority action for the nurse? 1. Remove the traction boot every 6 hours to provide skin care. 2. Check and clean the pin sites at least three times daily. 3. Check the area around the hip where the traction is applied. 4. Verify that weights are in the amounts ordered and are hanging freely.

4 Always check the weight amounts and make sure they are not lodged against the bed or another area. There are no pin sites because Buck's traction is skin traction, not skeletal traction. The traction boot does not need to be removed as often as every 6 hours to provide skin care. (Lewis, et al., 10 ed., p. 1470)

A client is scheduled for an electroencephalogram. What will the nurse explain to the client regarding the purpose of this test? 1. Evaluates electrical currents of skeletal muscles 2. Measures ultrasonic waves in the brain 3. Determines size and location of brain activity 4. Records brain electrical activity

4 An electroencephalogram measures the brain's electrical activity. There are no ultrasonic waves in the brain. An electroencephalogram determines the electrical activity but not the brain activity, such as thought or cognition. The electroencephalogram has no relevance to skeletal muscle activity because this type of test would be an electromyogram (EMG), which measures skeletal muscle contraction and electrical potential of the muscle. (Lewis et al., 9 ed., p. 1352)

A client is worried he may have been exposed to AIDS. What will be important for the nurse to explain to this client? 1. Symptoms of AIDS will develop immediately in sexually active individuals. 2. Clients may remain asymptomatic for an indefinite period of time. 3. Symptoms of AIDS are usually seen before the client is found to be HIV-positive. 4. After exposure to the virus, symptoms may develop within 6 to 12 weeks or as late as 6 months.

4 Clients usually have symptoms within 6 to 12 weeks of exposure, but symptoms may not develop until 6 months after exposure. This is the period of seroconversion. The symptoms do not develop immediately in sexually active individuals. The client may remain asymptomatic for an undetermined period of time. The client may be HIV positive for years before he is diagnosed as having AIDS. (Lewis et al., 10 ed., p. 220)

A pregnant client had an abruptio placentae after a hemorrhagic episode; an emergency delivery was completed. The client is stable, and the cesarean delivery was performed 2 days ago. During the postoperative period, the nurse is observing for potential complications. What would be important for the nurse to assess regarding the development of complications? 1. Check the blood sugar level every 2 hours. 2. Assess vital signs hourly. 3. Place client in side-lying position. 4. Monitor fibrinogen and coagulation studies.

4 Clients with abruptio placentae are prone to the development of disseminated intravascular coagulation after delivery, which is characterized by abnormal fibrinogen and coagulation studies. Although checking vital signs is important to continue to monitor, the delivery and hemorrhagic episode has occurred. Checking blood sugar would be appropriate for a client with gestational diabetes. Side-lying position would improve placental perfusion. (Lowdermilk et al., 11 ed., p. 683)

A teenager is diagnosed with conjunctivitis. Which statement indicates that the teenager understood the nurse's teaching? 1. "I can let my friends use my sunglasses while we are together." 2. "It's okay for me to softly rub my eye, as long as I use the back of my hand." 3. "I can pick the crusty stuff out of my eyelashes with my fingers when I wake up in the morning." 4. "I will use my own wash cloth and towel for my face while my eyes are sick."

4 Conjunctivitis is contagious through physical contact. The teenager should be directed not to touch the eye, if possible. Handwashing should be stressed after any contact with the eye, and the teen should not share any items that might be contaminated, such as facial towels. Conjunctivitis may be bacterial or viral, and sharing of possibly contaminated objects may result in the spread of infection. Rubbing or picking at the eyelashes may result in further irritation or contamination of the hands, which would further the risks of infection transmission. (Lewis et al., 10 ed., pp. 371-373)

A woman has experienced an incomplete abortion. She is scheduled for a dilation and curettage (D&C). What is the purpose of this procedure? 1. To protect the uterus and the ovaries for future pregnancies 2. To provide a healthier uterine environment for future pregnancies 3. To provide reinforcement for an incompetent cervix 4. To scrape the uterine walls and remove the uterine contents

4 D&C is the dilation (opening) of the cervix and curettage (scraping) of the inner walls of the uterus to remove the uterine contents to control bleeding and, in this instance, remove any products of conception that were not expelled. It does not protect the uterus or the ovaries. It is not recommended for incompetent cervical os, nor does it promote health of future pregnancies. (Lewis et al., 10 ed., p. 1246)

The nurse is preparing discharge teaching for a client with hypertension who is being treated with furosemide and clonidine. The nurse would caution the client about which overthe-counter medications? 1. Antihistamines 2. Acetaminophen 3. Topical corticosteroid cream 4. Decongestant cough preparations

4 Decongestants and over-the-counter cough medicines frequently contain pseudoephedrine. These medications will cause an increase in blood pressure and interfere with the effectiveness of the antihypertensive medications. (Lewis et al., 10th ed., p. 697)

What is the desired action of dopamine when administered in the treatment of shock? 1. It increases myocardial contractility. 2. It is associated with fewer severe allergic reactions. 3. It causes rapid vasodilation of the vascular bed. 4. It supports renal perfusion by dilation of the renal arteries.

4 Dopamine will support renal perfusion when administered in low doses in the initial stages of shock. At higher doses and as the client becomes more decompensated, the effect of the dopamine on the renal perfusion decreases. Vasodilation would further complicate the shock situation, and allergies are not a common problem. Vasoconstriction is not a primary property of dopamine in low doses. Dopamine increases cardiac rate, but that is not the desired therapeutic action for a client in shock. (Ignatavicius & Workman, 8th ed., pp. 748; Lewis et al., 10th ed., p. 1599)

The nurse is caring for a client who has had a mastectomy. What is important nursing care regarding the positioning of the affected arm? 1. Hold the arm close against the side of her body. 2. Secure the arm below the level of the heart. 3. Wrap the arm in an elastic bandage and keep it below the heart. 4. Elevate the arm above heart level.

4 Elevating the affected arm promotes drainage of lymph from the extremity and decreases fluid from the wound site, which reduces swelling. An elastic wrap may be applied to the affected arm to reduce swelling, but it would not be positioned below the heart level. (Lewis et al., 10 ed., p. 1219)

A client is experiencing difficulty breathing, periorbital swelling, flushing, and itching. He had a diagnostic test in which an iodine-based dye was used about an hour earlier. What medication will the nurse anticipate administering immediately? 1. A bronchodilator such as aminophylline 2. A corticosteroid such as dexamethasone 3. An antihistamine such as diphenhydramine 4. An adrenergic agonist such as epinephrine

4 Epinephrine, given subcutaneously or intravenously, is the drug of choice for anaphylactic reactions. The reaction described is a mild to moderate anaphylactic reaction. The other medications listed may be used in treatment of the reaction, but epinephrine is the immediate drug of choice. (Lewis et al., 10 ed., pp. 202-203.)

The client has had a right nephrostomy tube placed after a nephrolithotomy for removal of a kidney stone. When the client returns to the room, what is a priority nursing action? 1. Irrigate the tube with 30 mL of normal saline solution four times a day. 2. Clamp the tube if drainage is excessive. 3. Advance the tube 1 inch every 8 hours. 4. Ensure that the tube is draining freely.

4 Failure of the tube to drain freely can result in pain, trauma, wound dehiscence, and infection. If an irrigation is ordered for a nephrostomy tube, no more than 5 mL of sterile normal saline should be gently instilled. (Lewis et al., 10 ed., p. 1062)

The nurse receives the new orders below for a client admitted in thyroid crisis. Which order should the nurse question? Jane Johnson MR: 96837 DOB: 6/5/1962 Allergies: NKDA Admission Orders - 5/20/19 A. Admit to hospital for thyroid crisis B. Cardiac monitor continuous C. Hyperthermia blanket PRN D. IV fluids 0.9% 50 mL/hr 3 1 liter E. Propranolol F. Propylthiouracil G. Stat T3, T4, and TSH serum level 1. IV fluids. 2. Serum blood tests. 3. Propylthiouracil. 4. A hyperthermia blanket.

4 Fever (hyperthermia) is a symptom of thyroid storm. The correct treatment would be a hypothermia blanket to cool the client. All other choices (IV fluids, laboratory tests, and propylthiouracil) are appropriate prescriptions for this diagnosis. (Lewis et al., 10 ed., p. 1165)

A client is admitted because of benign prostatic hypertrophy and is scheduled to have a transurethral prostate resection. What assessment data would indicate to the nurse that a complication is developing? 1. The client has difficulty emptying his bladder. 2. Client states he feels like he cannot empty his bladder. 3. The client complains of frequency and nocturia. 4. Increasing complaints of flank pain and hematuria.

4 Flank pain may be indicative of an infection or a ureteral obstruction causing increased pressure on the renal pelvis. Other options are symptoms of benign prostatic hypertrophy, for which he will be treated while he is in the hospital. (Lewis et al., 10 ed., p. 1268)

Which is an appropriate nursing action for a child with acute glomerulonephritis?\ 1. Initiating contact isolation precautions 2. Encouraging increased fluid intake 3. Encouraging ambulation, as tolerated 4. Providing a fluid-restricted, low-sodium diet

4 For individuals with acute glomerulonephritis, edema is treated by restricting sodium and fluid intake and by administration of diuretics. Isolation is not required because this is an autoimmune problem. Although ambulation is not incorrect for the child, it is best to encourage rest periods and focus on ways to allow the kidneys to repair and restore themselves. (Lewis et al., 10 ed., p. 1042)

A client is scheduled for a routine glycosylated hemoglobin A1c. What needs to be included in the teaching about the test? 1. Drink only water after midnight and come to the clinic early in the morning. 2. Eat a normal breakfast and be at the clinic 2 hours later. 3. Expect to be at the clinic for several hours because of the multiple blood draws. 4. Come to the clinic at the earliest convenience to have blood drawn.

4 Glucose attaches to the hemoglobin molecule of the red blood cell. A glycosylated hemoglobin test gives an average of blood glucose over the past 3 to 4 months, and a blood sample can be obtained at any time during the day. It is not used in the diagnosis of diabetes and does not need to be a fasting specimen. (Lewis et al., 10 ed., p. 1118)

A teenage boy comes to the office complaining of intense burning while urinating and gray-green discharge coming from his penis. The nurse recognizes these as symptoms of what problem? 1. Herpes (HSV-2) with open lesions 2. Secondary stage syphilis 3. Urinary tract infection 4. Gonorrhea

4 Gonorrhea in men is the most symptomatic, with urethritis, dysuria, and purulent drainage, but the disease can be asymptomatic. The purulent secretion should be cultured to identify the microorganism. HSV-2 infections are characterized by painful vesicles surrounded by an erythematous base that progress to shallow ulcers, which eventually crust and epithelialize as they heal. In the secondary stage of syphilis the client is often asymptomatic; however, a maculopapular rash on the palms of the hands and soles of the feet may be noted with lymphadenopathy, sore throat, headache, and condylomata lata (flat lesions appearing in moist areas— not to be confused with condylomata acuminata in genital warts). A lower urinary tract infection would be characterized by dysuria, urgency, frequency, hematuria, and possible low back pain. (Lewis et al., 10 ed., p. 1230)

A primigravida client at 26 weeks' gestation has been administered a glucose tolerance test. What would the nurse anticipate as a normal finding? 1. Glycosylated hemoglobin A1c of 5.0% (0.05 proportion of hemoglobin) 2. Blood glucose of 200 mg/L at 60 minutes (11.1 mmol/L) 3. 24-hour urine glucose level of 5 mg/dL (0.28 mmol/L) 4. Blood glucose level of 110 mg/L (6.11 mmol/L) at 3 hours

4 In the oral glucose tolerance test (OGTT), the blood glucose level is evaluated before the test (a fasting blood glucose), 1 hour after a 100 g glucose load, 2 hours after the glucose load, and 3 hours after the glucose load. The normal value for 1 hour is less than 180 mg/L (10.0 mmol/L), the normal value for 2 hours is less than 155 mg/L (8.6 mmol/L), and the normal value for 3 hours is less than 140 mg/L (7.8 mmol/L). The glycosylated hemoglobin value reflects blood glucose control for the past 120 days, and the urine glucose level is not as reliable an indicator of control as the serum glucose level. (Lowdermilk et al., 11 ed., p. 700)

The nurse is caring for a client who has a temporal craniotomy, and this is the first postoperative day. What is an important nursing intervention? 1. Take temperature orally only. 2. Restrain the client as necessary. 3. Suction the client every 2 hours. 4. Maintain the client with his head elevated.

4 Lowering the client's head increases ICP. Restraints should not be used because pulling against restraints will increase ICP. The client should be suctioned only as necessary. The temperature monitoring depends on the equipment; if the client is prone to seizures, an oral thermometer should not be placed in the client's mouth. (Lewis et al., 9 ed, p. 1368)

What is an important nursing action in the safe administration of heparin? 1. Check the prothrombin time (PT) and administer the medication if it is less than 20 seconds. 2. Use a 20-gauge, 1-inch (2.5 cm) needle and inject into the deltoid muscle and gently massage the area. 3. Dilute in 50 mL 5% dextrose in water (D5W) and infuse by intravenous piggyback (IVPB) over 15 minutes. 4. Use a 25-gauge, 1⁄2-inch (1.25 cm) needle and inject the medication into the subcutaneous tissue of the abdomen.

4 Medication should be administered with a small-gauge (25 gauge) needle into the subcutaneous tissue without aspirating or massaging the area. Partial thromboplastin time (PTT) is used to monitor the effects of heparin. Although heparin may be administered IV, it must be diluted in more than 50 mL D5W and would be administered over a longer period of time than 15 minutes. (Lewis et al., 10th ed., p. 820)

A client is admitted for evaluation of his permanent pacemaker. Which assessment is most concerning? 1. Pulse rate of 96 beats/min with regular rate and rhythm 2. Irregular pulse rate with premature ventricular beats 3. Atrial premature beats shown on the monitor 4. Pulse rate of 48 beats/min with premature ventricular beats

4 Most demand pacemakers are set somewhere between 60 and 72. A pulse rate of 48 is too slow for a properly functioning pacemaker. If the client's pulse rate falls below the preset pacemaker rate, the pacemaker should take over the pacing. With bradycardia, ventricular escape beats occur. A pulse rate of 96 beats/min is within normal limits. An irregular pulse rate with PVCs or atrial contractions are related to irritable foci in the heart and not indicative of pacemaker problems. (Lewis et al., 10 ed., p.775)

What are the nursing interventions regarding care of a client with a vaginal radium implant? 1. Clamp and drain the urinary retention catheter. 2. Provide a high-residue diet. 3. Place the client in a semiprivate room. 4. Raise the head of the bed no more than 20 degrees.

4 Once the implant is in place, keeping it in the exact measured position without disruption is a primary goal of care. Strict bed rest is maintained. The head of the bed should be raised only slightly to accomplish this. A urinary retention catheter is placed for gravity drainage. The client should be in a private room. Constipation should be avoided, but a high-residue diet will increase the bulk of the stool and possibly dislodge the implant. (Lewis et al., 10 ed., p. 250)

The nurse is updating a teaching plan for a client who has cancer and has been taking doxorubicin for the past several months. What is important to review with the client? 1. Report symptoms of hematuria. 2. Increase intake of oral fluids. 3. Avoid folic acid intake. 4. Report symptoms of dyspnea.

4 One of the most common and most severe toxicities of doxorubicin is cardiotoxicity. After months of treatment, this can manifest as heart failure (dyspnea, tachycardia, peripheral edema). Early side effects include dysrhythmias and electrocardiogram (ECG) changes. These can occur within hours of receiving the medication. (Lewis et al., 10 ed., p. 256)

During peritoneal dialysis treatment, the nurse continually evaluates the client for poor dialysate flow. How will this complication be identified? 1. Increased urine albumin level 2. Decreased plasma osmolality 3. An increase in sodium transfer to serum 4. Outflow is intermittent

4 Outflow should be a continuous stream after the clamp is opened (end of dwell time). Constipation, kinked or clamped connection tubing, client's position, or catheter displacement can lead to poor dialysate flow. Sodium is often evaluated once daily but not in correlation with the outflow of the dialysate. Typically, the client does not have sufficient urine output for a fractional urine, which is a urine specimen collected that has a separate examination for different solutes (glucose, acetone, etc.). Serum plasma osmolarity does not give an indication of poor outflow. (Lewis et al., 10 ed. p. 1086)

A client with a diagnosis of AIDS has developed P. jiroveci pneumonia (PJP, PCP). What will be important for the nurse to include in the nursing care plan? 1. Put a mask on the client whenever he has visitors in his room. 2. Explain to the client why he cannot go outside his room. 3. Wear a mask and gown when providing direct care to the client. 4. Wear a gown and gloves when assisting the client with personal hygiene.

4 P. jiroveci pneumonia (PJP, PCP) is not easily transmitted from an infected person to a healthy person. The pathogen is frequently dormant in the body and is reactivated when the client's immune system is significantly depressed. There is no need for airborne or droplet precautions, but standard precautions must be strictly adhered to with this client. (Lewis et al,. 10 ed., p. 222)

A client with diabetes and a right below-the-knee amputation tells the nurse that he feels pain in the amputated leg, even though the leg is gone. The nurse's response is based on what information? 1. Phantom pain is experienced by most amputees; it will resolve without pain medication. 2. The client thinks he feels pain, but it is actually a response to his denial about the amputation. 3. The nurse cannot adequately assess the pain, therefore medication cannot be given. 4. Phantom pain occurs when the nerve endings have not adjusted to the loss of the extremity, and the client should be offered pain medication.

4 Phantom limb pain is real pain for the client and is common in amputees. Phantom pain can best be controlled by pain medication. It is important to respect a client's interpretation of the experience of pain and offer him or her pain medication. (Lewis et al., 10 ed., p. 1487)

The wife of a client with COPD is worried about caring for her husband at home. Which statement by the nurse provides the most valid information? 1. "You should avoid emotional situations that increase his shortness of breath." 2. "Help your husband arrange activities so that he does as little walking as possible." 3. "Arrange a schedule so your husband does all necessary activities before noon; then he can rest during the afternoon and evening." 4. "Your husband will be more short of breath when he walks, but that will not hurt him."

4 Physical conditioning is important for clients with COPD. Activity needs to be paced so that undue fatigue does not occur. Some increase in shortness of breath with exercise is to be expected but will not damage the lungs. If the client stops exercising before an increase in shortness of breath, he will not experience a training effect. (Ignatavicius & Workman, 8 ed., p. 565)

A client with hemophilia comes to the emergency department after bumping his knee. The knee is rapidly swelling. What is the first nursing action? 1. Initiate an IV site to begin administration of cryoprecipitate. 2. Perform a type and cross-match for possible transfusion. 3. Draw blood for determination of hemoglobin and hematocrit values. 4. Apply an ice pack and compression dressings to the knee.

4 Rest, ice, compression, and elevation (RICE) are the immediate treatments to reduce the swelling and bleeding into the joint. These are the priority actions for bleeding into the joint, regardless of the cause. The other options (administer cryoprecipitate, crossmatch for blood, draw blood to check hemoglobin and hematocrit) may be done, but they are not the initial priority action. It will be important to rest the joint to prevent hemarthrosis. No weight bearing on joint until all swelling is resolved. (Lewis et al., 10 ed., p. 628)

What important teaching instructions should the nurse relay to the client before discharge following a laparoscopic cholecystectomy? 1. Avoid dietary fat for at least 1 year. 2. Avoid heavy lifting for at least 2 weeks. 3. Expect bile-colored drainage from the incision. 4. Resume all activities gradually.

4 Resuming all activities gradually is correct. A diet low in fat is usually ordered, and the client needs to avoid heavy lifting for 4 to 6 weeks. Bile-colored drainage is not to be expected postoperatively. (Lewis et al., 10 ed., p. 1009)

A nurse is planning care for a client with syndrome of inappropriate antidiuretic hormone (SIADH). What is a priority problem that the nurse should consider for the patient, based on an understanding of this condition? 1. Disturbed sleep pattern related to nocturia. 2. Risk for fall related to hypovolemia. 3. Electrolyte imbalance related to metabolic acidosis. 4. Risk for seizures related to hyponatremia.

4 SIADH occurs when excessive antidiuretic hormone (ADH) is released, even when the plasma (serum) osmolality is normal. The excess ADH increases the permeability of the renal tubules, causing reabsorption of water into the circulation. As a result of extracellular fluid expansion, serum osmolality decreases, and sodium levels decline (as a result of being diluted), leading to hyponatremia and a risk for seizures. (Lewis et al., 10 ed., p. 1106)

A client has systemic lupus erythematosus (SLE). What statement best describes this client's immune response? 1. A delayed hypersensitivity that is cell mediated 2. An immediate reaction to prior exposure 3. An immune complex that forms with antibody production 4. An immune response that no longer recognizes normal body Tissue

4 SLE is characterized as an autoimmune disorder in which the body begins to invade and destroy normal tissue. A delayed hypersensitivity is a type IV response that is characteristic of a transplant rejection or reaction to tuberculin skin testing. An immediate reaction describes a type I reaction characterized by a prior exposure to antigen. This occurs with atopic reactions and anaphylaxis. An immune complex that forms with antibody production is a type III response, which occurs with acute glomerulonephritis. (Lewis et al., 10 ed., pp. 197-200)

What is an important aspect of client teaching regarding external radiation therapy? 1. Remain isolated after treatments. 2. Fast before the treatment. 3. Schedule treatments monthly. 4. Leave skin markings between treatments.

4 Skin markings are used by the radiotherapist to delineate the exact area of the body to be irradiated. Treatments are completed in a series, depending on the location of the malignancy and the level of radiation being administered. The treatments do not require isolation, fasting, or any form of activity restriction. (Lewis et al., 10 ed., pp. 249-250)

The nurse is reviewing with a certified nursing assistant (CNA) the care for a child who is diagnosed with acquired immunodeficiency syndrome (AIDS) and has developed P. jiroveci pneumonia (PJP, PCP). Which of the following precautions would the nurse review with the CNA? 1. Strict handwashing 2. Airborne precautions 3. Contact precautions 4. Standard precautions

4 The CDC recommends standard precautions for all clients; this is particularly important for the client with AIDS. Although strict handwashing is not an incorrect response, this should be performed when caring for all clients and is a part of standard precautions. Airborne precautions are not indicated for clients with opportunistic infections such as P. jiroveci pneumonia (PJP, PCP). Protective isolation is indicated for clients who are severely immunocompromised (e.g., clients who have undergone transplants). (Lewis et al., 10 ed., p. 218)

A client is 37 weeks' gestation and is admitted to the hospital with bright red vaginal bleeding, complaining of abdominal discomfort but no contractions. After assessing the client's vital signs and determining the fetal heart rate, which is 105, what is the most important information to obtain? 1. The amount of cervical dilation that is present 2. The exact location of her abdominal discomfort 3. The station of the presenting part 4. Assess urinary output.

4 The bright red bleeding may be an indication of placenta previa or abruptio placentae, and the client is more than 36 weeks' gestation, so the nurse should anticipate and plan for an emergency cesarean delivery. Vital signs may be normal even with heavy blood loss. Decreasing urinary output may be better indicator of acute blood loss than vital signs alone. The other options are appropriate to check, but they are not of any assistance in the woman's current situation, which could progress to hemorrhage and the need for emergency surgery. (Lowdermilk et al., 11 ed., p. 681)

Which statement correctly describes suctioning through an endotracheal tube? 1. The catheter is inserted into the endotracheal tube; intermittent suction is applied until no further secretions are retrieved; the catheter is then withdrawn. 2. The catheter is inserted through the nose, and the upper airway is suctioned; the catheter is then removed from the upper airway and inserted into the endotracheal tube to suction the lower airway. 3. With suction applied, the catheter is inserted into the endotracheal tube; when resistance is met, the catheter is slowly withdrawn. 4. The catheter is inserted into the endotracheal tube to a point of resistance, and intermittent suction is applied during withdrawal.

4 The catheter must be advanced to an adequate depth (to prevent secretion buildup at the end of the tube and to clear the airway as much as possible). To minimize trauma, suction is applied only during catheter withdrawal. If the upper airway is suctioned, another sterile catheter must be obtained to suction the endotracheal tube. (Ignatavicius & Workman, 8 ed., pp. 525-526)

A school-age child with a diagnosis of celiac disease asks the nurse, "Which foods will make me sick?" Which food items would the nurse teach the child to avoid? 1. Rice cereals, milk, and tapioca 2. Corn cereals, milk, and fruit 3. Corn or potato bread and peanut butter 4. Malted milk, white bread, and spaghetti

4 The child with celiac disease will need a gluten-free diet, eliminating foods such as pastas and breads that are made from wheat or dessert foods made from malt whey. Remember BROW—barley, rye, oats, and wheat. Foods that would be appropriate include rice and corn cereals, milk, corn and potato breads, tapioca, peanut butter, and honey. (Lewis et al., 10 ed., p. 967)

A client has returned to the room from the postoperative recovery area. He is lethargic but responsive. He has O2 via nasal cannula at 4 L/min and an IV infusing at 125 mL/hr. On the initial nursing assessment, the nurse notes that the O2 saturation is 82%. What is the priority nursing action? 1. Perform a complete neurologic check. 2. Increase the O2 flow and recheck the pulse oximetry. 3. Suction the client and recheck the vital signs. 4. Stimulate the client to cough and deep breathe.

4 The client is lethargic from the anesthetic and needs to be stimulated to deep breathe to facilitate ventilation. The client is at increased risk for development of respiratory acidosis and hypoxemia. Stimulation should be done before suctioning to determine whether it relieves the problem. Increasing the oxygen flow does not address the problem of hypoventilation. Neurologic checks can be done but are not a priority at this time. (Lewis et al., 10 ed., p. 288)

The client is receiving an IV of 0.9% NaCl at 125 mL/hr. The client had a colon resection this morning. He has a nasogastric tube to suction and an ileostomy, and he is becoming increasingly restless. The nurse reviews the serum laboratory values. Which value should the nurse consider a priority? 1. Blood urea nitrogen 28 mg/dL (10 mmol/L). 2. Serum glucose 155 mg/dL (8.6 mmol/L). 3. Hemoglobin 13.5 mmol/L, hematocrit 41% (0.41). 4. Sodium 155 mEq/L (mmol/L).

4 The client is losing fluids, but the replacement fluid only contains sodium. The laboratory value of 155 mEq/L (mmol/L) of sodium indicates that the client has hypernatremia. This increases retention of fluids and subsequently increases the cardiac workload. The glucose level is elevated, but that is not unusual in clients in the immediate postoperative period. The hemoglobin and hematocrit values are within the normal ranges. The blood urea nitrogen (BUN) level is elevated, and this needs to be investigated and correlated with the serum creatinine level. However, it is not alarmingly high and could be an indication of decreased fluids, and it is not the priority concern. (Lewis et al., 10 ed., p. 292)

During a well-woman physical examination, a young woman is diagnosed with a primary genital herpes lesion. When completing the client's history, the nurse would anticipate the client to report: 1. Purulent urethral discharge 2. Diffuse red rash 3. Generalized abdominal pain 4. Painful urination

4 The client usually seeks treatment for a variety of symptoms, including pain on urination from urine touching painful ulcerations. A purulent urethral discharge would be seen in STDs such as gonorrhea. A diffuse red rash might be caused by syphilis, and generalized abdominal pain might be caused by a variety of reproductive disorders but not genital herpes. (Lewis et al., 10 ed., p. 1232)

The nurse is caring for a client in labor. How are contractions timed? 1. End of one to the beginning of the next 2. Beginning of one to the end of the next 3. End of one to the end of the next 4. Beginning of one to the beginning of the next

4 The correct method of timing contractions is from the beginning of one contraction to the beginning of the next. The point at which the contraction ends is of concern only when the nurse needs to know the duration of the contraction, not the frequency or timing of the contractions. (Lowdermilk et al., 11 ed., p. 440)

After examining a painless sore on the penile shaft, the doctor asks the nurse to order a fluorescent treponemal antibody absorption (FTA-ABS) test. The nurse knows that the purpose of this test is to diagnose what condition? 1. Herpes simplex virus type 2 (HSV-2) 2. Trichomoniasis 3. Cytomegalovirus 4. Syphilis

4 The fluorescent treponemal antibody absorption test is a serum blood test used to identify the spirochete Treponema pallidum, which causes syphilis. HSV-2 is diagnosed by tissue culture from a specimen obtained from an active lesion and by a serologic blood test. Trichomoniasis is diagnosed by a wet mount slide obtained from vaginal or penile secretions. Cytomegalovirus is diagnosed by blood test and by urine and tissue culture. (Lewis et al., 10 ed., p. 1235)

What will the nurse identify as the goal of treatment for a client with chronic renal insufficiency? 1. Increase the urine output by increasing liver and renal perfusion. 2. Prevent the loss of electrolytes across the basement membrane. 3. Increase the concentration of electrolytes in the urine. 4. Maintain present renal function and decrease renal workload.

4 The goal in chronic kidney disease is to prevent acute failure and maintain whatever function is left. This is best done by minimizing stress and the workload on the kidneys. Increasing liver perfusion does not affect renal function. Until the renal function improves, there will be increased permeability in the basement membrane and electrolyte concentration in the urine will be diminished because solutes are being retrained rather than excreted. (Lewis et al., 10 ed., p. 1083)

A child is scheduled for a myringotomy with placement of tympanostomy tubes. What is the long-term goal of this procedure that the nurse will discuss with the parents? 1. To decrease pressure on the tympanic membrane 2. To irrigate the eustachian tube 3. To correct a malformation in the inner ear 4. To prevent recurrent ear infections

4 The goal of a myringotomy is to allow draining of the fluid within the ear that will help prevent recurrent ear infections. It will decrease the pressure immediately, but this is not the long-term goal. It neither corrects a malformation in the inner ear nor provides a way to irrigate the eustachian tube, and you do not want excessive fluid in the middle ear. (Lewis et al., 10 ed., p. 385)

The nurse understands the key to managing the therapeutic regimen and client compliance for an adolescent girl recently diagnosed with scoliosis includes consideration of which of the following? 1. The ability of the parents to afford the expenses of the braces and surgical procedures 2. The adolescent's understanding the importance of wearing the brace 3. The ability of the parents to control and enforce compliance with the therapeutic regimen 4. The psychological needs and developmental stage to encourage compliance by the adolescent

4 The identification of scoliosis as a "deformity," in combination with unattractive appliances and the potential of a surgical procedure, can have a negative effect on the already fragile adolescent's body image. The adolescent and family require excellent nursing care to meet not only physical needs but also psychological needs associated with a diagnosis of scoliosis. (Hockenberry, 10th ed., p. 1671)

The nurse would identify which situation as an indication for the administration of Rho(D) immune globulin? 1. A woman who has been Rh sensitized in the past two pregnancies 2. An infant with increased hemolysis of red blood cells because of ABO incompatibility 3. An infant with an increase in serum bilirubin levels as a result of the presence of Rh factor antibodies 4. A primigravida who is Rh negative is pregnant with an infant who is Rh positive

4 The medication Rho(D) immune globulin is given to prevent maternal sensitization to the Rh antibodies to women who are Rh negative. Rho(D) immune globulin will not prevent or treat the problem if it has already occurred. Rho(D) immune globulin is not given to the infant. (Hockenberry & Wilson, 10 ed., p. 322)

The nurse is verifying whether to give a medication to a client. What would be the first nursing action? 1. Check the client's name and hospital number. 2. Validate the expiration date of the drug. 3. Determine the appropriate route of delivery. 4. Review the orders on the medication administration record.

4 The medication administration record should be reviewed for current physician's orders. Medication administration records are routinely reviewed to determine the currency of medication orders. The client's name and hospital number are checked to validate client identification before administration of the medication. The nurse should check the medication for the expiration date when she prepares the medication, and the physician will order the route of administration. (Potter & Perry, 9 ed., p. 626.)

The best way for the nurse to maintain the safety of the newborn in the hospital is to: 1. Have the mother come to the nursery to pick up the baby for feedings. 2. Take the baby to the mother's room for rooming-in. 3. Ask the mother her name and social security number. 4. Compare the name band information of the mother and baby.

4 The mother and baby have identification bands secured to a wrist or ankle in the delivery room. The nurse should compare these every time the baby is returned to the mother and when the infant is prepared for discharge. The other options are incomplete and will not ensure the safety of the baby. (Hockenberry & Wilson, 10 ed., p. 268)

Which nursing action would be most effective in preventing venous stasis in the postoperative surgical client? 1. Raise the foot of the bed for 1 hour, then lower it to stimulate blood flow. 2. Massage the lower extremities every 6 hours. 3. Facilitate active range of motion of the upper body to stimulate cardiac output. 4. Help the client walk as soon as permitted and as often as possible.

4 The postoperative client has decreased mobility, which may create an environment in which clotting can be caused by venous stasis. Active exercise, such as having the client ambulate as soon as possible, will stimulate circulation and venous return. This reduces the possibility of clot formation. The lower extremities should not be massaged because this may disrupt a clot and cause a pulmonary embolism. (Ignatavicius & Workman, 8th ed., pp. 730-731)

The nurse is caring for a client who has hypersplenism. What laboratory test finding would indicate that the client has splenomegaly? 1. Presence of Reed-Sternberg cells 2. Elevated red blood cell count 3. Increased Bence-Jones protein in urine 4. Presence of Howell-Jolly bodies in a blood smear

4 The presence of pitted or packed RBCs or Howell-Jolly bodies in a peripheral blood smear is diagnostic of splenomegaly. The presence of Reed-Sternberg cells in a lymph node biopsy specimen is diagnostic for Hodgkin disease. Elevated Bence-Jones protein in the urine is found in multiple myeloma. With hypersplenism, you would find cytopenia characterized by anemia, leukopenia, and thrombocytopenia. An elevated RBC, WBC, or platelet count would occur after a splenectomy. (Lewis et al., 10 ed., p. 647)

The client with COPD is to be discharged home while receiving continuous oxygen at a rate of 2 L/min via cannula. What information does the nurse provide to the client and his wife regarding the use of oxygen at home? 1. Because of his need for oxygen, the client will have to limit activity at home. 2. The use of oxygen will eliminate the client's shortness of breath. 3. Precautions are necessary because oxygen can spontaneously ignite and explode. 4. Use oxygen during activity to relieve the strain on the client's heart.

4 The primary purpose of oxygen therapy is to decrease the workload of the heart in clients with chronic pulmonary diseases and to assist in preventing right-sided heart failure. Use of oxygen may help relieve shortness of breath but will not eliminate it. Oxygen supports combustion but is not explosive; supplemental oxygen will allow more activity for the client, not less. (Ignatavicius & Workman, 8 ed., pp. 515)

The nurse is admitting a client with type 1 diabetes. What values on the arterial blood gases would indicate the client is developing a complication because of his poorly controlled diabetes? 1. Paco2 48 mmHg, pH 7.34, Pao2 98 mmHg, HCO3 24 mEq/L (mmol/L). 2. Paco2 33 mmHg, pH 7.48, Pao2 88 mmHg, HCO3 26 mEq/L (mmol/L). 3. Paco2 40 mmHg, pH 7.45, HCO3 32 mEq/L (mmol/L), O2 saturation 90%. 4. Paco2 38 mmHg, pH 7.31, HCO3 20 mEq/L (mmol/L), base excess 22.

4 This series of values (Paco2 38 mmHg, pH 7.31, HCO3 20 mEq/L [mmol/L], base excess 22) best represents metabolic acidosis, a complication of type 1 diabetes, which is the correct answer. This series of values (Paco2 48 mmHg, pH 7.34, Pao2 98 mmHg, HCO3 24 mEq/L [mmol/L]) reflects respiratory acidosis; notice the elevated Paco2 and decreased pH. This series of values (Paco2 33 mmHg, pH 7.48, Pao2 88 mmHg, HCO3 26 mEq/L [mmol/L]) reflects respiratory alkalosis; notice the decreased Paco2 and increased pH. This series of values (Paco2 40 mmHg, pH 7.45, HCO3 32 mEq/L [mmol/L], O2 saturation 90%) reflects metabolic alkalosis; notice the elevated Bicarbonate and pH. (Lewis et al., 10 ed., p. 289)

In discharge planning for the client with heart failure, the nurse discusses the importance of adequate rest. What information is most important? 1. A warm, quiet room is necessary. 2. Bed rest promotes venous return. 3. A hospital bed is necessary. 4. Adequate rest decreases cardiac workload.

4 To help decrease pulmonary congestion and dyspnea, the nurse should encourage adequate rest to decrease cardiac workload; the client should not exert himself to the point of fatigue. Bed rest does promote venous return, but that is not the purpose of bed rest in the client with heart failure. A hospital bed is not necessary, and a quiet room is important if that is what promotes rest for the client. (Lewis et al., 10 ed., p. 752)

The nurse is caring for a client who is scheduled for a gastric endoscopy. Which of the following actions must the nurse perform before the client is able to eat or drink after the endoscopy? 1. Check oxygen saturation. 2. Give small sips of water. 3. Check all vital signs. 4. Assess the client's gag reflex.

4 Topical sedation during endoscopy helps block the gag reflex and numbs the esophagus, eliminating the discomfort of the tube. The nurse would know the dangers of allowing the client to eat or drink before the sedation has lost its effect. The nurse will test for the return of the client's gag reflex before allowing sips of water to be taken, to avoid aspiration. (Lewis et al., 10 ed., p. 849)

The nurse is assessing a client 12 hours after a prolonged labor and delivery. What assessment data would cause the nurse the most concern? 1. Oral temperature of 100.6°F (38.1°C) 2. Moderate amount of dark red lochia 3. Episiotomy area bruised with small amount of dark bloody drainage 4. Uterine fundus palpated to the right of the umbilicus

4 Uterus palpated to right of umbilicus may indicate a full bladder. The fundus should be at the level of the midline. The temperature, lochia, and episiotomy assessment findings are within normal limits. (Lowdermilk et al., 11 ed., p. 476)

An obese 44-year-old woman with a history of chronic cholecystitis is to receive vitamin K before surgery. What is the purpose of this medication? 1. To increase the digestion and utilization of fats 2. To support the immune system and promote healing 3. To aid in the emptying of bile from the gallbladder 4. To facilitate coagulation activities of the blood

4 Vitamin K is necessary for normal clotting. Cholecystitis can decrease the absorption of fat-soluble vitamins (A, D, E, and K) by interfering with fat metabolism, which can lead to potential difficulties with clotting. (Lewis et al., 10 ed., pp. 992, 1008)

An older adult client has a prescription for continuous fluid replacement at 75 mL/hr. The nurse is preparing to start the IV. Which option would be best? 1. A 22-gauge butterfly needle, right arm antecubital area. 2. An 18-gauge, 3-inch IV cannula, inserted in the left hand. 3. An 18-gauge, 1-inch IV cannula, in the antecubital area of left arm. 4. A 22-gauge, 1-inch IV cannula, top of the left hand.

4 With a continuous flow at 75 mL/hour, a small-gauge IV cannula (22 G), 1 inch (2.5 cm) is appropriate. Butterfly needles are used for short-term infusions and drawing blood. IVs should be started in the lowest vein possible and progress upward. The antecubital area is not a preferred area. (Potter & Perry, 9 ed., p. 958)

The physician calls the unit and leaves an order for a client. The order is for cefaclor 0.1 gm PO. The dose available in the unit is 125 mg/5 mL. How many milliliters will the nurse give? Answer: __________ mL

4 mL Rationale: 1 gm = 1000 mg; therefore, 0.1 gm = 100 mg 125 mg : 5 mL :: 100 mg : x mL

When caring for a client with diabetes insipidus, which assessment changes require a priority action? Select all that apply. 1. Urine output change from 270 mL/hr to 100 mL/hr. 2. Finger stick glucose of 182 mg/dL. 3. Weight decrease of 1 kg overnight. 4. Urine becoming paler in color. 5. Serum osmolality of 300 mOsm/kg

4, 5 Diabetes insipidus (DI) is associated with a decrease (or deficiency) in the secretion of antidiuretic hormone (ADH). Lack of ADH leads to increased urinary output (as much as 5-20 L/day). A concern would be demonstrated if the urine becomes more dilute, increases in quantity, blood pressure drops, or the blood becomes more viscous. (Lewis et al., 10 ed. p. 1160-1161)

The nurse identifies which problems as risk factors for the development of a sickle cell crisis? Select all that apply. 1. Recurrence of acute otitis media 2. A fall with swelling at the kneecap and joint 3. Fractured radius requiring internal fixation 4. Recurrence of respiratory tract infection 5. Traveling to a location of higher altitude 6. Dehydration

4, 5, 6 Recurrence of respiratory tract infections is associated with an increase in sickling crisis, not an acute case of otitis media. Respiratory infections generally involve fever, coughing, malaise, and anorexia. These factors contribute to dehydration. Dehydration may also precipitate an attack as the blood is thicker and more prone to clotting. Problems with oxygen saturation are also associated with sickling crisis, thus traveling to a higher altitude where there is less oxygen may precipitate a crisis. Injuries to joints or fractures needing surgical repair do not lead to a sickle cell crisis. (Lewis et al., 10 ed., p. 617)

To meet the goal of promoting infant feeding in a breastfed baby, the nurse should teach the mother to do which of the following? Select all that apply. 1. Feed the baby on a 3- to 4-hour schedule. 2. Alternate breast milk and formula for each feeding. 3. Stop breastfeeding if her nipples get sore. 4. Maintain on-demand breastfeeding for the first 4 weeks. 5. Drink lots of fluids and get adequate rest. 6. Offer a pacifier between feedings to satisfy sucking needs.

4, 5, 6 The mother should be taught to feed the baby on demand for at least the first 4 weeks until lactation is well established. Feeding only breast milk frequently stimulates milk production. Nipple soreness is one of the most common problems, but the use of a cream to soften the nipples is often helpful, as is offering a pacifier to satisfy sucking needs of the infant between feedings after breastfeeding has been established. Adequate rest and good fluid intake help promote milk production. (Hockenberry & Wilson, 10 ed., p. 279)


Conjuntos de estudio relacionados

5-10: Starch Hydrolysis (Amylase Test) Microbiology Homework

View Set

Chapter 9: Market Failure: Externalities and Healthcare

View Set

Chapter 12 - Lender Loan Process and Calculations for SC, Unit 2 : More on Qualifying the Borrower

View Set

Chapter 6: Skeletal System Bones and Bone Tissues

View Set

Principles of Management Final Exam 2020

View Set

Chapter 14: Somatosensory Function, Pain, Headache, and Temperature Regulation

View Set

Critical Thinking Questions (Chapter 4)

View Set

Accounting 2100 Pre Final quiz 2

View Set

Lecture 7 - Ch. 5 Access Control

View Set

United States and Capitals with pictures

View Set